Lots of Microbiology questions

Lakukan tugas rumah & ujian kamu dengan baik sekarang menggunakan Quizwiz!

Almost all bacterial intestinal infections may be attributed to: a. Vibrio spp. b. C. jejuni c. Salmonella spp. d. Escherichia spp. E. All of the above

- All of the above

Which is not a component of innate immunity? - Skin - Inflammation - Fever - Antibody

- Antibody

Helicobacter pylori is able to survive in the stomach by its ability to produce neutralizing proteins from glucans. fatty acids from sebum. lactic acid from sugar. ammonia from urea. ammonia from urease.

- ammonia from urea. • H. pylori survives acidic environment of stomach - Produces urease, which converts urea (from proteins) to ammonia, creates alkaline microenvironment

What do hepatitis A and hepatitis B have in common? Both often result in a mild disease. Both can be prevented by a vaccine. Both can result in a carrier state. Both are transmitted via the fecal-oral route.

- both can be prevented by a vaccine No carrier state in HepA, fecal-oral is only HepA and HepB ranges from asymptomatic to severe.

Which are the three ways in which fungi cause illness in humans? 1. Hypersensitivity - person has allergic reaction to fungal components. 2. Intoxication - fungus produces a toxin that is ingested. 3. Immune suppression - fungus destroys all red blood cells. 4. Infection - fungus grows on body as a mycosis.

1, 2, and 4

You are a nurse on a post-surgical ward. One of your patients is an elderly woman who has just had her gallbladder removed. When you go into her room to check on her, you find that she has significant...

5 of these... Gallbladder was originally spelled gallbaldder.

Effective treatment of gas gangrene primarily involves: - use of an antitoxin. - use of immune globulins. - vaccination with inactivated toxin. - surgical removal of dead and infected tissues.

- surgical removal of dead and infected tissues. Clostridial Myonecrosis (Gas Gangrene) slide 35 Treatment and Prevention • Prompt surgical removal of all dead and infected tissue • Antibiotics, hyperbaric oxygen treatment • No available vaccine

For growth, most bacteria require water activity levels above 1.00. 0.80. 0.70. 0.90

0.90 - which explains why fresh, moist foods spoil more quickly than dried, sugary or salted foods. Fungi can grow at a water activity as low as 0.80, so forgotten bread, cheese, jam and dried foods often become moldy.

The animal(s) often associated with Salmonella strains is/are: turtles. ducks. baby chickens. iguanas. All of the choices are correct.

All of the choices are correct p.644 Correct

All cases of paralytic polio acquired in the U.S. between 1980 and 1999 were caused by Salk's polio vaccine. True False

False - from Sabin vaccine

Oligotrophic waters are usually hypoxic. T/F

False - these waters have few nutrients - not in danger of going hypoxic.

Coagulase is an enzyme. T/F

False But online resources say it is a protein enzyme. Maybe here it is counting it as a virulence factor instead?

How do cytokines function?

They are secreted by one cell type in the vicinity of another cell. They then bind to a receptor on the 2nd cell, causing a signal within that cell that turns on (or off) certain genes to achieve a response.

What is the significance of immune complex formation in infective endocarditis? They lead to the formation of biofilms that are difficult to eliminate. They cause disseminated intravascular coagulation. They result in the enzymatic destruction of neutrophils and macrophages. They lead to the development of autoimmunity. They can be deposited in tissues and organs, causing damage.

They can be deposited in tissues and organs, causing damage. p.667 Correct

Promotes protein coagulation in fermenting milk

Rennin

Reovirus (diarrhea)

Reoviridae RNA virus

Rotavirus (respiratory infections; gastroenteritis)

Reoviridae RNA virus

In which of the following does a rash start on the palms and soles and progress toward the trunk? A) Epidemic typhus B) Typhoid C) Measles D) Impetigo E) Rocky Mountain spotted fever

Rocky Mountain Spotted Fever Rash from blood leakage appears on palms, wrists, ankles, soles; spreads up arms and legs to rest of body.

Defensins are short antimicrobial peptides found within mucus membranes and phagocytes.

True

Fungi are able to handle acidic environments much better than bacteria. T/F

True

Muscles aid the flow of venous blood. T/F

True

Mycobacterium leprae is only grown in living cells. T/F

True

Pattern recognition is involved in innate immunity.

True

Plasmodium falciparum causes the most serious form of malaria. T/F

True

Staphylococci are the leading cause of wound infections. T/F

True

Streptobacillus moniliformis is unusual in that it spontaneously forms L-forms. T/F

True

Streptomyces produces geosmins which give soil a characteristic musty odor. T/F

True

Sucrose is one of the major contributors to the development of dental caries. T/F

True

T/F Most of the medically important multicellular parasites are arthropods or helminths.

True

T/F Typically arthropods serve as vectors of disease while helminths directly cause disease.

True

T/F - All pathways of complement activation follow the same sequence after C3.

True

T/F - Defensins are short antimicrobial peptides found within mucous membranes and phagocytes.

True

T/F - Pattern recognition is involved in innate immunity.

True

Trickling filters may be used in place of activated sludge in secondary sewage treatment. T/F

True

Virulent strains of Shigella typically carry an R plasmid. T/F

True

White blood cells called leukocytes are important in immunity.

True

The toxin produced by Staphylococcus aureus is heat-stable. T/F

True - heat stable and survives cooking

The major virulence factors of Yersinia pestis are carried on a plasmid. T/F

True Apparently on 3 of them.

H. pylori is able to survive the acidic environment of the stomach as a result of adaptations that allow it to increase the pH of the immediate environment. T/F

True INCREASE the pH means make it less acidic.

Complications of measles may include pneumonia and encephalitis. T/F

True Often secondary infections lead to earaches, bacterial pneumonia. Very rarely, 2-10 years later, measles followed by subacute sclerosing panencephalitis (SSPE)

Fungi capable of dimorphism grow either as

Yeast-like or mycelium

Plauge

Yersinia gram - bacteria

The patient is non-infectious in the third stage of syphilis. T/F

True p.747 Table 27.9 Primary stage - infectious Secondary stage - infectious Latent stage - early infectious, late non-infectious. Tertiary stage - non-infectious

The most notorious typhoid carrier was: A. Typhoid Tilly. B. Typhoid Tom. C. Typhoid Mary. D. Typhoid Mark.

Typhoid Mary

Lactic acid bacteria such as Lactococcus

are obligate fermenters

The methanogens

are part of the Domain Archaea AND oxidze hydrogen gas to produce methane

Pseudomonas

are resistant to many disinfectants and antimicrobials AND are mostly harmless except for the opportunistic P. aeruginosa.

Haustoria

are specialized hyphae used by parasitic fungi.

Mycorrhizae

are vital for the survival of many plants AND increase the absorptive ability of roots.

M. leprae may infect birds. iguanas. skunks. raccoons. armadillos.

armadillos

Wolbachia are found only in

arthropods

children 5-15 who have chickenpox and take _____ are at greater risk for developing Reye's syndrome.

aspirin Reye's syndrome is a rare but serious disease that causes swelling in the liver and brain. It can affect people of any age, but it is most often seen in children and teenagers recovering from a virus such as the flu or chickenpox. - teenagers recovering from viral infections should avoid taking aspirin

Turning inorganic carbon into organic carbon is called mineralization. carbon fixation. eutrophication. respiration.

carbon fixation

The enzyme ____ catalyzes the conversion of peroxide to water and oxygen.

catalase. Superoxide dismutase deals with the superoxide anion of the reactive oxygen species.

The diarrhea of cholera has been described as a viscous fluid. small in volume. somewhat watery. a rice water stool.

d. a rice water stool.

Which characteristic(s) is/are NOT common to C. tetani and C. perfringens?

got this right! further up i put it

Mumps is a good candidate for elimination from the population due to: - the existence of an effective vaccine. - a human only reservoir. - the absence of a latent state. - a single serotype. - All of the choices are correct.

- All of the choices are correct. Remember the MMRV/MMR vaccine

The infectious agent(s) that may arise in a wound from a human bite is/are: A. Escherichia coli. B. Bacteroides. C. Actinomyces israelii. D. Staphylococcus aureus. E. Bacteroides AND Staphylococcus aureus.

- Bacteroides AND Staphylococcus aureus Human Bites slide 40, p.615 Causative Agents • Aerobic and anaerobic members of normal microbiota including streptococci, fusiforms, spirochetes, Bacteroides species, often in association with Staphylococcus aureus (on my chapter quiz - correct)

Cat scratch fever is caused by: - Bartonella (Rochalimaea) henselae. - Pasteurella multocida. - Teddis nugentaea. - Staphylococcus aureus. - Bartonella (Rochalimaea) henselae AND Staphylococcus aureus.

- Bartonella (Rochalimaea) henselae. slide 45 zoonosis of cats Causative Agent is: • Bartonella henselae: curved, Gram-negative rod

Toxoplasma

invovles cats, fetus can be infected if a woman gets this disease during pregancy -protozoan diseases

Rickettsia rickettsii is an obligate intracellular pathogen. This means that: A) it is a virus. B) it needs oxygen for growth. C) it cannot grow in the presence of oxygen. D) it can only grow within host cells. E) it is non-motile.

it can only grow within host cells. Rickettsia rickettsii: tiny Gram-negative non-motile coccobacilli that are OBLIGATE INTRACELLULAR BACTERIA - Difficult to grow in culture.

Protozoan classification used to be based on their means of

locomotion

The water activity in foods that have high levels of salt or sugar is high. dense. low. osmotic

low It may have a lot of moisture but that water is chemically tied up by the sugar or salt and is not available for use by the spoilage bacteria.

Effective treatment of wastewater/sewage is reflected in a(n) increase in sulfur. lower BOD. higher BOD. unchanging BOD. decrease in biomass.

lower BOD

The cells responsible for adaptive immunity are the: A) lymphocytes. B) monocytes. C) eosinophils. D) neutrophils. E) leukocytes.

lymphocytes - B and T cells

Egg white is rich in the antimicrobial hydrogen peroxide. lysosome. benzoic acid. lysozyme. allicillin.

lysozyme

Shigella and cholera toxin both: have an A-B arrangement. work through ADP ribosylation. increase cAMP levels. prevent protein synthesis.

have an A-B arrangement.

Pregnant women with bacterial vaginosis are at risk of: placental insufficiency. being unable to nurse. being unable to deliver vaginally. having a premature baby.

having a premature baby p.735 - BV is common in pregnant women and puts them at risk for premature deliveries.

All of the following are eukaryotic organisms that cause diarrheal disease EXCEPT: A) Cryptosporidium. B) Cyclospora. C) Entamoeba. D) Giardia. E) Campylobacter

- Campylobacter Campylobacter is a spirally curved, Gram-negative rod that needs be grown under microaerophilic conditions.

Microscopic examination of a patient's fecal culture shows spiral bacteria. The bacteria probably belong to the genus: A) Campylobacter jejuni. B) Escherichia coli. C) Salmonella typhi. D) Shigella spp. E) Vibrio cholerae

- Campylobacter jejuni - C. jejuni is a spirally curved, Gram negative rod. It can be cultivated from feces under microaerophilic conditions using a selective medium to suppress the growth of other intestinal organisms.

Which of these bacteria require a special medium and microaerophilic conditions? Escherichia coli Pseudomonas aeruginosa Campylobacter jejuni Salmonella enterica Staphylococcus aureus

- Campylobacter jejuni p.646 Correct

Which of the following has not been associated with human bites? - syphilis - tuberculosis - hepatitis B - Haverhill fever

- Haverhill fever slide 40 Human Bites • Common; can result in serious infections from normal mouth microbiota; occasionally syphilis, tuberculosis, hepatitis B, hepatitis C transmitted

The foodborne form of rat bite fever is called: - rat bite fever. - Haverhill fever. - cat scratch fever. - Lyme fever.

- Haverhill fever.

Which of the following causes inflammation of the liver? A) Salmonella enterica B) Shigella spp. C) hepatitis A virus D) Vibrio cholerae E) Escherichia coli

- Hepatitis A Virus All the hepatitis viruses

The member of the normal microbiota sometimes considered responsible for tinea versicolor is: A) Staphylococcus species. B) diphtheroids. C) Candida species. D) Malassezia species. E) Streptococcus species.

- Malassezia species. Fungal diseases: Malassezia furfur generally harmless, commonly found • Can cause scaly rash, dandruff, or tinea versicolor - TINEA VERSICOLOR characterized by patchy scaliness and increased pigmentation of light skin or decreased pigmentation of dark

Which of the following normal skin flora is a small yeast? A. staphylococci B. diphtheroids C. Candida spp. D. Malassezia spp.

- Malassezia spp. - MALASSEZIA: tiny lipid-dependent yeasts We already established that Malassezia are not diphtheroids, they are not bacteria like staphylococci (G+) and Candida is a yeast but it's not supposed to be on the skin normally.

chapter quiz - second try

one wrong, noted, all rest correct.

Why do patients with leptospirosis sometimes undergo unnecessary surgery?

p.733 - Severe pain is characterisitc of the first (septicemic) phase, and patients sometimes undergo unnecessary surgery because the pain is mistaken for a symptom of appendicitis or gallbladder infections.

Why is tertiary syphilis difficult to treat?

p.746 - tertiary syphilis occurs after a latent period that last for years! It affects the body systemically - gummas anywhere, cardiovascular or neurosyphilis...p.747 - the organisms may be present in almost any part of the body.

Algae are important environmentally as

major producers of oxygen

Algae are important environmentally as

major producers of oxygen.

Yersinia pestis typically contains one plasmid. two plasmids. three plasmids. four plasmids.

three plasmids.

Poultry products are a likely source of infection by: A) Helicobacter pylori. B) Salmonella enterica. C) Vibrio cholerae. D) Shigella spp. E) Clostridium perfringens

- Salmonella enterica

Dental plaque is an example of: A) a pellicle. B) a biofilm. C) gingivitis. D) dentin. E) tooth enamel

- a biofilm Bacteria come in and make a biofilm on the pellicle (made of proteinaceous material from saliva on the enamel).

The passage from the mouth to the anus is termed the: gut canal. oral cavity. grand canal. alimentary canal.

- alimentary canal Also called the Gastrointestinal tract (saw this question too, same but GI tract instead of alimentary canal.)

Which of the following virulence factors used by Staphylococcus puts holes in host cells? A) Protein A B) Alpha toxin C) Leukocidin D) Clumping factor E) Membrane attack complex

- alpha toxin Table 22.3 Alpha toxin - makes holes in host cell membranes.

The symptoms of cholera are due to the action of: an endotoxin. modified mucus. an exotoxin flagella.

- an exotoxin. cholera toxin, an A-B toxin - B portion attaches to receptors of microvilli - A portion enters cells, activates a G protein that turns on adenylate cyclase to convert ATP to cAMP - High cAMP causes cell to secrete chloride ions - Sodium and other ions follow, and water follows the salts, yielding outpouring from cells - Toxin does not affect large intestine, but volume of fluid is too much to be absorbed, causing diarrhea - Toxin encoded by bacteriophage: lysogenic conversion

The most serious consequence of rubella is: A) encephalitis. B) birth defects. C) meningitis. D) deafness. E) subacute sclerosing panencephalitis

- birth defects.

Factor(s) not found in abscesses is/are: A. pus. B. dead leukocytes. C. tissue remnants. D. blood vessels.

- blood vessels Localized collection of pus surrounded by inflamed tissue. • Pus: thick, yellowish fluid composed of living and dead leukocytes, tissue debris, and proteins

Which of the following are involved in coating Staphylococcus with host proteins? A. clumping factor B. coagulase C. protein A D. leukocidin E. clumping factor, coagulase AND protein A

- clumping factor, coagulase AND protein A Clumping factor - attaches the bacterium to fibrin, fibrinogen and plastic devices. Coagulase - produces clots in the surrounding capillaries protein A - binds to Fc portion of antibody, prevents opsoniziation Correct

. Staphylococci can be divided into pathogenic and relatively nonpathogenic strains based on whether or not they synthesize _________. A. catalase B. coagulase C. leukocidins D. a capsule

- coagulase Coag-positive is S. aureus Coag-negative is the other Staphylococcus species, including S. epidermidis.

Wound infections may result in: A. delayed healing. B. abscess formation. C. extension of bacteria or their products into surrounding tissues or bloodstream. D. aerobic conditions. E. delayed healing, abscess formation AND extension of bacteria or their products into surrounding tissues or bloodstream.

- delayed healing, abscess formation AND extension of bacteria or their products into surrounding tissues or bloodstream.

A common source of cholera infection is - acid rain. - unpasteurized milk. - fecal contaminated material, especially water. - boiled water. - acid rain AND boiled water.

- fecal contaminated material, especially water. Correct

The disease most feared to develop after an animal bite is: - tetanus. - rabies. - gas gangrene. - actinomycosis

- rabies (on my chapter quiz - correct)

Tetanus prevents the release of neurotransmitters from: - muscle cells. - excitatory neurons. - inhibitory neurons. - tetano cells.

- inhibitory neurons. slide 29 Prevents release of neurotransmitter from INHIBITORY NEURONS, so muscles contract without control Correct

Single cell protein -refers to the use of single-celled organisms as a protein source. - refers to a single-celled protein produced by several different organisms. - is an enzyme. - causes food intoxication.

-refers to the use of single-celled organisms as a protein source.

Serological methods

1) are useful in identifying bacteria. 2) rely on the specificity of an antibody-antigen interaction. 3) may be simple and rapid.

Members of the Archaea typically thrive in conditions of excessive

1) heat 2) acidity 3)alkalinity 4)salinity

Members of the family Enterobacteriaceae

1) include E. coli, Enterobacter, Salmonella, and Shigella. 2) include many medically relevant bacteria. 3) primarily reside in the intestinal tract of humans and animals. 4) are facultative anaerobes.

The early symptoms of rabies generally begin: A. 1 day after viral entry. B. 1-2 months after viral entry. C. 6 months after viral entry. D. 1 year after viral entry.

1-2 months after viral entry Rabies is a classic zoonotic disease. Symptoms appear 1-2 months after infection and progress rapidly.

The portal of entry of Bacillus anthracis is the: A) respiratory tract. B) epidermis. C) gastrointestinal tract. D) epidermis OR gastrointestinal tract. E) respiratory tract, epidermis, OR gastrointestinal tract.

- respiratory tract, epidermis, OR gastrointestinal tract.

With which of the following substrates can Streptococcus mutans make a capsule? a. Sucrose b. Xylitol c. Mannitol d. Glucose e. All of the above

- sucrose

Rocky Mountain spotted fever is an example of a(n): A) animalosis. B) tickonosis. C) plantonosis. D) zoonosis. E) aviosis.

- zoonosis Zoonosis maintained in various species of ticks, mammals - Humans are accidental host

Gramnegative bacteria are less susceptible to complement lysis than Gram-positive bacteria.

-False

Identify the meninges from closest to the brain to furthest away: 1. dura mater 2. pia mater 3. arachnoid mater 2, 3, 1 3, 2, 1 3, 1, 2, 3 1, 3, 3, 1 1, 2, 3

2, 3, 1 pia is on the brain, then arachnoid, and dura is outmost.

The incubation period of gonorrhea is approximately: 2-5 days. 1-2 weeks. 14-21 days. several months.

2-5 days. Neisseria gonorrheae - gonococcus (GC) - a fastidious Gram-negative diplococcus that needs a rich medium like chocolate agar for cultivation - often asymptomatic in men and women. if signs and symptoms occur, they appear after 2-5 day incubation period.

Vocab quiz - don't even know where some of these came from.

20/20

The incubation period of leprosy is: A. 1 week. B. 3 weeks. C. 3 months. D. 3 or more years.

3 or more years p.721 - It is difficult to eradiacte, due to the long generation time of Mycobacterium leprae. This results in an incubation time of 3 years (range 3 months to 20 years) during which time the disease can remain undetected.

The incubation period of leprosy is 1 week. 3 or more years. 3 months. over 10 years. 3 weeks.

3 or more years.

Put the following steps of bacterial replication in the correct order, starting from a parent cell. 1.Cell elongation 2.Septum formation 3.Chromosome replication 4.Separation of daughter cells A. 3, 1, 2, 4 B. 1, 4, 3, 2 C. 2, 4, 3, 1 D. 4, 3, 1, 2 E. 3, 2, 1, 4

3, 1, 2, 4

Please select the correct sequence for the steps of phagocytosis: 1. Conditions in the phagosome change, increasing its antimicrobial activities. 2. The phagocytic cell binds microbial invaders and engulfs them, internalizing them in a phagosome. 3. Phagocytic cells are recruited to the site of an injury by chemoattractants. 4. Within the phagolysosome, various factors work together to destroy an engulfed invader. 5. The phagosome fuses with enzyme-filled lysosomes, forming a phagolysosome.

3, 2, 1, 5, 4

What percent of Staphylococcus aureus infections are due to the patient's own flora? 10-20% 30-100% 40-50% 0%

30-100%

The approximate BOD value for raw sewage is A. 2000-7000 milligrams per milliliter. B. 500-800 grams per milliliter. C. 0-50 kilograms per milliliter. D. 300-400 milligrams per liter

300-400 milligrams per liter p.787 - the BOD of raw sewage is approximately 300-400 mg/liter, whereas the dissolved oxygen content of natural waters is generally 5-10 mg/liter. So adding raw sewage to a lake could easily deplete the dissolved oxygen from the water.

Most human pathogens grow best at temperatures near 37ºC. 97ºF. 25ºC. 45ºC. 20ºC.

37ºC.

Pure water has a water activity value of ________. 0.90 1.00 0.70 0.80 100

1.00 Most fresh foods have a value above 0.98. Ham has a value of 0.91. Jam has a value of 0.85. some cakes have a value of 0.75

What gene sequences are used to identify the prokaryote organisms present in an environmental sample? 16S rRNA genes 18S rRNA genes 70S rRNA genes 80S rRNA genes All of these

16S rRNA genes Key word: parkaryotes. In eukaryotes it would be 18S rRNA genes.

The scientist responsible for the development of the first anti-plague vaccine in 1866 was Alexandre Yersin. Robert Koch. Louis Pasteur. Josef Marburg.

Alexandre Yersin p.664 - Yersin studies plague - reported discovery of a bacillus (rod) always present in swollen lymph nodes of plague victims. Could be cultivated, made rats sick and they made other rats sick. Yersin's plague bacillus was named Yersinia pestis, used to make a VACCINE and then an antiserum - led to first successful treatment of a plague victim.

You are reviewing for your nursing school entry exam with your study group. Each person in the group agrees to cover one disease. You select toxoplasmosis for your presentation. Toxoplasmosis can be a serious problem for those who are immunosuppressed, as well as for fetuses. Even in healthy people, the immune system may not clear the organism, resulting in a latent infection that can reactivate when immunity declines.Please identify the correct sequence of events in the pathogenesis of toxoplasmosis. 1. Each cyst contains large numbers of bradyzoites, a smaller form of T. gondii which persists in a latent infection. 2. As host immunity develops, a tough, fibrous capsule forms around infected host cells, forming a tissue cyst. 3. Within the small intestinal cells, sporozoites develop rapidly into tachyzoites which spread systemically. 4. T. gondii enters the host when either mature oocysts containing two sporocysts are eaten. 5. Sporozoites emerge from each of the sporocysts, and invade the cells of the small intestine. 4, 5, 3, 2, 1 1, 2, 4, 3, 5 1, 5, 4, 2, 3 2, 4, 5, 1, 3 3, 2, 1, 4, 5

4, 5, 3, 2, 1

Most bacteria are inhibited by a pH of 6.5. 7.0. 6.0. 7.2. 4.5.

4.5

The pH at which most fungi thrive is ___________.

5

You are looking after you sister's cat Singa. While playing one evening, Singa accidentally bites you instead of the toy mouse that you are holding. The bite wound is very small but is in the fleshy part...

5 of these...

Your patient had abdominal surgery three days ago. She seemed to be doing well after the surgery, but today the skin around the surgical incision is red and swollen. In addition, your patient has a h...

5 of these...

Which characteristic(s) is/are not common to c. tetani and c. perfringens...

?

Which signs and symptoms are not quickly resolved in a person with tetanus when they are given antibiotics?

?

how may lung damage or pneumonia occur in a person with tetanus?

?

Which of the following does not usually result from a wound infection?

? Infected wounds can have serious consequences • Delayed healing • Formation of abscesses • Spread of bacteria or toxins elsewhere in body

Which of the following statements about interferon is incorrect? A) It only works on a few specific types of virus. B) It makes cells resistant to viral infection. C) It is a species specific molecule. D) It does not directly inactivate viruses. E) IFNs regulate several immune responses.

A

Enlargement of lymph nodes or spleen is often associated with tularemia. brucellosis. plague. infectious mononucleosis. All of the choices are correct.

All of the choices are correct.

Entamoeba histolytica cysts survive passage through the stomach. causes amebiasis. may form cysts. may produce a cytotoxic enzyme. All of the choices are correct.

All of the choices are correct.

Fungi are important in

All of the choices are correct.

Fungi that are important for fermentation of fruits

All of the choices are correct.

In which of the following diseases does the spleen enlarge? infectious mononucleosis malaria leishmaniasis brucellosis All of the choices are correct.

All of the choices are correct.

The growth of microorganisms in a food product is influenced by the availability of acidity. moisture. temperature. nutrients. All of the choices are correct.

All of the choices are correct.

Treatment of HIV attempts to -block reverse transcriptase activity -block attachment to host cells -block viral integrate -block viral protease activity -All of the choices are correct.

All of the choices are correct.

Which is true of listeriosis? It is usually asymptomatic in healthy people. It may result in meningitis. It is a foodborne disease. It is caused by an organism that can grow at refrigerator temperatures. All of the choices are correct.

All of the choices are correct.

Which of the following groups contain diarrhea-causing E. coli? A. enterotoxigenic B. enteroinvasive C. enteropathogenic D. enterohemorrhagic E. All of the choices are correct.

All of the choices are correct.

Which of the following play some role in sewage treatment? A. activated sludge B. trickling filter C. septic tank D. lagooning E. All of the choices are correct

All of the choices are correct.

Traditionally the animal(s) associated with hosting Brucella is/are cattle. dogs. goats. pigs. All of the choices are correct.

All of the choices are correct. - domestic animals. Brucellosis melitensis if people but there are others: Brucellosis abortus infects cattle Brucellosis canis infects dogs Brucellosis melitensis infects goats Brucellosis suis infects pigs

Blood and lymph may carry antibodies. complement. lysozyme. interferon. All of the choices are correct.

All of the choices are correct. Antibodies = duh complement are blood proteins p.655 - Blood and lymph both carry infection-fighting leukocytes and antimicrobial proteins, including ANTIBODIES, COMPLEMENT, LYSOZYME AND INTERFERON. Correct

Campylobacter jejuni

Campylobacteraceae; campylobacteriosis; Gram-negative spirilum

Microscopic examination of a patient's fecal culture shows spiral bacteria. The bacteria probably belong to the genus A) Campylobacter. B) Escherichia. C) Salmonella. D) Shigella. E) Vibrio.

Campylobactor - spirals!

Candidiasis thrush yeast infections

Candida fungi

tetanus botulism gas gangrene colitis diarrhea

Clostridium gram + bacteria

The enzyme associated with the release of influenza virions from the infected cell is: A. Catalase B.KinaseC.Reverse transcriptaseD.NeuraminidaseE.Hyaluronidase

D

A group of interacting serum proteins that provide a nonspecific defense mechanism is..

Complement.

Which of the following produce lactic acid? Lactobacillus Leuconostoc Streptococcus Lactococcus All of the choices are correct.

All of the choices are correct. Know these so you will remember which do and will be better able to choice "Which of the following does NOT produce lactic acid?" if asked.

The antimicrobial aspect(s) of the skin is/are: A. dryness. B. saltiness. C. acidity. D. toxicity. E. All of the choices are correct.

All of the choices are correct. Too dry, salty, acidic, and toxic for most pathogens - Those that tolerate often shed with dead skin cells.

The secretions of the sweat and sebaceous glands provide ________ to the microbiota. A. water B. amino acids C. lipids D. All of the choices are correct.

All of the choices are correct. Use substances in sweat, sebum as nutrients; byproducts inhibit other microbes.

Which of the following statements regarding protozoa is FALSE?

All protozoa lack mitochondria

Once Y. pestis is in the human host, it is ingested by macrophages. Conditions within the macrophage activate certain genes in the bacterium, one of which is a gene for a capsule production. The capsule

Allows the bacteria to avoid phagocytosis

The complement pathway that is activated by binding of C3b to cell surfaces is the..

Alternate pathway

Select the TRUE statement regarding algae.

Although most algae have a cellulose/pectin cell wall, some algae lack a cell wall.

The cellular organelle responsible for the digestion of ingested infectious agents is the: A) endoplasmic reticulum. B) phagolysosome. C) Golgi apparatus. D) lysosome. E) peroxisome.

B. phagolysosome

Which level of biosafety (BSL) is appropriate for handling organisms that present the highest risk level for infection and the lowest success rate for prevention and treatment? A. BSL-1 B. BSL-4 C. BSL-3 D. BSL-2

BSL-4

anthrax food poisoning

Bacillus gram + bacteria

The key molecule upon which all complement pathways converge is..

C3

In S. pyogenes, which of the following interferes with phagocytosis? A) M protein B) Protein A C) Collagen D) Pilin E) Peptidoglycan

- M protein

The most frequent genus causing wound infections in healthy people is: - Pseudomonas. - Staphylococcus. - Pasteurella. - Rochalimea.

- Staphylococcus. Correct

The oily layer that lubricates the hair follicles of the skin is: A) suder. B) eczema. C) sebum. D) acnus. E) mucus.

- sebum Made by sebaceous glands open into hair follicles, moves up the hair to the outside, keeps hair and skin soft and water-repellent.

What is true of leukocidins? - they bind to Fc regions of antibodies - they kill neutrophils - they are superantigens - they make holes in host cell membranes

- they kill neutrophils - First is protein A and protein G - last one is alpha-toxin

Please select the FALSE statement regarding tooth decay.

...

To study the phylogeny of eukaryotes

18S rRNA is used

Mycobacterium leprae has a generation time of: A. 20 minutes. B. 1 hour .C. 6 hours. D. 12 days.

12 days

t is estimated that the mass of bacteria and fungi in the top six inches of an acre of soil is 500 lbs. 10 tons. 2 kilograms. 100 lbs. 2 tons.

2 tons

The PH at which most fungi prefer (or thrive at) is?

5.0

Iron..

All of the choices are correct

Oral Cavity Infections

Actinomyces gram + bacteria

Which of the following is/are microorganisms that can be used as food? A. Spirulina B. Single-cell protein C. Algae D. Fusarium graminearum E. All of the choices are correct

All of the choices are correct.

"Flu-like" cold

Adenoviridae DNA virus

Bordetella pertussis

Alcaligenaceae; pertussis (whooping cough); Gram-negative bacillus

Agar is obtained from

Algae

Aflatoxins

All of the choices are correct.

Which of the following is/are obligate intracellular parasites?

Chlamydia and Rickettsia

A particular characteristic of disease-causing Streptococcus is

BETA-HEMOLYSIS

The purple sulfur and green sulfur bacteria

Both use hydrogen sulfide as a source of electrons.

Clostridium botulinum

Botulism is caused by ________ ________

Which organism may be implicated in arteriosclerosis? Escherichia coli Pseudomonas aeruginosa Chlamydia pneumoniae Staphylococcus aureus

Chlamydia pneumoniae No idea where this comes from. Correct

Chlamydia trachomatis

Chlamydiaceae; sexually transmitted chlamydia; trachoma; Gram-negative pleiomorphic

The key molecule upon which all complement pathways converge is ________. A) C1 B) C2 C) C3 D) C6 E) C9

C3

The complement pathway that requires antibodies to be activated is the

Classical pathway

The causative agent of Lyme disease is: A. Ixodes scapularis B.Bordetella pertussisC.Brucella melitensisD.Borrelia burgdorferiE.Aedes aegypti

D

Cryptococcosis respiratory infection

Cryptococcus fungi

T/F Protozoans are usually multicellular and found in arid environments.

False

T/F Algae have a vascular system very similar to that found in plants.

False

Orthohepadnavirus (HBV)

virus, Hepatitis B

rotective features of the skin include all of the following except: A.Low pHB.High salt contentC.Keratinized surfaceD.Resident biotaE.Lysozyme

E

The conversion of ammonia to nitrate could be accomplished by the presence of both

Nitrosomonas and Nitrobacter.

Everything above I was asked

Everything below to help me study

nosocomial (hospital) infection urinary tract infections

Enterobacter gram - bacteria

Yersinia pestis

Enterobacteraceae; bubonic/black/septisemic plague; small Gram-negative bacilli

T/F Fungi are often capable of locomotion through the use of flagella.

False

Salmonella enterica

Enterobacteriaceae; Salmonellosis- systemic infection, enteric fever; Gram-negative bacilli

Surgical wound infection urinary tract infection

Enterococcus gram + bacteria

Following digestion of a microorganism by phagocytes, the debris is excreted by

Exocytosis

Rhizobium

FIX nitrogen inside nodules on the roots of legumes.

The conversion of organic to inorganic matter is called co-metabolism. T/F

False Co-metabolism occurs when an enzyme produced by a microbe to degrade an added substrate in an environment degrades a pollutant as well.

Genital herpes may be cured with the use of acyclovir or famciclovir. T/F

False p.750 - there is no cure for genital herpes. Anti-HSV medications such as acyclovir and famciclovir can decrease the severity of the first attack and the incidence of recurrences.

"Flesh-eating" Streptococcus pyogenes is considered a newly emerging disease. T/F

False slide 20 Cases in U.S. generally sporadic - Less than 2% of deaths from S. pyogenes infections are due to necrotizing fasciitis

T/F - Mast cells are only found in blood.

False - in tissues

T/F Algae may directly infect humans and cause disease.

False.

Pyrogens are

Fever-inducing substances.

What causes asthma?

Fungal spores.

Paralytic shellfish poisoning occurs when humans eat shellfish that have fed on

Gonyaulax species

What genus causes neurotoxins that is toxic to humans after eating shellfish?

Gonyaulax spp

Which of the following is more likely to cause fatal septicemias? Gram-positive bacteria Gram-negative bacteria negative stained bacteria acid-fast stained bacteria

Gram-negative bacteria Systemic infection by any microorganism can cause sepsis- more than 50% of cases are caused by Gram-positives but most fatal cases involve Gram-negatives because of the LPS (endotoxin) in their outer membrane.

Meningitis earaches

Haemophilus gram - bacteria

Hepatitis B

Hepadnaviridae DNA virus

Which of the following hepatitis viruses are transmitted by the fecal-oral route?

Hepatitis A and E

Human Herpesvirus 2 (HHV-2)

Human herpes simples virus 2 (genital herpes)

Helicobacter pylori

INHABIT THE STOMACH

Streptococcus pyogenes

IS A BETA-HEMOLYTIC

Which of the following would NOT trigger an inflammatory response? - Activation of complement proteins. - Binding of microbial products to PPRs. - Release of TNF by macrophages. - Tissue damage caused by microbial invasion. - Induction of antiviral protein synthesis by IFNs.

Induction of antiviral protein synthesis by IFNs.

What is agar made from?

It is obtained from algae.

Fever

Inhibits bacterial growth AND speeds up the body's reactions.

The low molecular weight protein produced by animal cells in response to viral infections is

Interferon

Describe the macroscopic algae special structure.

It acts as an anchor and is commonly called a holdfast.

Why is Staphylococcus epidermidis able to colonize plastic materials used in medical procedures? It can bind to fibronectin, a blood protein that coats plastic implants. It has fimbriae that allow it to attach to plastic surfaces. It produces coagulase that allows it to colonize inert surfaces such as titanium. It can produce a glycocalyx that allows it to form biofilms. These are all factors that allow S. epidermidis to colonize implants.

It can bind to fibronectin, a blood protein that coats plastic implants. Correct

Which of the following statements about Mycoplasma genitalium is not true?

It has a wide array of regulatory genes, many of which other bacteria lack. Has a tiny genome actually. p.744

Describe the sexual reproduction of Deuteromycetes.

It has not been observed. They have been further classified using rRNA analysis.

Choose the one FALSE statement about Pseudomonas aeruginosa. It is a Gram-positive, spore-forming encapsulated rod. Some strains can grow in nutrient-poor environments, including distilled water. It secretes pigments that together produce a green color. Under certain circumstances, it can grow anaerobically. It is widespread in nature, commonly found in plants.

It is a Gram-positive, spore-forming encapsulated rod.

The cells primarily involved in all immune responses are the

Leukocytes

Choose the one FALSE statement about Pseudomonas aeruginosa. It secretes pigments that together produce a green color. Under certain circumstances, it can grow anaerobically. Some strains can grow in nutrient-poor environments, including distilled water. It is widespread in nature, commonly found in plants. It is a Gram-positive, spore-forming encapsulated rod.

It is a Gram-positive, spore-forming encapsulated rod. p.607/608 - Pseudomonas aeruginosa is a motile, G- rod with a single polar flagellum. Found in a variety of environments such as soil and water, where it grows easily and fast. Is an aerobe but it also respires anaerobically i the absence of O2 if nitrate is present (important for it to form biofilms. Widespread in nature. Can grow in most places there is moisture: contact lens soln, hot tubs, even distilled water. Can be introduced into hospitals on flowers, potted plants and fruit baskets so these aren't allowed in ICU.

What is schizogony?

It is a means of multiple fissions and is performed by protozoa.

Which genus does not include skin-invading molds? A) Epidermophyton. B) Microsporum. C) Trichophyton. D) Ixodes. E) These are all skin molds.

Ixodes. - this is in the lyme disease section. The most important vector of Lyme disease in the eastern U.S. is: Ixodes scapularis.

The Euryarchaeota includes all

KNOWN PATHOGENS

The malolactic fermentation of wine is performed by Aspergillus species. Saccharomyces species. Leuconostoc species. Lactobacillus species. acetic acid bacteria.

Leuconostoc species.

Legionellosis (Legionnaire's Disease)

Legionella gram - bacteria

Borrelia burgdorferi

Lyme disease is caused by ______ _________

The leukocyte responsible for adaptive immunity is the

Lymphocyte

Abundant egg white component that degrades peptidoglycan

Lysozyme

The only known human pathogen that preferentially attacks the peripheral nerves is: A. N. meningitidis. B. polio virus. C. E. coli. D. M. leprae.

M. leprae p.720 - Mycobacterium leprae is the only known human pathogen that preferentially infects peripheral nerves.

Allergic reactions mainly involve

Mast cells

Complement..

May be activated through three pathways, disrupts the cytoplasmic membrane of invading bacteria and foreign cells AND is a group of blood proteins.

The childhood disease that damages the body defenses and is frequently complicated by secondary infections involving mostly Gram-positive cocci is: A) German measles. B) measles. C) mumps. D) chickenpox. E) shingles.

Measles Measles = rubeola German measles = rubella

Hookworm

Necator helminths (worms)

Which organism(s) is/are acid-fast?

Nocardia and Mycobacterium

Pulmonary infections

Norcardia gram + bacteria

Bdellovibrio

PREY ON OTHER BACTERIA

Pasteurellosis wound infections

Pasteurella gram - bacteria

Clostridioides difficile

Peptostreptococcaceae; pseudomembranous colitis, abx induced diarrhea; Gram-positive (spore forming) bacillus

People who contract M. genitalium infections are also at risk of acquiring other STIs. T/F

Probably true Seem to pick it up the same way and it causes the same signs/symptoms as other STIs...

The four cardinal signs of inflammation are:

Redness, heat, swelling, pain

Norovirus

virus, viral gastroenteritis

How could heavily fertilized lawns contribute to cyanobacterial blooms in lakes and oceans?

Run-off from the lawns will get into the water system, leading to large amounts of nitrogen that can be used by cyanobacteria in water systems.

How is Staphylococcus aureus distinguished from Staphylococcus epidermidis? It is spherical in shape. It is Gram-positive. S. aureus produces coagulase. It is an opportunistic pathogen. It does not produce coagulase.

S. aureus produces coagulase. Staphylococcus aureus is coag-positive Staphylococcus apedermidis is coag-negative just as all other Staphylococcus species other than S. aureus are. Correct

blood fluke

Schistosoma helminths (worms)

dysentary

Shigella gram - bacteria

Who determined that the cholera outbreak in 1850s London was due to contaminated water and approached the problem by removing the pump handle at the contaminated site? Pasteur Snow Koch Smith

Snow • John Snow (London physician) demonstrated CHOLERA transmitted by contaminated water in 1854 • During outbreak, observed victims got water from well on Broad Street; neighbors elsewhere unaffected • Even though germ theory not yet described, Snow convinced authorities to remove handle on pump

Yeasts

Single celled fungi

When cellular slime molds run out of food, they form a

Slug

Which of the following is not a matching pair? Soil—minimal biodiversity Rhizosphere—soil that adheres to plant root Streptomyces—geosmin production Fungi—lignin degradation Bacillus—endospores

Soil—minimal biodiversity Straight from the end of chapter multiple choice questions.

Clostridium tetani

Tetanus- lockjaw effects neuromuscular system reservoir: soil and animal intestinal tract anaerobic

Which is true of MRSA strains? A) They are all resistant to vancomycin. B) They carry the R plasmid AND they are all resistant to vancomycin. C) They may be susceptible to linezolid. D) The carry the R plasmid AND they may be susceptible to linezolid. E) The carry the F plasmid AND they may be susceptible to linezolid.

The carry the R plasmid AND they may be susceptible to linezolid. Pretty much just rules out the rest. - not all MRSA is resistant to vancomycin - Yes to R plasmid but no to B because no to A - No to E as F plasmid is fertility plasmid I want the R plasmid one so had to be D.

What enables the copied chromosomes to separate during binary fission? A. The chromosomes are attached to different parts of cell membrane, which elongates and thus separates the chromosomes. B. The septum splits the replicated chromosomes. C. The nuclear membrane dissolves. D. The mitotic spindle drives the separation.

The chromosomes are attached to different parts of cell membrane, which elongates and thus separates the chromosomes.

How may lung damage or pneumonia occur in a person with tetanus? A) The person may inhale regurgitated (vomited) stomach contents. B) Tetanospasmin causes the release of degradative enzymes in the lungs. C) Tetanospasmin is an A-B toxin that binds to lung epithelial cells, killing them. D) C. tetani releases alpha-toxin that causes the alveoli of the lungs to collapse. E) Spasms of major chest muscles may be so severe that the lungs are damaged.

The person may inhale regurgitated (vomited) stomach contents. Found in textbook.

As much as 95% of BOD can be removed during secondary treatment. T/F

True

Toxoplasmosis transmitted by cats

Toxoplasma Protozoa (apicomplexa)

Syphilis

Treponema gram - bacteria

The causative agent of syphilis is: papilloma virus. Neisseria gonorrheae. Chlamydia trachomatis. Treponema pallidum.

Treponema pallidum Herpes Chlamydia Gonorrhea Syphilis Treponema pallidum: very slender, highly motile (endoflagellum with corkscrew motion) spirochete that can't be seen using a gram stain. Must visualize by dark field microscopy, immunofluorescense or a special tissue staining called Silver Stain. Doesn't grow in vitro but does in rabbit testes. No enzymes for TCA cycle or ETC so can't generate much ATP. Gets what it needs from host.

Which one of the following is NOT transmitted by water? A) Salmonella B) Cyclospora C) Trichinella D) Cryptosporidium E) Hepatitis A virus

Trichinella

Whipworm

Trichuris helminths (worms)

Plague is caused by Yersinia pestis, a Gram-negative facultatively anaerobic rod. It is non-motile and grows best at 28°C. Because you are a microbiology student, you know that the organism is

a mesophile that grows best in the presence of oxygen but can also grow without it.

Purple sulfur bacteria and filamentous sulfur-oxidizers both

accumulate sulfur as intracellular granules.

The lactobacilli, in their role as normal flora of the vagina, help the vagina resist infection by contributing to

acidity of the vagina.

Sulfur occurs in all living matter primarily as a component of carbohydrates. fatty acids. amino acids. DNA and RNA. nucleotides.

amino acids

Which of the following soil organisms play a major role in decomposing plant matter? bacteria fungi protozoa viruses bacteria AND fungi

bacteria AND fungi

As an added safety precaution, low-acid, home canned foods should be - eaten with spoons. - boiled at least 15 minutes before consumption. - frozen for 1 hour before consumption. - dried prior to eating.

boiled at least 15 minutes before consumption.

The reservoir of T. pallidum is the: guinea. pig. fox. bat. human.

human

The organisms isolated from hydrothermal vents are typically bacteroid. phototrophs. rhizobia. chemolithoautotrophs. phototrophs, rhizobia, AND bacteroid.

chemolithoautotrophs.

The attraction of leukocytes to the area of inflammation is referred to as: A) parasitism. B) infection. C) phototaxis. D) chemotaxis. E) exotaxis.

chemotaxis

Cholera is the classic example of a(n) a. food-borne illness. b. zoonosis. c. opportunist. d. very severe form of diarrhea.

d. very severe form of diarrhea.

The group of Fungi in which sexual reproduction has not been observed is

deuteromycetes

The study of interactions between microbes and their environment and how those interactions affect the earth.

ecology

Storage temperature, atmosphere

extrinsic factor

Azotobacter

forms cysts.

The fluid which bathes and nourishes the tissue cells is the cytoplasm. interstitial fluid. lymph. blood.

interstitial fluid.

Sulfuric acid is

involved in bioleaching AND produced by unicellular sulfur-oxidizers.

Which is true of the poliomyelitis virus? A. non-enveloped B. single-stranded DNA C. double-stranded RNA D. single-stranded RNA E. non-enveloped AND single-stranded RNA

non-enveloped AND single-stranded RNA Poliomyelitis virus - naked, ssRNA virus is a member of the enterovirus subgroup of the family Picornaviridae. - poliomyelitis is caused by 3 serotypes of poliovirus (1,2,3) but type 1 is the only one still circulating today.

Phosphorus is a component of nucleic acids. phospholipids. nucleotides. carbohydrates. nucleic acids, phospholipids AND nucleotides.

nucleic acids, phospholipids AND nucleotides.

Treating foods with large amounts of salt draws the water out of foods by A. reverse osmosis B. efflux C. active transport D. osmosis E. lyophilization

osmosis The water follows the salt.

The "Black Death" may also be known as tularemia. brucellosis. endocarditis. plague.

plague.

two isolates with identical RFLPs are considered

possibly the same strain

Streptococcus pyogenes

rheumatic fever is caused by ___________ _______ heart issue, heart valves affected, heterophilic Ag response

Salmonella choleraesuis

salmonella zoonosis, pigs gram - negative rods chills, diarrhea, fever

The T. pallidum of syphilis can be transmitted by: sexual or oral contact. contact with contaminated objects. clothing. the fecal-oral route.

sexual or oral contact. Syphilis is usually transmitted by sexual intercourse. However, infections can occur from kissing a person with secondary syphilis, or by contact with a primary ulcer infected with Treponema pallidum.

In which organism were phagocytes first reported? - humans - Starfish larvae - Amoeba - Bacteria - Red blood cells

starfish larvae - Russian scientist Metchnikoff

Streptococcus pyogenes

strep throat is caused by __________ _______ throat, tonsils, mucus membranes, pus-filled nodules

All of the following are foodborne pathogens except A. Salmonella B. Staphylococcus aureus C. Clostridium D. Campylobacter E. Streptococcus

streptococcus Staphylococcus aureus and Clostridium cause foodborne intoxications. Salmonella and Campylobacter cause foodborne infections.

The polio viruses usually enter the body through: A. the oral route. B. contaminated blood. C. cuts in the skin. D. the respiratory route.

the oral route Poliovirus is transmitted by the fecal-oral route. It enters the body orally, infects cells that line the throat and intestinal tract, and then invades the bloodstream. Rarely does the virus enter the CNS (motor neurons).

Staphylococcus aureus

toxic shock syndrome is caused by __________ _______

All of the following infections can result from drinking contaminated water EXCEPT A) Cyclospora infection. B) giardiasis. C) trichinellosis. D) cholera. E) cryptosporidiosis

trichinellosis

Cytokine(s) released from macrophages in response to exposure to endotoxin is/are tumor necrosis factor. interleukin-1. protein A. alpha-toxin. tumor necrosis factor AND interleukin-1.

tumor necrosis factor AND interleukin-1.

Nucleic acid techniques have great power as diagnostic tools because they may be

very specific

Woese

was involved in determining rRNA sequences and proposed breaking prokaryotes into two groups.

What is eschar (a sign of cutaneous anthrax)? A) A small, red, fluid-filled vesicle B) An ulcer surrounded by swelling C) A large black scar D) Dead tissue resembling a flat scab E) Swelling with trapped gas

Dead tissue resembling a flat scab Yes, the pic in the book shows black tissue but don't be misled! An eschar (/ˈɛskɑːr/; Greek: eschara) is a slough or piece of DEAD TISSUE that is cast off from the surface of the skin, particularly after a burn injury, but also seen in gangrene, ulcer, fungal infections, necrotizing spider bite wounds, tick bites associated with spotted fevers, and exposure to cutaneous anthrax.

Organisms inhabiting soil or water which breakdown and absorb the organic matter of dead organisms.

Decomposer

One of the organisms that may cause red tide is

Dinoflagellates

Dog heartworm

Dirofilaria helminths (worms)

Which is a characteristic of the septicemic phase of leptospirosis?

Disease is often asymptomatic. 2 phases (biphasic) 1 - First phase is mild = Septicemic phase - characterized by PHOTOPHOBIA (sensitivity to light) and red eyes (dilation of small blood vessels). 2 - Second (Immune) phase - symptoms recur after 1-3 days of feeling well. Same symptoms plus bleeding from various sites, vomiting, rash and confusion. More severe.

Which of the following statements about gonorrhea is false? a) The incubation period is less than a week. b) Disseminated gonococcal infection (DGI) is almost invariably preceded by prominent urogenital symptoms. c) DGI can result in arthritis of the knee. d) Phase and antigenic variation helps the causative organism evade the immune response. e) Pelvic inflammatory disease (PID) is common in untreated women.

Disseminated gonococcal infection (DGI) is almost invariably preceded by prominent urogenital symptoms. p.742 - certain strains of N. gonorrhoeae can produce disseminated gonococcal infection (DGI). This systmeic infection is characterized by fever rash and arthritis caused by growth of the pathogen within the joint spaces. It can also lead to infective endocarditis and meningitis. DGI is not usually preceded by urogential symptoms.

What is the purpose of the mashing step in beer making? Mashing is required to remove residual solids, leaving wort that can be fermented by bacteria. During mashing, the enzymes of malted barley degrade starches, converting them to fermentable sugars. During mashing, the enzymes of malted barley degrade fermentable sugars, converting them to starches. Mashing increases the surface area available for microbes to adhere to during the fermentation process. All of the answer choices are correct.

During mashing, the enzymes of malted barley degrade starches, converting them to fermentable sugars.

A frequent cancer that is seen in AIDS patients is: A.MyelomaB.LeukemiaC.MelanomaD.Hodgkin's lymphomaE.Kaposi's sarcoma

E

The cause of infectious mononucleosis is varicella virus. Staphylococcus aureus. Epstein-Barr virus. Francisella tularensis.

Epstein-Barr virus. - enveloped, dsDNA virus of family Herpesviridae, no relation.

foodborne illness urinary tract infections

Escherichia gram - bacteria

The bacteria associated with hemolytic uremic syndrome is Lactobacillus. Campylobacter jejuni. Escherichia coli O157:H7. Salmonella cholerasuis. Pseudomonas aeruginosa.

Escherichia coli O157:H7.

Why would it be more difficult to treat diseases in humans caused by members of the Eukarya than diseases caused by the Bacteria?

Eukaryotic microbes use many of the same enzymes and systems as humans, so we lose the ability to target certain molecules that might be present ONLY in the cell type we want to eliminate.

Prions are small single-stranded pieces of RNA.

False - misfolded copy of normal protein

The water activity in foods with high levels of salt or sugar is high. T/F

False Seems like it should be but any water is tied up with the salt/sugar and unavailable for use by spoilage organisms.

EHEC is a major cause of diarrhea in developing countries and may result in the death of small children. T/F

False There is no EHEC. There is STEC, ETEC, EIEC, EPEC, EAEC, DAEC.

The saliva of asymptomatic carriers of herpes simplex is not infectious. T/F

False p.635 - even the saliva of asymptomatic people can be infectious, posting a risk to dentists and other healthcare workers.

Which of the following are most likely to be found growing on salted or dried foods? Fungi Viruses Bacteria Fungi AND bacteria Protozoa

Fungi

The return of carbon dioxide to the atmosphere and nitrogen to the soil is due to the action of

Fungi and bacteria

Which of the following viruses maintains a latent state in nerve cells? Rotavirus. Varicella zoster virus. Herpes simplex virus AND varicella zoster virus. Hepatitis A virus. Herpes simplex virus.

Herpes simplex virus AND varicella zoster virus.

The varicella-zoster virus is a member of which virus family? A) Paramyxoviridae B) Herpesviridae C) Togaviridae D) Papillomaviridae E) Retroviridae

Herpesviridae

Herpes cold sores chickenpox shingles mononucleosis

Herpesviridae DNA virus

Select the pair that is INCORRECTLY matched. High BOD—low organic content Potable water—lacks pathogenic microbes and harmful chemicals Total coliforms—lactose-fermenting members of the family Enterobacteriaceae Primary treatment—removal of material that settles High BOD—high organic content

High BOD - low organic content This one is almost identical to a multiple choice question at the end of the chapter.

Please choose the reason why gas gangrene infections can be treated with oxygen therapy successfully. A) oxygen reacts with water within the cell producing hydrogen peroxide, which kills the bacterial cells.The oxygen, delivered in low concentration, creates an anaerobic environment in which the Clostridium does poorly. B) Oxygen poisons the clostridial cells. C) The oxygen delivery produces a radioisotope of oxygen within the tissue, destroying the necrotic tissue.High pressure oxygen is delivered to the infected tissue, creating an unfavorable habitat for anaerobic Clostridium. D) High pressure oxygen is delivered to the infected tissue, creating an unfavorable habitat for anaerobic Clostridium.

High pressure oxygen is delivered to the infected tissue, creating an unfavorable habitat for anaerobic Clostridium. (on my chapter quiz - correct)

The first host response to a nonspecific tissue injury is described as

Inflammation

C3a and C5a are involved in

Inflammation AND enhancement of phagocytosis.

pneumonia

Klebsiella gram - bacteria

Smallpox

Poxviridae DNA virus

Which of the following practices would be most effective in breaking an infection cycle in a family infected with Giardia? Treat pets with animal medication for Giardia. Purchase a filtration system for household drinking water. Practice good hygiene by carefully washing hands, cleaning under the fingernails, and avoiding fecal contamination when changing diapers or using the restroom. Swim in lakes when no one else is present.

Practice good hygiene by carefully washing hands, cleaning under the fingernails, and avoiding fecal contamination when changing diapers or using the restroom.

acne

Propionibacterium gram + bacteria

Single-celled eukaryotic organisms that lack chlorophyll are called

Protozoa

The ability to exist as either a trophozoite or a cyst is characteristic of many

Protozoa

Single-celled eukaryotic organisms that lack chlorophyll include

Protozoa AND yeasts

Describe the importance of the food chain as it relates to protozoans.

Protozoans are an important part of the food chain ingesting large numbers of bacteria and algae.

Wound urinary tract infections

Pseudomonas gram - bacteria

Sarcodina move by means of

Pseudopodia

Which statement is FALSE? Trickling filter systems are used for secondary water treatment. Chlorine, ozone and UV light can be used to disinfect water. Artificial wetlands provide a habitat for wildlife. Methane is a by-product of anaerobic digestion. Removal of nitrates by microorganisms requires aerobic conditions.

Removal of nitrates by microorganisms requires aerobic conditions. p.789 - anaerobic respiration of nitrifying bacteria

Mushrooms

Reproductive structures of some fungi

Interferons function to make cells

Resistant to viral replication

Lentivirus (Human Immunodeficiency Virus—HIV) oncoviruses

Retroviridae RNA virus

Which of the following statements regarding sepsis and septic shock is TRUE? - Sepsis can be prevented with an attenuated vaccine. - Sepsis initially starts due to an overstimulation of the inflammatory response. - A patient surviving the initial stages of sepsis will always recover fully. - Sepsis and septic shock are always caused by Gram-negative bacteria.The signs and symptoms of sepsis include drop in urine output and dusky skin color. - Sepsis and septic shock are easily treated by administering combinations of antibiotics.

Sepsis initially starts due to an overstimulation of the inflammatory response. Problems with each of the other statements.

nosocomial (hospital) infection respiratory pathogen

Serratia gram - bacteria

Symptoms of infectious mononucleosis include: A. Vesicular lesions in oral mucosa B. Fever and pocks on skin C. Sore throat with pus coating, fever, cervical lymphadenopathy and splenomegaly D. Fever, severe diarrhea, pneumonitis, hepatitis and retinitis E. None of the choices is correct

Sore throat with pus coating, fever, cervical lymphadenopathy and splenomegaly

Borrelia burgdorferi

Lyme disease gram - negative skinny spirochete nymph (deer) tick transmits small mammal reservoir- zoonosis vector-borne illness virulence factor- OspC membrane red bullseye rash flu-like symptoms bad arthritis invades CNS, heart, liver, eyes, muscles, joints treatment: antibiotics prevention: behavioral, socks over pants, wear DEET, check for ticks when coming inside, mow your yard 30,000 cases per year

Which is NOT a reason that it would be more difficult to eradicate Lyme disease than rubeola or rubella? Rubeola and rubella only infect humans—if all humans were immune (through vaccination for example), the diseases would be eliminated. The causative agent of Lyme disease is harbored in intermediate hosts (mice and deer)—elimination of these would be impossible. Lyme disease is transmitted by Ixodes tick vectors—it would be impossible to eliminate these ticks from earth. Lyme disease is cannot be treated successfully with antimicrobials, while rubella and rubeola can be successfully treated—it is thus easier to eliminate them. These are all valid reasons to explain why eradicating Lyme disease would be more difficult than eliminating rubella and rubeola.

Lyme disease is cannot be treated successfully with antimicrobials, while rubella and rubeola can be successfully treated—it is thus easier to eliminate them. This is NOT a reason because Lyme disease can be treated with antimicrobials. CORRECT!

Many spirochetes are difficult to cultivate, so their classification is based on their

MORPHOLOGY AND ABILITY TO CAUSE DISEASE

Organisms that typically produce colonies with a fried egg appearance are the

MYCOPLASMA

Complex structures called fruiting bodies are a characteristic of

MYXOBACTERIA

What is the target of anthrax protective antigen (PA), edema factor, and lethal factor? A. Endothelial cells lining the capillaries B. Cells lining the alveoli in the lungs C. Neurons D. Macrophages

Macrophages

The terms yeast, mold, and mushrooms refers to fungal

Morphology

They terms yeast, mold, and mushrrom refers to fungal....?

Morphology.

List the habitat requirement of protozoa.

Must have moisture.

Intracellular Gram-negative diplococci found in a urethral sample of a male is indicative of

Neisseria gonorrhoeae

Neisseria gonorrhoeae

Neisseriaceae; gonorrhea; Gram-negative cocci

Neisseria meningitidis

Neisseriaceae; meningingicocal meningitis; fulminant disease; Gram-negative cocci

The first kind of leukocyte lured to the site of inflammation is the

Neutrophil

Which of the following is a phagocytic cell found in the human body?

Neutrophil

The virulence factor of Yersinia pestis that is a protease that destroys C3b and C5a is Yops. PLA. F1. protein A.

PLA. Table 25.3/p.670 - Pla (protease) activates plasminogen, destroys C3b, C5a and clots (allows bacteria to spread) - Once Y. pestis enters via a flea bite, it releases a protein-degrading protease (Pla) that clears the lymphatics and capillaries of clots and inactivates certain complement system components, allowing the organism to spread - goes to lymph nodes, gets taken up by macrophages.

Common warts genital warts

Papovaviridae DNA virus

lung fluke

Paragonimus helminths (worms)

Morbillivirus (measles)

Paramyxoviridae RNA virus

The leading cause of bacterial meningitis in adults is: A. Escherichia coli. B. Haemophilus influenzae. C. Streptococcus pneumoniae. D. Neisseria meningitidis.

Streptococcus pneumoniae p.698 - Pneumococcal meningitis is caused by Streptococcus pneumoniae, part of the normal microbiota in the nose and throat of many healthy individuals. Although best known as a cause of pneumonia, it is also the leading cause of meningitis in adults. Streptococcus pneumonia = pneumococcus. Gram-positive diplococcus, lancet shaped cells, polysaccharide capsule. common cause of otitis media, sinusisit and pneumonia. Penicillin. Some strains resistant.

Which of the following has been associated with the flesh-eating organism? H. lechter Pseudomonas aeruginosa Staphylococcus aureus Streptococcus pyogenes

Streptococcus pyogenes Slide 17/18 Group A Streptococcal "Flesh-Eating Disease" Nectrotizing Fasciitis • Streptococcus pyogenes commonly infects wounds • Generally easy to treat: susceptible to penicillin

Which bacteria ferment milk lactose, producing acids that curdle milk? A. Streptococcus thermophiles, Lactococcus, Lactobacillus B. Leuconostoc mesenteroides C. Saccharomyces cerevisiae D. Propionibacterium E. Spirulina

Streptococcus thermophiles, Lactococcus, Lactobacillus

Why might the Yersinia pestis from a patient with pneumonic plague be more dangerous than the same organism from fleas? The organism acquired this way is already fully virulent, so is especially dangerous. The organism mutates in a person with pneumonic plague, so is more dangerous. The organism involved in pneumonic plague is resistant to multiple antibiotics. Y. pestis in the lungs has a polysaccharide capsule while that in fleas does not. These are all true.

The organism acquired this way is already fully virulent, so is especially dangerous. p.670 Correct

Which of the following is specific to H. pylori strains associated with stomach cancer? They produce VacA. They have sheathed flagella. The are acidophilic. They are microaerophilic. They produce CagA.

They produce CagA. p.632 Correct

Which of the following applies to gonorrhea? The causative agent is a slow-growing, bottle-shaped, Gram-positive bacterium. The causative agent uses a capsule to attach to squamous epithelium and ciliated cells. The causative agent is a human-specific pathogen that survives poorly in the environment. Recovery from gonorrhea confers lifelong resistance to the causative agent. A trivalent vaccine protects people from the most virulent strains of N. gonorrhoeae.

The causative agent is a human-specific pathogen that survives poorly in the environment. p.742

Why is it difficult to determine the causative agents of periodontal disease? The agents are fastidious bacteria that cannot be cultured in the laboratory. The infections are polymicrobial, so multiple species of interacting bacteria are involved. The causative agents are encased in biofilm polysaccharides that cannot be removed. Identification requires 16S rDNA sequencing and not all bacteria have this gene. The causative agents are typically viruses, which are difficult to identify.

The infections are polymicrobial, so multiple species of interacting bacteria are involved.

Nector americanus

animal (hookworms), Necatoriasis

Enterobius vermicularis

animal (pinworms), Enterobiasis

Which is not used as a chemical preservative in food? A. Sulfite B. Salt C. Ethylene and propylene oxide gases D. Antibiotics E. Organic acids

antibiotics

During which stage of pregnancy can T. pallidum cross the placenta and possibly infect the fetus? first trimester. second trimester. third trimester. any stage.

any stage p.746 - In pregnant women, Treponema pallidum (syphilis) easily corsses the placenta and infects the fetus, causing congenital syphilis. Fetal infections can occur in the absence of any signs or symptoms of syphilis in the mother and at any stage of pregnancy, but damage to the fetus does not generally happen until the 4th month.

Food contaminated with Clostridium botulinum often: A. looks odd. B. smells bad. C. tastes bad. D. appears normal.

appears normal Foodborne botulism - eat toxin Intestinal botulism - ingest endospore - happens more in kids = infant botulism Wound botulism - endospore in wound

Endospores

are a dormant form of bacterium AND are form by members of medically relevant groups bacteria

Coliforms

are an informal grouping of enterics. ferment lactose. include E. coli. are used as indicators of fecal contamination. Correct All of the choices are correct.

Nitrifiers are chemolithotrophs. are reducers. are obligate anaerobes. are thermophiles. convert nitrate to nitrite.

are chemolithotrophs

Urinary tract infections:

are the most common nosocomial infections AND are not considered STIs

The leukocyte that contains histamine is the lymphocyte. monocyte. macrophage. basophil.

basophil - granules that stain purplish-blue with methylene blue. - involved in allergic rxns, like mast cells. - granulocyte - 2nd most numberous WBC

Streptococcus pyogenes would be

beta hemolytic and catalase negative

A particular characteristic of disease-causing Streptococcus is

beta-hemolysis

The breakdown of man-made compounds by decomposers is called A. Parasitism B. Mineralization C. Saprobism D. Bioremediation E. Decomposition

bioremediation Decomposition is the process by which organic substances are broken down into simpler organic matter. - it's the man-made part that means BIOREMEDIATION.

The region of the earth inhabited by living organisms is called the ecosystem. niche. community. biosphere. environment.

biosphere "The biosphere is that part of the earth inhabited by living organisms, including land, ocean and the atmosphere in which life can exist. It is the global ecosystem."

HIV may be present in tears, sweat, AND colostrum. vaginal secretions, colostrum, AND sweat. blood, semen, AND tears. blood, semen, AND vaginal secretions. semen, vaginal secretions, AND tears.

blood, semen, AND vaginal secretions.

Protozoans are an important part of the food chain ingesting large numbers of

bacteria and algae.

Citrus fruits generally escape bacterial contamination because A. citrus fruits are high in salt B. citrus fruits are warm-temperature fruits C. bacteria cannot form spores within fruits D. bacteria do not favor acidic conditions E. U.V. radiation in sunlight kills off bacteria on their surface

bacteria do not favor acidic conditions

The most common urinary infection is vulvovaginal candidiasis. toxic shock syndrome. bacterial cystitis. bacterial vaginosis.

bacterial cystitis Also called bladder infection. p.730 - Cystitis is the most common type of UTI. Is common among otherwise healthy women and is a frequent HAI.

The point at which two organisms diverged form a common ancestor

can be determined by comparing the nucleic acid sequences.

HPV: can integrate into the host's chromosome. may be split into cancer-associated and wart-causing types. infections always lead to cancer. cannot infect newborns at birth. can integrate into the host's chromosome AND may be split into cancer-associated and wart-causing types.

can integrate into the host's chromosome AND may be split into cancer-associated and wart-causing types. p.752 - HPV - genome of high-risk (cancer associated) HPV can integrate into the chromosome of the host cell. - Gardasil 9 vaccine - protects against HPV types 16/18 - cancers and HPV types 6/11 - genital warts.

Lyme disease is transmitted by?

Ticks.

Rubella is a member of which virus family? A) Paramyxoviridae B) Herpesviridae C) Togaviridae D) Papovaviridae E) Retroviridae

Togaviridae Virus families end in -viridae. Rubella virus of togavirus family: - enveloped, single-stranded RNA • Easily cultivated in cell cultures

Lactic acid, an end product of fructose fermentation, causes breakdown of tooth enamel and, eventually, cavities.

True

T/F - White blood cells also called leukocytes are important in immunity.

True

T/F -pH determines production of toxins (i.e. they are not produced below a pH of 4.5)

True

Tetanus antitoxin can cross the placenta. T/F

True

The PPSV23 vaccine may protect against pneumococcal pneumonia, pneumococcal meningitis, and otitis media. True False

True

The bacteria primarily responsible for dental caries is Streptococcus mutans. T/F

True

The initial symptoms of bacterial meningitis are the same regardless of the causative agent. T/F

True

The media used to successfully grow Francisella tularensis must contain cysteine. T/F

True

The mumps virus initially infects the respiratory tract. T/F

True

The organisms found near warm vents on the bottom of the ocean are typically chemoautotrophic. T/F

True

The term "potable water" refers to water that is not necessarily pure, but is safe to drink. T/F

True

Zero coliforms per 100 ml of water is considered safe for treated potable water. T/F

True - I put false, wrong! So definitely true.

Lactobacillus acidophilus can potentially colonize the intestinal tract. T/F

True - in the discussion of Acidophilus milk on slide 30

Non-toxin-producing strains of Corynebacterium diphtheriae could infect the body but would not cause the typical symptoms of diphtheria. T/F

True Ch 21 - it's the ones that are virus infected lysogens that make the toxin that is causes diphtheria.

Both Giardia lamblia and Cryptosporidium parvum are resistant to chlorine. T/F

True Correct

The skin-invading molds are collectively called dermatophytes. T/F

True Dermatophytes are group of skin-invading molds including members of genera Epidermophyton, Microsporum, and Trichophyton Dermatophytes can invade hair, nails, keratin in skin • Result in jock itch, athlete's foot, ringworm, or Latinized names describing location: tinea capitis (scalp), tinea barbae (beard), tinea axillaris (armpit), tin

Both Staphylococcus aureus and Streptococcus pyonese have fibronectin binding proteins. T/F

True It is the protein F in Streptococcus pyogenes.

Meningoencephalitis and African sleeping sickness are both caused by protozoans. T/F

True Naegleria fowleri - free-living protozoa that is pathogenic for humans. Ameboid trophozoite gives rise to flagellated forms and spherical cysts. Trypanosoma brucei -

Women are more likely than men to get urinary tract infections due to anatomy. T/F

True Short urethra (4 cm) plus opening very close to anus means women get more UTIs.

When a person becomes infected with the Epstein-Barr virus (EBV), a member of the herpesvirus family, they are infected for life. They may occasionally suffer reactivation events that may make them infectious to other individuals once again. T/F

True p.679 - EBV is present in saliva for up to 18 months after infectious mononucleosis and then intermittently for LIFE.

Death from botulism is usually due to respiratory paralysis. T/F

True p.721 - progressive paralysis generally involves all voluntary muscles, respiratory paralysis is the most common cause of death.

Most strains of gonococcus do not survive long outside the body. T/F

True p.742 - Neisseria gonorrheae (gonococcus, GC) is well adapted to grow within the human host. Like other STI agents, it is a human-specific antigen that survives poorly in the environment. p.743 - N. gonorrhoeae infects only humans, living mainly on the mucus membranes...most strains are susceptible to UV light, cold, desiccation and do not survive well outside the host.

There is no long-lasting immunity to gonorrhea and one may contract gonorrhea repeatedly. T/F

True p.743 - there is little or no immunity following recovery from gonorrhea - a person can contract gonorrhea repeatedly.

Humans are the only reservoir for varicella-zoster virus. T/F

True • Humans are only reservoir

Diseases caused by fungi are called mycoses. T/F

True • Mycoses = diseases caused by fungi

Varicella is a member of the herpes family of viruses and produces a latent infection. T/F

True • Varicella-zoster virus (VZV) of herpesvirus family: enveloped, double-stranded DNA virus

CMV is a member of the herpes family of viruses and produces a latent infection. T or F

True, HHV5

Protozoa

Unicellular heterotrophs that are not fungi

Documented transmission of HIV involves: A. Mosquitoes B. Unprotected sexual intercourse and contact with blood/blood products C. Respiratory droplets D. Contaminated food E. All of the choices are correct

Unprotected sexual intercourse and contact with blood/blood products

The most prevalent cause of wine spoilage is the presence of sulfur dioxide. ethanol. carbon dioxide. lactic acid bacteria. acetic acid bacteria.

acetic acid bacteria. Add sulfur dioxide (SO2) to inhibit growth of other microbes on the grapes - including these else your wine could become expensive vinegar.

Shigella moves from cell to cell using actin tails. pili. cilia. flagella AND pili. flagella.

actin tails p.640 Correct

Which of the following has shown some effectiveness in treating a herpes infection? AZT protease inhibitors acyclovir cephalosporin

acyclovir Acyclovir, penciclovir target HSV DNA polymerase - Do not affect latent virus and so cannot cure.

The practice of cells turning on the production of some enzymes and turning off others in response to a changing environment is called adaptation. metabolism. nutritional metabolism. compromise. quorum sensing.

adaptation

Why would soil and water be added to a compost pile?

adds microbes and moisture to degrade organics

Bioaugmentation adds specific microorganisms to a contaminated site. aims at enhancing the growth of the resident population of microbes. is typically done offsite. usually utilizes genetically engineered bacteria. depends on competition between resident and added microbes.

adds specific microorganisms to a contaminated site.

In sewage treatment, the removal of phosphates and nitrogen compounds is achieved during advanced treatment. quaternary treatment. secondary treatment. primary treatment. All of the choices are correct.

advanced treatment

Which of the following does not usually result from a wound infection? Bacterial spread. Aerobic conditions. Delayed healing. Abscess formation. These all result from delayed wound healing.

aerobic conditions

Most fungi are

aerobic or facultatively anaerobic.

The nitrogen in our atmosphere is only usable to us after it is converted to ammonia. amino acids. fertilizer. nitrous oxide.

amino acids

The biochemical oxygen demand (BOD) is a measure of the A. B-cell receptors of dendritic cells B. degree of chemical pollution in water C. number of hepatitis viruses in underground springs D. effectiveness of fluoroidation in the preparation of drinking water E. amount of biological pollution in water

amount of biological pollution in water

C. botulinum toxin is - heat-stable. - an exotoxin. - endotoxin. - heat-sensitive. - an exotoxin AND heat-sensitive.

an exotoxin AND heat-sensitive.

During aerobic decomposition of organic matter the primary gas produced is oxygen. hydrogen sulfide. nitrogen . carbon dioxide.

carbon dioxide

The carbon cycle revolves around carbon monoxide carbon dioxide. water. phosphorus. nitrogen.

carbon dioxide

The most common cause of chronic lymph node enlargement at a localized body site in young children is: A. rat bite fever. B. dead bat fever. C. cat scratch fever. D. mouse itch fever.

cat scratch fever. Bartonellosis (Cat Scratch Disease) slide 46 -In the US, cat scratch disease is the most common cause of chronic, localized lymph node enlargement in young children

Fungi are important because of their ability to

cause disease in plants, make certain foods and beverages, spoil food, help many plants grow.

To determine the causative agent of a central nervous system (CNS) infection, a sample is taken from the: A. cerebrospinal fluid. B. arterial blood. C. venous blood. D. urine.

cerebrospinal fluid A needle is inserted at the lumbar vertebrae where the spinal cord narrows into a tail, into the subarachnoid space to get a sample of the CSF = Spinal Tap/lumbar puncture

Water treatment processes for drinking water A. are similar to wastewater treatment. B. are only necessary when using recycled water. C. includes disinfection but not litration. D. has no biological treatment phase

has no biological treatment phase - for wastewater, secondary treatment is a biological process that converts most of the suspended solids to inorganic compounds and cell mass that can be removed. Microbial growth is actively encouraged, allowing aerobic organisms to oxidize the biologically degradable organic material to carbon dioxide and water. p.787 - This doesn't happen for drinking water.

Pediculus humanus

only uses humans as a host AND can transmit a bacterial disease.

The treatment of neonates with an erythromycin ointment placed directly into the eyes is to prevent: viral conjunctivitis. cataracts. herpes. ophthalmia neonatorum.

ophthalmia neonatorum Also called neonatal conjunctivitis - it is by law in the US that newborn babies have erythromycin put in their eyes within an hour of birth. The disease if often asymptomatic and could cause blindness in the babe so prophylaxic treatment. p.743

Vaccines can be used to protect at-risk individuals from all of the following liver diseases EXCEPT __________. hepatitis A hepatitis C hepatitis B hepatitis D

hepatitis C there is no vaccine available for preventing HepC.

Which of the following viruses maintains a latent state in nerve cells? herpes virus. hepatitis A virus. Norwalk virus. rotavirus.

herpes virus p.750 - most of the time, the viral DNA exists within nerve cells in a circular, non-infectious form, causing no symptoms.

Disagreements between conclusions obtained from rDNA data and other techniques may be explained by

horizontal DNA transfer.

The organic matter in soil is referred to as inorganic substance. humus. mineral soil. bedrock. mulch.

humus

Painful spasms of the throat triggered by swallowing or the sight of water is called A. hydrophobia. B. hydrophagia. C. tetany. D. paralysis.

hydrophobia Fear of water. Rabies = zoonosis - rabies virus - bullet shaped, has envelope, ss negative sense RNA virus - family Rhabdoviridae

The conical sea floor vents that spout sulfide-rich, super-heated water at temperatures up to 300ºC are called hot vents. yellow smokers. warm vents. hyperthermophiles. hydrothermal vents.

hydrothermal vents

Most North American outbreaks of Cyclospora cayetanensis have been associated with: A. cattle. B. iguanas. C. imported leafy vegetables and berries, especially raspberries. D. chickens.

imported leafy vegetables and berries, especially raspberries.

A wound created by the drag of a knife across skin can be classified as a(n) abrasion. laceration. incision. puncture. contusion.

incision Correct

Results from ingestion of whole microbial cells that target the intestine.

infection - think Salmonella and Campylobactor

Which of the following may be transmitted by saliva? infectious mononucleosis rabies plague yellow fever infectious mononucleosis AND rabies

infectious mononucleosis AND rabies Rabies was covered in a different chapter but it is carried by animal in their saliva and we worry about rabies with animal bites. Mono - Kissing disease, saliva

The production of heterophile antibody is associated with tularemia. brucellosis. plague. infectious mononucleosis.

infectious mononucleosis. Most B cells get non-productive (latent) infection. EBV activates the B cells, they multiply and make useless antibodies - HETEROPHILE ANTIBODY that can be used to test = Monospot test - causes agglutination when mixed with animal RBCs.

The condition that develops on a previously damaged heart valve is called acute bacterial endocarditis. myocarditis. an aneurysm. infective endocarditis. effective endocarditis.

infective endocarditis Correct

Helicobacter pylori

inhabit the stomach

Among the major causes of vulvovaginal candidiasis is/are: sexual promiscuity. intense antibacterial treatment. disruption of normal flora.the use of oral contraceptives. intense antibacterial treatment, disruption of normal flora AND the use of oral contraceptives.

intense antibacterial treatment, disruption of normal flora AND the use of oral contraceptives. Caused by Candida albicans yeast. - Like BV, it seems to occur aftera disruption of the normal microbiota - factors that predisposition = use of oral contraceptives or antibiotics. p736

The presence of a hard chancre in primary syphilis is representative of the body's: intense inflammatory response. antibody response. rejection of the invaded tissue. septicemia due to infiltration of the organism.

intense inflammatory response Chancres usually develop on the genitalia but they may occur anywhere in the body. Painless so they can go unnoticed. Lymph nodes near the chancre may swell. p.747 - the hard chancre is caused by an intense inflammatory response to the bacteria, which are present in high numbers in the lesion.

Results from ingestion of exotoxin secreted by bacterial cells growing in the food.

intoxication Think Staphylococcus aureus and Clostridium botulinum

The conditions normally present in food such as moisture, acidity, and nutrients are referred to as: extrinsic factors. intrinsic factors. endogenous factors. exogenous factors

intrinsic factors Things about the food itself not about it's environment.

Irradiation of foods - involves the use of gamma radiation. - is used on all foodstuffs. - changes the taste of foods. - is not regulated by the government. - involves the use of gamma radiation, changes the taste of foods AND is not regulated by the government.

involves the use of gamma radiation. Gamma radiation is used to kill microbes without changing the taste of the food.

A microbial mat is a form of biofilm. is poorly organized. is constructed to prevent interactions between the various layers of microorganisms. is a form of biofilm AND is poorly organized. contains only obligate anaerobes.

is a form of biofilms

6. The nervous system typically: A. is sterile. B. has a small number of normal flora. C. contains Gram-positive bacteria. D. contains a small number of viruses

is sterile The nervous system lies entirely within body tissues and has no normal microbiota. CSF is generally sterile so the presence of microbes indicates an infection.

The activated sludge process is used during secondary treatment of sewage. is meant to increase the BOD. is meant to convert inorganic to organic matter AND is meant to increase the BOD. is meant to convert inorganic to organic matter. removes large objects from the sewage.

is used during secondary treatment of sewage.

Phage typing

is used to distinguish bacterial strains

E. coli O157:H7 characteristics include all the following except: A. it only causes occupational illness in people who work with animals B. it is transmitted by ingestion of contaminated, undercooked food, especially hamburger C. it has a reservoir of cattle intestines D. it causes a bloody diarrhea E. some cases go on to hemolytic uremic syndrome (HUS) with possible kidney failure

it only causes occupational illness in people who work with animals

The complement pathway that is activated by mannan-binding protein is the: A) classical pathway. B) alternative pathway. C) C3 pathway. D) lectin pathway. E) properdin pathway.

lectin pathway The lectin pathway is responsible for an antibody-independent pathway of complement activation that is initiated by binding of the mannose-binding lectin (MBL) to carbohydrates on the surface of pathogens.

Cryptococcal meningoencephalitis caused by Cryptococcus gattii differs from the general pattern of fungal central nervous system (CNS) disease by occurring in: A. otherwise healthy people. B. children. C. AIDS patients. D. immunosuppressed people.

otherwise healthy people Cryptococcal meningoencephalitis - Cryptococcus neoformans (opportunistic) - Cryptococcus gattii (causes disease even in healthy individuals) These are spherical yeasts generally 3 to 20 micrometers in diameter surrounded by a thick capsule (capsule stain with Inida Ink) that resists the immune response.

White blood cells are called:

leukocytes

HIV adversely affects: red blood cells. nerve cells. macrophages. T helper cells. macrophages AND T helper cells.

macrophages AND T helper cells.

When Gram-negative bacteria are multiplying in the bloodstream, sepsis/septic shock can result. This is due to binding of endotoxin to toll-like receptors (TLRs) on _________ and neutrophils, provoking an uncontrolled release of pro-inflammatory _________ and activating the complement cascade. These events result in a widespread, self-stimulating inflammatory response. macrophages; cells macrophages; cytokines basophils; cells lymphocytes; cytokines

macrophages; cytokines

Helicobacter can grow in the stomach because it: A) Hides in macrophages. B) Makes a capsule. C) Makes NH3. D) Makes HCl. E) Invades epithelial cells.

makes NH3 - ammonia

Allergic reactions mainly involve: A) macrophages. B) monocytes. C) mast cells. D) neutrophils. E) eosinophils.

mast cells - basophils too, had that been a choice.

Various strains of E. coli

may cause disease and may be harmless

Dimorphic fungi

may grow as mycelia or yeast AND are often associated with disease in humans.

Streptomyces may produce antibiotics. may produce geosmins. will swarm together to form fruiting bodies. are typically found in the soil. may produce antibiotics, may produce geosmins AND are typically found in the soil.

may produce antibiotics, may produce geosmins AND are typically found in the soil.

The complex resulting from complement activity that leads to cell lysis is the: A) prostaglandin complex. B) leukotriene activating complex. C) membrane attack complex. D) histamine complex. E) macrophage antibody complex.

membrane attack complex MAC is composed of a complex of four complement proteins (C5b, C6, C7, and C8) that bind to the outer surface of the plasma membrane, and many copies of a fifth protein (C9) that hook up to one another, forming a ring in the membrane.

Protists

microscopic eukaryotes that are not fungi, plants, animals

The most necessary habitat requirement of protozoa is

moisture

The terms yeast, mold, and mushrooms refers to fungal

morphology

The poliomyelitis virus appears to selectively destroy autonomic nerve cells. mixed nerve cells. muscle cells. motor nerve cells. sensory nerve cells.

motor nerve cells

The poliomyelitis virus appears to selectively destroy: A. sensory nerve cells. B. mixed nerve cells. C. autonomic nerve cells. D. motor nerve cells.

motor nerve cells Characteristic feature of polio is the destruction of motor neurons, resulting in paralysis of a group of muscles, like those of an arm or a leg.

Schizogony means

multiple fissions AND is performed by protozoa.

The viral disease that characteristically infects the parotid glands is: measles. herpes. chickenpox. mumps.

mumps Onset marked by fever, loss of appetite, headache • Followed by painful swelling of one or both parotid glands • Spasm of underlying muscle makes talking, chewing hard • Symptoms can arise elsewhere in body

The resulting solids and juices of grapes used to make wine are termed its skin. malt. germinater. mash. must.

must

Dimorphic fungi may grow as

mycelia or yeast AND are often associated with disease in humans.

A tangle of fungal hyphae is generally known as a

mycelium

Fungal diseases are generally referred to as

mycoses

Complex structures called fruiting bodies are a characteristic of

myxobacteria

. In rabies, the virus multiplies in one kind of cell then binds to receptors in the: A. neuromuscular region. B. spinal cord. C. respiratory area. D. brain.

neuromuscular region (does take out motor nerve endings so...) Rabies virus is introduced into the body, multiplies at the site of entry, enters a sensory neuron, travels by retrograde transport up the axon to the spinal cord and eventually to the brain. Once in brain tissue the virus multiples and causes the symptoms of encephalitis. - look for inclusion bodies called Negri bodies made up of viral nucleocapsids - can diagnose rabies from stained smears from surface of eye

In rabies, the virus multiplies in one kind of cell then binds to receptors in the spinal cord. neuromuscular region. brain. respiratory area. cerebrospinal fluid.

neuromuscular region.

The toxin of botulism is classified as a(n) - enterotoxin - endotoxiin - neurotoxin - toxoid

neurotoxin

The first kind of leukocyte lured to the site of inflammation is the: A) neutrophil. B) monocyte. C) macrophage. D) basophil. E) B cell.

neutrophil

Next to carbon and oxygen, the element organisms most require is sulfur. iron. potassium. nitrogen. phosphorus.

nitrogen

The sulfur cycle is similar to the cycle of water. carbon. oxygen. nitrogen. phosphorus.

nitrogen

The organisms that obtain energy by oxidation of hydrogen sulfide and use carbon dioxide as a carbon source are called chemoliths. heterotrophs. autotrophs. chemolithoautotrophs. chemoheterotrophs.

chemolithoautotrophs. Inorganic sources of energy and carbon.

Haemophilus influenzae

conjunctivitis (pink eye) is caused by _________ ________

Viral gastroenteritis that affects people of all ages and usually lasts less than 3 days is caused by influenza virus. herpes. norovirus. hepatitis B. rotavirus.

norovirus p.649 - Noroviruses are the most common cause of viral gastroenteritis in the US. They are a Category B bioterrorism agent because they spread easily with the potential of causing large, demoralizing outbreaks. Noroviruses are naked, ssRNA viruses from the family Caliciviridae.

In order to culture the organism responsible for tularemia, the growth media must contain charcoal. glucose. cysteine. NAD.

cysteine Francisella tularensis -bacteria- non-motile, aerobic Gram negative rod that needs added cysteine to grow in vitro.

Acute endocarditis is most commonly contracted through: A.FomitesB.Casual contactC.Parenteral entryD.IngestionE.Droplets

d wrong, wonder if it's C

The natural host(s) for Borrelia burgdorferi is/are _________. A. mosquitoes B. birds C. ticks D. deer and field mice

deer and field mice - Ticks of the genus Ixodes transmit B. burgdorferi between hosts and are the only natural agents through which humans have been shown to become infected

The transmission of leprosy is by: A. direct human-to-human contact. B. the fecal-oral route. C. contact with infected blood products. D. contact with infected nasal secretions. E. direct human-to-human contact AND contact with infected nasal secretions.

direct human-to-human contact AND contact with infected nasal secretions. Most of the human population is not susceptible to the disease. WHO provides free multidrug tx.

The oocytes of Cyclospora cayetanensis: are mature when eliminated in the stool. do not contain sporozoites when passed in the feces. are smaller than the oocytes of Cryptosporidium parvum. give rise to three sporozoites. All of the choices are correct. The oocytes of Cyclospora cayetanensis are mature when eliminated in the stool. contain sporozoites when passed in the feces. give rise to three sporozoites. do not contain sporozoites when passed in the feces. are smaller than the oocytes of Cryptosporidium parvum.

do not contain sporozoites when passed in the feces Slightly different questions - same answer

Tapeworms

do not have a digestive system AND may be transmitted by eating undercooked meat.

Vibrio cholerae is most often associated with touching animals. breathing air. eating food. drinking water. having unprotected sex.

drinking water.

All of the following pertain to Clostridium difficile infection except that it is: A. associated with disruption of normal flora due to broad spectrum antimicrobials B. the major cause of diarrhea in hospitals C. often from an endogenous source D. due to ingestion of contaminated, improperly stored, cooked meats and gravies E. a colitis that is a superinfection

due to ingestion of contaminated, improperly stored, cooked meats and gravies

The term used to describe the interrelationship between the living and non-living environments in an area is called a(n) group. community .ecosystem. ecological niche.

ecosystem population - organisms of the same type in the same location community - all of the different organisms in the location ecosystem - consists of a community of organisms and the non-living environment in which they interact. ecological niche - the role an organism plays in a particular ecosystem (where it belongs)

The food products most commonly contaminated with Salmonella strains are: milk and cheese. fruit and vegetables. meat and seafood. eggs and poultry. eggs and fruit.

eggs and poultry. p.644 Correct

The infectious form of Chlamydia trachomatis is a(n): reticulate body. elementary body. primary element. core body.

elementary body p.740 Figure 27.7 Elementary body is infectious and is outside host. Elementary body attaches to target cell with pili and injects (G-) effector protein to get engulfed, endosome turns into INCLUSION where elementary body forms reticulate body that multiples by binary fission before all the progeny reorganize into elementary bodies and the cell bursts or they get extruded in vesicles back to the outside.

Where is the majority of earth's water found? A. Soil moisture B. Freshwater lakes C. Icecaps and glaciers D. Oceans E. Ground water

oceans The ocean holds about 97 percent of the Earth's water. Next comes glaciers, rivers, lakes...

Amebiasis: A. is caused by C. parvum. B. often causes a bloody diarrhea. C. is an infection of the stomach. D. is restricted to temperate climates. E. is caused by C. parvum AND often causes a bloody diarrhea.

often cause a bloody diarrhea p.659 - Trophozoites of Entamoeba histolytica feed on mucus and tissue of the large intestine. They may also invade the intestinal lining, causing ulcerations and BLOODY DIARRHEA = amebic dysentery.

Algae

often grow in areas where other forms of life may have difficulty.

Nutrient poor waters are described as being autotrophic. oligotrophic. eutrophic. hypoxic. anoxic.

oligotrophic

The poliomyelitis virus is in the picornavirus family in the subgroup: A. adenoviruses. B. enteroviruses. C. arboviruses. D. dermotropic.

enteroviruses

The poliomyelitis virus is in the picornavirus family in the subgroup arboviruses. enteroviruses. dermotrophs. adenoviruses. retroviruses.

enteroviruses.

The human immunodeficiency virus is a(n): A. non-enveloped double-stranded DNA virus. B. enveloped double-stranded DNA virus. C. enveloped single-stranded RNA virus. D. non-enveloped single-stranded DNA virus.

enveloped single-stranded RNA virus p.755 - HIV is a retrovirus, common term for members of the Retroviridae family. HIV is an enveloped ssRNA virus. Most AIDS cases are caused by HIV-1.

The unique characteristic of Lyme disease is erythema migrans. induration. rash on palms. carbuncle. furuncle.

erythema migrans Bull's eye rash caused by the spreading of LPS and causing an inflammatory response. Borrelia burgdorferi - bacteria- Gram negative, microaerophilic spirochete with many plamsids and a chromosome that is linear and present in multiple copies

A region where the river meets the sea.

estuary

The products of yeast fermentation in breads are A. Carbon dioxide and water B. Ethanol and water C. Ethanol and carbon dioxide D. Lactic acid and carbon dioxide E. Lactic acid and water

ethanol and carbon dioxide Carbon dioxide helps bread rise, the ethanol bakes out.

The S. aureus product that causes scalded skin syndrome is: A) lipases. B) exfoliation toxin. C) leukocidins. D) protein M. E) peeling toxin.

exfoliation toxin. "Exfoliatin is a Staphylococcus aureus exotoxin[1] that causes a blistering of the skin known as staphylococcal scalded skin syndrome, usually in infants."

Following digestion of a microorganism by phagocytes, the debris is excreted by: A) ingestion. B) exocytosis. C) extrusion. D) budding. E) lysis.

exocytosis

Which of the following virulence factors has been associated with Pseudomonas aeruginosa? endotoxin A exoenzyme S pyogenic exotoxin endoenzyme T

exoenzyme S slide 23 Exoenzyme S is a phospholipase, works synergistically with protease lecithinase to hydrolyze lecithin, results in membrane disruption and cell death

Convergent evolution

explains the morphological similarity yet major genetic differences found between slime molds and fungi AND refers to two different organisms that develop similar characteristics in adaptation to similar environments.

What is the difference between fermented food and spoiled food?

fermented: foods intentionally altered during production by carefully controlling bacteria, yeasts, or molds spoiled: unintentional growth of bacteria on foods leading to quality deterioration

One of the strongest indications of infectious disease is: A) a rash. B) fever. C) pustules. D) vesicles. E) vomiting.

fever

Diatoms are algae whose silicon dioxide-containing shells are useful economically as

filters.

Considering that Cryptosporidium oocysts are resistant to chlorine, which step in water treatment protects us from this pathogen? Sedimentation Filtration Settling Coagulation Disinfection and storage

filtration

Rhizobium

fix nitrogen inside nodules on the roots of legumes

Rhizobium

fix nitrogen inside nodules on the roots of legumes.

The plague is typically transmitted via the bite of ticks. fleas. lice. mites.

fleas. Y. pestis forms a biofilm in flea guts, they starve and want to bite.

Growth of pathogens in a food generally does not result in perceptible changes in the quality of the food but the ingestion of this food can result in spoilage. foodborne illness. toxin infection. respiratory illness. aging.

foodborne illness - 2 types: foodborne intoxication from ingesting a toxin, and foodborne infection from ingesting live cells

Truffles are a mushroom. form a sheath around a root of an appropriate tree. are endomycorrhizas. are a lichen. are endomycorrhizas AND form a sheath around a root of an appropriate tree.

form a sheath around a root of an appropriate tree Truffles are ECTOMYCORRHIZAL fungi. This type of fungi grow around the plant cells, forming a sheath around the root. - S I'd pick the second one if given another try

Swarmer cells are

formed by sheathed bacteria.

Streptobacillus moniliformis is unusual in that it has a cell wall. forms spores. is unicellular. forms L-forms. is anaerobic.

forms L-forms

Phycobiliproteins are

found in cyanobacteria AND used to gather wavelengths of light that are not well absorbed by chlorophyll

Leptospirosis is often contracted: by eating infected animals. by eating contaminated vegetables. from contaminated animal urine. by the respiratory route.

from contaminated animal urine p.732 - Leptospira interrogans - slender, aerobic G- spirochete with hooked ends and more than 250 antigenic types Bacteria are excreted in an animal's urine and urine spots on the ground remain infectious while moist - contaminated urine is the main mode of transmission to other hosts. - Humans contract leptospirosis from water, soil or food contaminated with infected animal urine. Person-to-person transmission does not seem to occur. - gets into mucus membrane or wounds - not by respiratory route.

Which of the following is associated with cervical cancer? gonorrhea genital warts syphilis chancroid

genital warts p.751 - not exactly according to this page but who am I to quibble?

The most commonly identified waterborne illness in the United States is: A. amoebiasis. B. cryptosporidiosis. C. balantidiasis. D. giardiasis.

giardiasis.

The inflammatory effects of immune complexes lodged in the kidney is called renal phritis. rendema. glomerulonephritis. urethritis.

glomerulonephritis. Correct

Neisseria gonorrhoeae

gonococcal opthalmia in infants eyes is caused by _______ __________

Neisseria gonorrhoeae

gonorrhea is caused by _______ ___________

Streptococcus pneumoniae

gram + positive coccus pneumonia and meningitis, otitis media 5-50% of people are carriers treatment: antibiotics virulence factor: large capsule prevention: vaccine, teach kids now to sneeze and blow nose

Which of the following are most susceptible to complement lysis? A) Gram-positive bacteria B) Gram-negative bacteria C) Bacteriophages D) Prions E) Acid-fast bacteria

gram negative bacteria

Escherichia coli

gram negative rod makes vitamin K in intestines *enterotoxigenic* toxins that hit GI & increase fluid loss *enteropathogenic* wasting form of infant diarrhea *enterohemorrhagic* hemorrhage causing kidney damage causes 70% of *traveler's diarrhea* & 80% of *UTI's* Treatment: Pepto Bismol, hydrate, antibiotics Prevention: don't drink the water Strain: 0157:H7 infamous strain, lethal because it got Shiga toxin through transformation Infection: minimum infectious dose is 100 cells Signs/symptoms: gastroenteritis, fever, abdominal pain, dysentery, hemolytic uremic syndrome rarely

Legionella species

gram negative rods, fastidious eaters, motile, *aqueous*, aerosolized water transmits, hot tubs *Legionnarie's*- pneumonia; american legion members got it from water in AC at bicentennial gathering *Pontiac Fever*- flu-like, fever, not fatal

One virulence factor of Listeria monocytogenes is the ability to: A. release lactic acid B. produce abundant, branching hyphae C. lyse RBCs D. form endospores E. grow inside host cells

grow inside host cells

Streptococcus pneumoniae

otitis media is caused by __________ ___________ middle ear infection, fluid and inflammation behind ear drum

Clostridium, Lactobacillus, and Propionibacterium all

oxidize organic compounds AND use organic compounds as terminal electron acceptors.

All of the following are methods used to detect total coliforms in a water sample EXCEPT ones that are: - ONPG/MUG test - Presence/Absence test - MPN method - Membrane Filtration

p.793 - methods used to detect total colifroms in a water sample: - ONPG/MUG test - lactose-fermenting bacteria hydrolyze ONPG - turns yellow (all coliforms do this) - E. coli produces an enzyme that hydrolyzes MUG, making a (blue) fluorescent compound - Presence/Absence test - add 100 ml water to lactose-containing broth that is selective for G- rods - inverted tube to trap gas. Yes gas, test tube to confirm coliforms are present. - Most probably number (MPN) method - successive dilutions, inverted tubes to trap gas, further testing to confirm coliforms. - Membrane Filtration - filter that retains bacteria. Place filter on a lactose containing selective and differential agar medium.

Why do bacteria generally predominate on fresh meats?

p.801 - as a general rule, bacteria will predominate in fresh meats and other moist, pH neutral, nutrient-rich foods. Yeasts and molds can also grow in these foods but the more rapid increase in bacteria overwhelms the competitors. When conditions such as lack of moisture or high acidity restrict the growth of bacteria, fungi predominates despite their relatively slow growth.

Which of the following may lead to infertility? Vaginosis Cystitis Pelvic inflammatory disease Pyelonephritis All of these

pelvic inflammatory disease Correct

The disease that results from Chlamydia trachomatis possibly attaching to sperm and ascending into the fallopian tubes is: conjunctivitis. neonatorum ophthalmia. pelvic inflammatory disease. trachoma.

pelvic inflammatory disease Trachoma is an infectious disease caused by bacterium Chlamydia trachomatis. The infection causes a roughening of the inner surface of the eyelids. This roughening can lead to pain in the eyes, breakdown of the outer surface or cornea of the eyes, and eventual blindness.

Untreated gonorrhea in males may lead to all of the following EXCEPT pelvic inflammatory disease. prostatic abscesses. sterility. orchitis. urinary tract infections.

pelvic inflammatory disease.

chapter quiz - first time

perfect score - all correct

The sac which surrounds the heart is called the endocardium. pericardium. atrium. myocardium.

pericardium. peri- = around endocardium is lining inside heart atrium is heart chamber myocardium is heart muscle

In many aquatic habitats, the growth of algae and cyanobacteria is limited by the amount of phosphorus. calcium. nitrites. sulfur. nitrogen.

phosphorus

Free-floating, photosynthetic organisms found in marine environments are

phytoplankton

The initial attachment required for establishment of an intestinal infection is by A. flagella. B. cilia. C. pseudopodia. D. pili.

pili Attachment is always about pili...

The menigococcus attach to the mucus membrane via: A. flagella. B. pili. C. cilia. D. a capsule. E. axial filaments.

pili flagella = movement pili = attachment cilia = on eukaryotic cells capsule = anti-phagocytic

Gonococci selectively attach to certain epithelial cells by: pili. flagella. cilia. actin bridges. flagella AND cilia.

pili pili are for attachment. Flagella are movement. Cilia for movement of eukaryotic cells. N. gonorrhoeae use pili to attach to non-ciliated cells. Can't attach to squamous or ciliated cells.

The floating, surface microbial community that drifts with the currents and waves.

plankton

Fungi are particularly adept at infecting

plants

Streptococcus pyogenes causes all of the following diseases except A. strep throat B. scarlet fever C. pneumococcal pneumonia D. glomerulitis E. All of the choices are correct

pneumococcal pneumonia - is caused by Streptococcus pneumoniae, a Gram-positive diplococcus known as pneumococcus.

Streptococcus pneumoniae

pneumonia is caused by _________ __________ breathing issues, cyanosis, fever

With poliomyelitis, we worry about

post-polio syndrome - muscle pain, increased weakness and muscle degeneration 15-50 years after surviving acute paralytic poliomyelitis. It is a secondary effect of the initial damage. The nerves that took over for the ones killed finally give out and die. Salk (IPV) and Sabin (OPV) vaccines

The problem(s) with using sludge as a fertilizer is/are the A. presence of heavy metals and similar pollutants. B. presence of pathogenic organisms and viruses. C. inhibitory effect it has on plant growth. D. stimulatory effect it has on methane production. E. presence of heavy metals and similar pollutants AND presence of pathogenic organisms and viruses.

presence of heavy metals and similar pollutants AND presence of pathogenic organisms and viruses.

In the 1930s Kluyver and van Niel proposed a classification scheme based on

presumed evolutionary relationships.

Bdellovibrio

prey on other bacteria.

Which group is composed of autotrophs? Secondary consumers Primary consumers Rotters Primary producers Decomposers

primary producers

Lactic acid bacteria consume lactic acid, allowing them to grow on foods such as yogurt. produce lactic acid, allowing them to produce foods such as yogurt. are important spoilage organisms. can grow on lemons. produce lactic acid, allowing them to produce foods such as yogurt AND are important spoilage organisms

produce lactic acid, allowing them to produce foods such as yogurt AND are important spoilage organisms

Aflatoxins

produced by Aspergillus species; found in grains, peanuts; carcinogenic

The organisms that convert carbon dioxide into organic matter are the secondary consumers. primary consumers. producers. decomposers.

producers

Gonyaulax

produces a non-protein neurotoxin AND is a dinoflagellate.

Staphylococcus epidermidis can be characterized by all of the following except that it: A. does not produce coagulase. B. is not pigmented. C. produces a yellow pigment. D. can be associated with nosocomial infections.

produces a yellow pigment.

Propionibacterium

produces proionic acid AND is responsible for the holes in swiss cheese.

Herpes simplex, like other ulcerating genital diseases,: promotes the spread of AIDS. inhibits the spread of AIDS. inhibits the growth of other viral disease. is easily cured with antibiotics.

promotes the spread of AIDS

Cryptosporidium parvum

protist; cryptosporidiosis

Toxoplasma gondii

protist; taxoplasmosis

Sugar is used in the making of fermented sausages to help preserve the sausage. provide raw material for the fermentation process. sweeten the taste. counteract the action of the salt in the sausage.

provide raw material for the fermentation process.

Sarcodina move by means of

pseudopodia

Which of the following is most likely to grow at refrigerator temperatures? mesophiles psychrophiles thermophiles acidophiles

psychrophiles They like -5-15 degrees Celsius. - The ideal refrigerator temperature is 35° F (1.6° C) Mesophiles like 20-45 degrees Celsius (people are 37C) Thermophiles like 45 to 70 degrees Celsius - hot tubs and compost piles. Acidophiles - organisms are those that thrive under highly acidic conditions (usually at pH 2.0 or below).

Deuteromycetes has been further classified using

rRNA analysis

The common name for tularemia is Bang's disease. rabbit fever. Hansen's disease. Chagas' disease.

rabbit fever. Tularemia is a zoonotic disease that occurs among wild animals. Winter months people get it from skinning rabbits. Also called deer fly fever - from bites of infected ticks and deer flies in summer. Can also contract tularemia from eating contaminated meat or from dust arising from mowing or from rodent-infested buildings.

Stained smears of the surface of the eye might be useful in diagnosing: A. influenza. B. herpes. C. polio. D. rabies

rabies p.710 - Rabies can be diagnosed before death by identifying the virus in stained smears collected from the surface of the eyes.

Streptomyces

resemble fungi in their pattern of growth AND produce a number of antibiotics

Streptomyces

resemble fungi in their pattern of growth AND produce a number of antibiotics.

Meningococcal meningitis is typically acquired through the: A. genitourinary tract. B. respiratory tract. C. gastrointestinal tract. D. skin.

respiratory tract p.699 - transmission of Neisseria meningitiis (meningococci) is through airborne droplets that attach by pili to mucus membranes and multiply.

Which of the following involve symbiotic relationships? mycorrhizae AND ruminants ruminants AND myxobacteria rhizobia, mycorrhizae, AND ruminants myxobacteria, ruminants, AND rhizobia rhizobia AND ruminants

rhizobia, mycorrhizae, AND ruminants

The zone of soil that surrounds the roots and contains a variety of organic exudates is called the mycorrhizae. rootsphere. rhizosphere. geosmin

rhizosphere

The heart chamber that passes blood to the lungs is the left ventricle. right ventricle. right atrium. left atrium.

right ventricle. Left ventricle send blood to the body Right atrium sends blood to right ventricle Left atrium sends blood to left ventricle

The final characteristics of beer such as color, flavor, and foam primarily depend on the alcohol. whey. adjuncts. wort. roasted malt.

roasted malt

The term "potable water" refers to water that is only good for irrigation of crops. safe to drink. in carrying containers. safe to swim in but not to drink. contaminated with chemicals.

safe to drink

Streptococcus pyogenes

scarlet fever is caused by __________ _______ prophage (virus inside bacteria)

In which phase of sewage treatment are trickling filters sometimes used? A. primary treatment. B. secondary treatment. C. tertiary treatment. D. quaternary treatment.

secondary treatment

The nerves that carry information to the central nervous system (CNS) are termed: A. motor nerves. B. endocrine nerves. C. sensory nerves. D. cerebrospinal nerves.

sensory nerves - sensory nerves TO the CNS, motor nerves AWAY from the CNS.

Identification of the various strains of N. meningitidis is through: A. Gram staining. B. serogrouping. C. fluorescent antibodies. D. titering.

serogrouping Neisseria meningitidis - gram-negative diplococcus with capsule. Most serious infections are due to serotypes A, B, C, W and Y.

An early attempt by Cohn at bacterial classification grouped bacteria according to their

shape.

Sphaerotilus and Leptothrix are examples of

sheathed bacteria

Reactivation of chickenpox is called: A) shingles. B) herpes zoster. C) pneumonia. D) exanthems. E) shingles AND herpes zoster.

shingles AND herpes zoster

The rubeola virus contains: A) single-stranded DNA. B) double-stranded DNA. C) single-stranded RNA. D) double-stranded RNA. E) single-stranded RNA OR double-stranded RNA.

single-stranded RNA. Rubella virus of togavirus family: - enveloped, single-stranded RNA • Easily cultivated in cell cultures

Your first patient of the day is complaining of pain in the area of his new tattoo. When you examine the tattoo, you note that the area around it is firm, red, and swollen. There is no evidence of pus, and your patient has a normal pulse, and no fever. You tell your patient that you have seen this type of response to a tattoo before, and that it is called a granulomatous reaction. You give him more information on the condition. You tell your patient that when he got his tattoo, his first line of defense of his innate immunity was breached because his skin was damaged by the process. He asks you to explain what is included in the first line of defense and you tell him: - skin, mucous membranes, and lymphocytes. - mucous membranes. - skin and mucous membranes. - skin - white blood cells.

skin and mucous membranes.

When cellular slime molds run out of food, they form a

slug

Product of mold-fermentation

soy sauce

In higher organisms, successful mating can occur between members of the same

species

The basic taxonomic unit in the classification scheme of plants and animals is

species

In E. coli O157:H7, the O157:H7 refers to the

specific LPS and flagella type present.

Undesirable biochemical changes in food

spoilage

Foods that have been unacceptably changed due to uncontrolled bacterial growth are called spoiled. refrigerated. preserved. fermented. oxidized.

spoiled

Which of the following has been involved in causing small epidemics in the U.S.? tineasis candidiasis trichomoniasis sporotrichosis

sporotrichosis Fungal wound infection - more serious than skin mycoses. slide 49 Sporotrichosis (Rose Gardener's Disease)

Which of the following foods has the greatest amount of available water? Steak Cake Jam Syrup Bread

steak p.801 - fresh meats and milk have plenty of water to support the growth of many microbes. Bread, nuts and dried foods are relatively dry. Jams, jellies and some other sugar-rich foods are seemingly moist but most of that water is chemically interacting with the sugar, making it unavailable for use by microbes. Similar for salty foods.

Pseudomonas aeruginosa

swimmer's ear is caused by ________ _________

Pseudomonas aeruginosa

swimmer's ear, folliculitis, UTI Gram - negative rod motile 100,000 die per year aerobic blue/green pigment virulence factors: endotoxin exotoxin slime layer fimbriae disease: burn victims & people with cystic fibrosis are susceptible treatment: combination therapy bc resistant

Lichens are

symbiosis between a fungus and photosynthetic partner

Treponema pallidum

syphilis gram negative spirochete sensitive to heat, disinfectants, dry, oxygen, change in pH transmission: sexual or birth signs/symptoms: primary- *chancre* red hard painless bumps secondary- then systemic w/ rash on palms and soles of feets tertiary- dementia, cario problems, joints, *gumma*s rubbery lesions anywhere on body congenital syphilis: notched, barrel teeth treatment: penicillin G Tuskegee study: 400 black men left to let disease fester for 40 years

The disease that involves the muscles and often manifests itself first with spasms of the jaw muscles is: - polio. - rabies. - tetanus. - gastritis.

tetanus

The stage of Lyme disease that is characterized by arthritis is the fourth. third. second. primary. intermediate.

third p.672 - the third stage of Lyme disease (Late persistent infection) is characterized by ARTHRITIS and the affected joints have high concentrations of reactive immune cells and immune complexes. Correct

The symptom at the site of an animal bite that suggests rabies as a possible diagnosis is: A. inability to heal. B. blue-green pus. C. tingling or twitching. D. festering without fever.

tingling or twitching p.709 - the characteristic symptom is a tingling or twitching sensation at the sight of viral entry, usually an animal bite.

Staphylococcus aureus is the causative agent of: bacterial vaginosis. puerperal fever. toxic shock syndrome. gas gangrene. bacterial vaginosis AND puerperal fever

toxic shock syndrome bacterial vaginosis -> E. coli and others puerperal fever -> group A streptococcus gas gangrene -> Clostridum perfringens Toxic Shock Syndrome -> strains of Staphylococcus aureus that produce toxic shock syndrome toxiToxic Shock Syndrome -> strains of Staphylococcus aureus that produce toxic shock syndrome toxin-1 (TSST-1) or other related exotoxins (G+ bacteria)n-1 (TSST-1) or other related exotoxins (G+ bacteria)

Subacute bacterial endocarditis is usually caused by bacteria: - trapped in thin blood clots on a deformed heart valve and multiplying very rapidly. - trapped in thin blood clots on a deformed heart valve and multiplying slowly. - attaching to and colonizing the walls of the atria of the heart, multiplying very rapidly. - attaching to and colonizing the ventricles of the heart, multiplying slowly.

trapped in thin blood clots on a deformed heart valve and multiplying slowly. Slowly - subacute happens more gradually. Acute - happens rapidly.

Bacteria which cause subacute bacterial endocarditis may gain access to the bloodstream by trauma. dental procedures. brushing teeth. ingestion. trauma, dental procedures AND brushing teeth.

trauma, dental procedures AND brushing teeth. p.667 - The bacteria the cause infective endocarditis may enter the bloodstream from another infected body site or during DENTAL PROCEDURES, TOOTH BRUSHING, OR TRAUMA.

The limited type of leprosy in which cell-mediated immunity suppresses proliferation of the bacilli is called: A. lepromatous. B. limited. C. immune. D. tuberculoid.

tuberculoid Cell-mediated immunity successfully stops the proliferating bacteria, is called tuberculoid leprosy (pauchibacillary) - this isn't transmitted. When cell-mediated immunity to M. leprae fails to develop or is suppresses, unrestricted growth of M. leprae occurs, leads to a form called lepromatous leprosy (multibacilliary). Mucus of throat/nose contain high number of pathogen and is easily transmitted to others.

The development of lymph node enlargement in the region of a skin ulcer after a tick or insect bite or handling of a wild animal suggests brucellosis. endocarditis. septicemia. tularemia.

tularemia. p.676 - Tularemia is a zoonotic disease that occurs among WILD ANIMALS...in the western US infections mostly result from the bites of infected TICKS AND DEER FLIES (insects) Brucellosis is a chronic infection of domestic animals. Zoonosis.

Atmospheric nitrogen is used directly by plants. used directly by animals. converted into urea by bacteria. directly used to make carbohydrates. turned into ammonia by microorganisms.

turned into ammonia by microorganisms.

Salmonella typhi

typhoid fever- diarrhea and vomiting, small red dots gram negative rod fecal/oral enteric Treatment: antibiotics, hydrate, replace electrolytes Prevention: sanitation, safe food handling, immunizations CDC: 1.5 million cases, ingested in food Five F's: food, fingers, flies, feces, fomites

Helicobacter pylori appears to have some connection with: acid reflux disease. ulcers.dental caries. stomach cancer. ulcers AND stomach cancer.

ulcers AND stomach cancer.

The cavities inside the brain are termed: A. sinuses. B. ventricles. C. cavities. D. sulci.

ventricles - 4 ventricles, where cerebrospinal fluid is made, exits a base of brain and bathes brain/spinal cord CSF cushions and supports the brain and transports nutrients and other materials throughout the CNS.

Encephalitis is most commonly caused by a: A.BacteriaB.ProtozoanC.VirusD.HelminthE.All of the choices are correct

virus

Hepatovirs A (HAV)

virus, Hepatitis A

Human Papillomavirus (HPV)

virus, genital warts, cervical cancer, penile cancer

Influenzavirus

virus, influenza

Morbillivirus

virus, measles

The amount of moisture available in foods is designated by the term fluid availability. water activity. dampness quotient. aqueous usability

water activity a subscript-w. For pure water it = 1.0

In the late nineteenth century, immigration from Ireland to the United States occurred in high numbers due to an infection of potatoes by

water molds

Plant pathology grew in importance as a field of study after it was shown that the Irish potato blight was caused by

water molds

The preferred host of Ixodes scapularis is the moose. wood rat. ground squirrel. white-footed mouse. human.

white-footed mouse.

go back to bogrady microbiology ch 24 set for help jkins1389 Mirobiology h 25 chelsea_walter Chapter 25 Microbial Diseases of the Digestive AND Chapter 26 Urinary & Reproductive Systems julioambriz Ch. 24- Study Bank Questions JKMoe Chapter 25: Microbial Diseases of the Digestive System ** https://www.studystack.com/flashcard-485339 sobrok at https://quizlet.com/252095176/microbiology-ch-24-flash-cards/

windham_mcguire - micro final practice test 25 - q/a

Biofilms are likely to be found in environments that are nutrient rich. that are relatively warm. with rapidly flowing water. with low nutrients.

with low nutrients p.769 - LOW-NUTRIENT ENVIRONMENTS such as lakes, rivers and streams are common...most microbial growth in these settings is in BIOFILMS, and the cells are shed from the biofilm into the aqueous solution.

Fungi capable of dimorphism grow either as

yeast-like or mycelium.

The most promising sources of single cell protein are bacteria. viruses. worms. yeasts.

yeasts Saw this, don't think it applies to my chapter but hey.

Which of the following is transmitted by mosquitoes? plague yellow fever malaria tularemia yellow fever AND malaria

yellow fever AND malaria plague is fleas, Y. pestis bacteria tularemia is zoonosis - wild animals, ticks Yellow fever is Aenes mosquitos and a virus malaria is Anopheles mosquitoes and a Plasmodium protozoa

The disease caused by an enveloped single-stranded RNA arbovirus of the flavivirus family is chikungunya. herpes. malaria. yellow fever. brucellosis

yellow fever. AIDS - not arbovirus malaria - protozoa herpes - not arbovirus brucellosis - bacteria - chik is due to the chikungunya virus, enveloped, ssRNA virus of family Togaviridae. Aedes mosquitoes. Yellow fever is caused by an enveloped, ssRNA arbovirus of the family Flaviviridae. - transmitted by bite of infeccted Aedes or Haemagogus mosquitoes, it biological vector. Correct

Respiratory psittacosis

Chlamydophila gram - bacteria

The site in a photosynthetic eukaryotic cell where photosynthesis occurs is the

Chloroplast

T/F - Fever often enhances bacterial survival during an infection.

False

T/F - Gram-negative bacteria are less susceptible to complement lysis than Gram-positive bacteria.

False

T/F - Interferon directly interacts with and destroys viruses.

False

T/F - Neutrophils are the second phagocytic cell to respond to an infection.

False

T/F - Substances with a molecular weight of less than 10,000 Daltons make good antigens.

False

T/F Fungi are often capable of locomotion through the use of flagella.

False

T/F Fungus grows well on fruits and many vegetables due to their alkaline pH.

False

Chickenpox and measles are both acquired by the respiratory route. T/F

True

Clumping factor, coagulase and protein A serve to coat Staphylococcus with host proteins. T/F

True

Coagulase-positive S. aureus is often involved in disease. T/F

True

Cyclospora cayetanensis has, so far, not been identified with an animal source. T/F

True

Helicobacter pylori appears connected to stomach cancer and ulcers. T/F

True

If a compost pile is turned frequently and other conditions are adequate for aerobic digestion, the composting can be completed in 6 weeks. T/F

True

The tart taste of yogurt, pickles, and sharp cheeses is due to the presence of acetic acid. benzoic acid. lactic acid. sorbic acid. hydrochloric acid.

lactic acid. p.803

Coliforms are Gram-positive. spore-forming. lactose-fermenting. coccus-shaped. All of the choices are correct.

lactose-fermenting Enterobacteriaeae - Gram-negative rods, non-spore formers, anaerobic, lactose fermentors.

Which of the following contains bacteriochlorophyll?

Chromatium, Thisopirillum, Thidictyon

Rhinovirus (common cold)

Picornaviridae RNA virus

Sulfur occurs in all living matter primarily as a component of carbohydrates. T/F

False - component of amino acids methionine and cysteine

Coccidioidomycosis is

a fungal disease caused by Coccidioides sp.

Molds

Filamentous fungi

Why don't medications such as acyclovir cure oral herpes simplex infections?

- Do not affect latent virus and so cannot cure.

The most common cause of traveler's diarrhea is: A) Shigella spp. B) Salmonella enterica. C) Giardia lamblia. D) Escherichia coli. E) Campylobacter jejuni

- E. coli "Contrary to common belief, food - not water - is the primary cause. The CDC estimates up to 80% of cases of travelers' diarrhea are caused by bacteria. The most common bacterium that causes travelers' diarrhea is enterotoxigenic E. coli, one of six classes of enterovirulent E. coli. Most E. coli are harmless."

Which of the following organisms feeds on red blood cells? A) Giardia lamblia B) Escherichia coli C) Taenia spp. D) Vibrio parahaemolyticus E) Entamoeba histolytica

- Entamoeba histolytica "E. histolytica is also capable of phagocytizing red blood cells."

A mysterious sequel to Campylobacter jejuni infections is: A. Reye's syndrome. B. Tourette's syndrome. C. Pasteur's syndrome. D. Guillain-Barré syndrome.

- Guillain-Barre syndrome. Begins about 10 days after the onset of diarrhea, with tingling in the feet followed by progressive paralysis of the legs, arms and rest of the body. Most need to be hospitalized but recover completely - about 5% die even with treatment.

What are possible complications of mumps?

- I think: miscarriage, infertility

Which description of vegetative Bacillus anthracis is correct? It is an endospore-forming, Gram-negative, non-motile, rod-shaped bacterium. It is a Gram-positive, non-motile, encapsulated, spore-forming, rod-shaped bacterium. It is an endospore-forming, Gram-positive, motile, rod-shaped bacterium. It is a Gram-positive, non-motile, spherical, encapsulated bacterium. It is an acid-fast, non-motile, spherical, nonspore-forming, encapsulated bacterium.

- It is a Gram-positive, non-motile, encapsulated, spore-forming, rod-shaped bacterium. I picked A - wrong

The most important vector of Lyme disease in the eastern U.S. is: a. Dermacentor virabilis b. Dermacentor andersoni c. Staphylococcus aureus d. lxodes scapularis

- Ixodes scapularis Black-legged (deer) tick Ixodes scapularis most important vector.

You are looking after your sister's cat Singa. While playing one evening, Singa accidentally bites you instead of the toy mouse you are holding. The bite wound is very small but is in the fleshy part of your hand. You wash the wound, which is not bleeding much, and put a band aid on it. The next day, your whole hand is red, swollen, and painful, and some lymph nodes in your arm are enlarged and tender. You call your sister, who is an RN. She urges you to go to the doctor, telling you that animal bites can sometimes be infected, and that if that is the case, you will need treatment before the infection gets any worse.The PA goes on to explain that cat bite infections and bartonellosis (cat scratch disease) differ in that: A) the virulence factors of P. multicoda are as yet unknown while B. henselae has an antiphagocytic capsule. B) P. multicoda is transmitted by cat bites, while B. henselae may be transmitted by bites or scratches. C) P. multicoda may be transmitted form person-to-person while B. henselae can only be transmitted by cat bites. D) P. multicoda is a Gram-positive organism while B. henselae is a Gram-negative organism. E) cat bite infections can be prevented with prophylactic antibiotics while bartonellosis may be prevented with an attenuated vaccine.

- P. multicoda is transmitted by cat bites, while B. henselae may be transmitted by bites or scratches. Not A or D, (This was on my chapter quiz) I chose B = Correct

Which of the following is included in GALT? A) thymus B) Peyer's patches C) tonsils D) liver E) salivary glands

- Peyer's Patches GALT - Gut Associated Lymphatic Tissue

Acute gastroenteritis that occurs after an incubation period of two to three days and commonly affects children is probably caused by: A) Giardia. B) rotavirus. C) Salmonella. D) Staphylococcus aureus. E) Trichinella

- Rotavirus Rotaviruses are naked viruses with a double-walled capsid and a dsRNA genome in 11 segments. The viruses represent a major subgroup of the family Reoviridae. - fecal-oral route

Viral gastroenteritis in infants and children is most commonly caused by: A. herpes. B. hepatitis B. C. Norwalk virus. D. rotavirus.

- Rotavirus p.648 - most cases of viral gastroenteritis in infants and children around the world are caused by rotaviruses.

Disease-causing exotoxins are produced by all of the following organisms EXCEPT: A) Clostridium perfringens. B) Vibrio cholerae. C) Shigella dysenteriae. D) Staphylococcus aureus. E) Clostridium botulinum

- Shigella dysenteriae S. dysenteriae strains can make Shiga toxin - an CYTOTOXIN, a chromosomally encoded A-B toxin that interacts with ribosomes in kidney cells, stopping protein synthesis which leads to cell death. Shiga toxin is responsible for HUS - hemolytic uremic synfrom where RBCs lyse in the tiny blood vessels, resulting in anemia and kidney failure. Clostridium perfringins makes the exotoxin alpha toxin that is a pore-forming protein; it causes potassium and fluid leakage from cells, is responsible for gas gangrene and myonecrosis in infected tissues. Vibrio cholerae makes cholera toxin, an enterotoxin, bacteriophage A-B toxin that activates ion transport channels so electrolytes and water leave the cells. Staphylococcus aureus makes the exotoxin exfoliatin - Staphylococcal scalded skin syndrome. Clostridium botulinum makes the exotoxin botulinum toxin.

Formation of biofilms attached to fibronectin and fibrinogen coating plastic devices like catheters and heart valves is a virulence mechanism of: - Staphylococcus - Streptococcus - Clostridium - Pseudomonas

- Staphylococcus slide 15 - Staphylococcus epidermiditis - Adheres to and colonizes medical devices by binding to fibronectin, which quickly coats surgical implants. Biofilms also problematic

Which of the following statements about salmonellosis is FALSE? A) It is a bacterial infection. B) Severity of disease depends on number of organisms ingested. C) A healthy carrier state exists. D) The mortality rate is high. E) It is often associated with poultry products

- The mortality rate is high. It is bacterial: Salmonella enterica - G- rod, member of Enterobacteriaceae. Often short-lived and mild depending on strain, dose MILD AND MOST PEOPLE RECOVER W/O ANTIMICROBIAL TREATMENT Poultry, eggs often contaminated; many other products (e.g., tomatoes, alfalfa sprouts) have started outbreaks.

Which of the following statements concerning mumps virus infection is FALSE? - The virus is present in urine approximately 10 days after the onset of symptoms. - The testes can become inflamed about 2 to 3 weeks after the onset of symptoms. - The illness frequently begins with painful swelling of one or both parotid glands. - The virus is transmitted in saliva and respiratory secretions.

- The testes can become inflamed about 2 to 3 weeks after the onset of symptoms. LOOK IN BOOK

"Rice water stools" are characteristic of: A) salmonellosis. B) cholera. C) bacillary dysentery. D) amebic dysentery. E) tapeworm infestation

- cholera

Which of the following is NOT true about diphtheroids? A) They are part of the normal microbiota of the skin. B) They are responsible for body odor. C) They include Cutibacterium acnes. D) They include Malassezia species. E) These are all true.

- They include Malassezia species. These little yeasts are not lumped in with the diphtheroids.

The existence of extensive scalded skin syndrome does not indicate that Staphylococcus is growing in all the affected areas. Why not? Scalded skin occurs when a person eats exfoliatin-contaminated food, not by an infection. This condition isn't caused by Staphylococcus at all. This condition is caused by an endotoxin produced by certain strains of this microbe, and NOT directly by the microbe itself. This microbe grows in the blood; endotoxins it releases are transported to the skin, where it causes the observed effect. This condition is caused by an exotoxin produced by certain strains of this microbe, and NOT directly by the microbe itself.

- This condition is caused by an exotoxin produced by certain strains of this microbe, and NOT directly by the microbe itself.

Giardiasis may be contracted from: another person. clear mountain streams. chlorinated city water. cold filtered beer. another person, clear mountain streams AND chlorinated city water.

- another person, clear mountain streams AND chlorinated city water.

Tetanus antitoxin is: - antibody against the bacteria. - inactivated toxin. - inactivated bacteria. - antibody against the toxin.

- antibody against the toxin A TOXOID is a vaccine; it is used to prevent disease. An ANTITOXIN is a treatment; it is used when a problem is immediately at hand. And that is why this is the answer it is. Correct

S. aureus clumping factor: A) causes fibrinogen to clump together. B) causes bacteria to clump together in plasma. C) reacts with prothrombin. D) produces staphylothrombin. E) causes phagocytes to clump bacteria.

- causes bacteria to clump together in plasma. Table 22.3 - clumping factor - attaches the bacteria to fibrin, fibrinogen and plastic devices.

You are studying for your NCLEX exams with a group of friends. Each of you presents a case study to the other students in your group. Your friend Sue gives the following information in her case study: the patient is a child, who presents with a high fever (41oC) and a rash. The parents report that three days prior to the onset of the rash, the child had a runny nose, diarrhea, and conjunctivitis, all of which they thought indicated influenza. The child has not received any of the common recommended vaccines because until recently, he had been receiving chemotherapy for acute lymphoblastic leukemia that was diagnosed at 9 months. Confirmation of the rubeola diagnosis can by made by: A) measuring the intensity of the rash. B) presence of rubella antibodies in the blood. C) detecting Koplik spots on the oral mucosa. D) detecting rubeola viruses using a Gram stain. E) treating the patient with antibiotics.

- detecting Koplik spots on the oral mucosa. Rubeola is measles. KOPLIK SPOTS of mucous membranes are identifying feature.

The principal species of Staphylococcus found on the skin is: A. aureus. B. acnes. C. pyogenes. D. epidermidis.

- epidermidis

The surface layer of the skin is the: A) cutaneous. B) keratin. C) epidermis. D) dermis. E) hypodermis.

- epidermis Epidermis: SURFACE LAYER made from layers of flat cells • Outermost =dead,filled with water-resistant keratin • Constantly flake off and replaced • Dermis: nerves, glands, blood and lymphatic vessels • Subcutaneous tissue: fat, other cells that support skin

The unique characteristic of Lyme disease is: A. erythema migrans. B. induration. C. carbuncle. D. furuncle.

- erythema migrans Signs and Symptoms • Early localized infection: erythema migrans, or circular skin rash, follows few days, weeks after tick bite Erythema migrans (New Latin, literally, "migrating redness") is an expanding rash often seen in the early stage of Lyme disease.

Clostridium difficile-associated diarrhea is usually preceded by: A) eating contaminated food. B) a blood transfusion. C) extended use of antibiotics. D) improper food storage. E) travel to an underdeveloped country

- extended use of antibiotics. Clostridium dificile Infection (CDI) Clostridium difficile: Gram-positive, obligate anaerobic rod • Forms endospores highly resistant to disinfectants, environmental conditions, so control is difficult • Produce one or more cytotoxins Primarily in hospitalized patients on ANTIBIOTIC THERAPY

A young mother brings her three-month old baby into the doctor's office where you work as an RN. The baby is miserable and the mother reports that he has developed a fever, which is why she has brought him to the doctor. She tells you that the child is particularly upset when she changes his diaper and that he seems to have diaper rash, which she attributes to the fact that her son's diaper isn't always changed promptly enough at the day-care facility he goes to daily. In fact, she has recently switched him to another facility. You remove the child's diaper and immediately see that he has a very red rash on his buttocks and genital area. There is also red, scaly skin in the area where his diaper touches his thighs. You suspect that this may be more than just a typical case of diaper rash. You send a skin scraping to the hospital lab for analysis. The report comes back indicating the presence of unicellular organisms that stained with calcofluor white stain, which binds to chitin. This organism is thus a ________, which is a ________ organism. A) virus; parasitic B) bacterium; prokaryotic C) protozoan; eukaryotic D) fungus; eukaryotic E) protozoan; multicellular

- fungus; eukaryotic

Which of the following is a fairly reliable method of characterizing strains of S. aureus? A. complement fixation B. protein fingerprint C. genome typing D. LPS pattern

- genome typing I don't know what protein fingerprinting is, S. aureus is G+ so no LPS. Let's look at the DNA!

Gums bleeding while brushing one's teeth is most commonly associated with: A) halitosis. B) cavities. C) gingivitis. D) periodontitis. E) trench mouth

- gingivitis Red, inflamed gums, brush them and they bleed easily.

Which of the following is true about protein A? - it binds to the Fc region of antibody - it hides bacteria from phagocytes - it enhances the binding of phagocytes - it digests antibodies - it binds to the Fc region of antibody AND it hides bacteria from phagocytes

- it binds to the Fc region of antibody AND it hides bacteria from phagocytes

Which of the following is directly involved in the initiation of dental caries? a. Dentin b. Lactic acid c. Crevicular fluid d. Lysozyme e. Sucrose

- lactic acid

The preferred habitat of S. aureus is the: A. throat. B. urethra. C. nasal chamber. D. bladder.

- nasal chamber. S. aureus found in NOSTRILS of nearly everyone - 20% (text says 30%) of healthy adults carry continuously for year or more; 60% will be colonized during a given year

Skin and mucous membranes are mostly involved in: A) adaptive immunity. B) innate immunity. C) autoimmunity. D) irregular immunity. E) secondary immunity.

B) innate immunity.

he growth stage of the vector that is mainly responsible for transmitting Lyme disease is the: A. nymph stage. B. egg. C. moulter. D. adult.

- nymph stage

Influenza is caused by: - orthomyxovirus. - H. influenza. - cytomegalovirus. - adenovirus.

- orthomyxovirus. Not ch 14. That's ok.

Warts are caused by: A) papillomaviruses. B) parvovirus. C) adenoviruses. D) herpes virus. E) coronavirus.

- papillomaviruses. Warts are caused by Papillomaviruses that infect skin via minor abrasions • Warts are small tumors (papillomas) consisting of multiple protrusions of tissue covered by skin or mucous membrane

Approximately 90% of S. aureus strains are resistant to: A. methicillin. B. tetracyclin. C. polymyxin B. D. penicillin.

- penicilliin

The digestive tract is essentially one long tube. The order of the structures, beginning with the mouth, is: A) pharynx, esophagus, stomach, small intestine, large intestine. B) esophagus, pharynx, stomach, small intestine, large intestine. C) pharynx, esophagus, stomach, large intestine, small intestine. D) esophagus, stomach, pharynx, small intestine, large intestine. E) pharynx, stomach, esophagus, small intestine, large intestine

- pharynx, esophagus, stomach, small intestine, large intestine.

The initial attachment required for establishment of an intestinal infection is by: - flagella. - cilia. - pseudopodia. - pili.

- pili flagella is movement cilia is eukaryotic cells pseudopodia is for engulfing things pili are for attachment

Helicobacter pylori can grow in the stomach because it: A) hides in macrophages. B) makes a capsule. C) possesses an enzyme that neutralizes HCl. D) makes HCl. E) invades epithelial cells

- possesses an enzyme that neutralizes HCl.

The primary treatment for cholera is A. the administration of antibiotics. B. vaccination. C. by blood transfusion. D. simply rehydration. E. vaccination AND by blood transfusion.

- simply rehydration

The normal habitat of Clostridium tetani is: A. humans. B. animals. C. plants. D. soil and dust.

- soil and dust. "Spores of Clostridium tetani found in SOIL AND DUST"

Which of the following pertains to Borrelia burgdorferi? A. coccus B. bacillus C. spirochete D. filament

- spirochete

Streptobacillus moniliformis is unusual in that it: - forms spores. - spontaneously forms L-forms. - is anaerobic. - has a cell wall.

- spontaneously forms L-forms. Streptobacillary Rat Bite Fever slide 48 Streptobacillus moniliformis: facultatively anaerobic, Gramnegative pleomorphic rod; spontaneously develops L-forms lacking cell wall Correct

What is a Dane particle? A) the chlorine-resistant virus that causes HAV B) the infectious virion that causes HBV C) the spherical particles found in serum of patients with HBV D) the filamentous particles that cause HAV E) another name for an HBV surface antigen

- the infectious virion that causes HBV The complete virion - not just an empty envelope with no DNA inside it.

Microorganisms in abscesses often are not killed by antimicrobial agents because: - the microorganisms stop dividing. - of the chemical nature of the pus. - of the lack of blood vessels. - of the high level of oxygenation. - the microorganisms stop dividing, of the chemical nature of the pus AND of the lack of blood vessels.

- the microorganisms stop dividing, of the chemical nature of the pus AND of the lack of blood vessels.

Frequently sporotrichosis is caused when the infectious agent is introduced into the body by: - lotions. - scissors. - thorns. - animal bites.

- thorns Fungal wound infection - sporotrichosis (Rose Gardener's Disease). Think plants. Correct

The vector for RMSF is a(n): A) mosquito. B) flea. C) tick. D) fly. E) deer.

- tick Transmitted by tick bite. Humans are accidental hosts. Rickettsia rickettsii: tiny Gram-negative non-motile coccobacilli that are obligate intracellular bacteria

Why does secondary treatment of wastewater require O2?

- to encourage microbial growth. p.787 - microbial growth is actively encouraged during secondary treatment, allowing aerobic organisms to oxidize the biologically degradable organic material to carbon dioxide and water. - Secondary processes rely on the aerobic activity of bacteria.

All of the following infections can result from drinking contaminated water EXCEPT: A) Cyclospora infection. B) giardiasis. C) trichinellosis. D) cholera. E) cryptosporidiosis

- trichinellosis Trichinellosis, also called trichinosis, is caused by eating raw or under-cooked meat of animals infected with the larvae of a species of worm called Trichinella. Cholera - yes to waterborne - bacterial infection, causes water diarrhea. Giardia, Crypto, Cyclosporia as all about waterborne disease of lower digestive system caused by protozoans.

Intestinal protozoan infections are typically spread by:

- water

The secretions of the sweat and sebaceous glands provide ________ to the microbiota. A) water B) amino acids OR water C) lipids AND amino acids D) water, amino acids, AND lipids E) sebum OR amino acids

- water, amino acids, AND lipids They break down lipids into fatty acids and glycerol for food. Also, these products toxic to other bacteria. Break down proteins too.

The preferred host of Ixodes scapularis is the: A. wood rat. B. white-footed mouse. C. moose. D. human.

- white-footed mouse

The presence of long double stranded RNA (> 30 bp)

-Indicates infection by an RNA virus other than a retrovirus AND induces synthesis of interferon.

The complement pathway that is activated by mannose binding lectins is the

-Lectin pathway

Why is it that the tongue and cheek epithelium doesn't provide a sufficient anaerobic environment for plaque anaerobes to grow, but the surface of teeth might? A. The epithelium is supplied with oxygen by capillary beds. This makes it a relatively aerobic environment and hostile to anaerobes. B. The tooth enamel is supplied with oxygen by capillary beds. This makes it a relatively anaerobic environment and hostile to aerobes. C. The surface of the tongue and cheek are constantly scraped by the action of consuming food. Layers of bacterial growth that might help to supply an anaerobic environment are scraped away, exposing lower levels to oxygen-rich air. D. The surface of teeth (especially the molars) have many pits and crevices that can serve as 'pockets' for layers of bacteria to grow in. Once the layers get deep enough, the bottom portions are anaerobic. This isn't possible on the very smooth surface of the tongue and cheek epithelium.

. The epithelium is supplied with oxygen by capillary beds. This makes it a relatively aerobic environment and hostile to anaerobes. Correct

Why are older people (over 65) more likely to lose teeth than to develop cavities?

... -dry mouth - less saliva -waning immunity -don't take a good care of their teeth as before -chronic illnesses let microorganisms grow

Why does removal of plaque reduce the chance of forming cavities?

... Tooth decay occurs when foods containing carbohydrates (sugars and starches), such as breads, cereals, milk, soda, fruits, cakes, or candy are left on the teeth. Bacteria that live in the mouth digest these foods, turning them into acids. The bacteria, acid, food debris, and saliva combine to form plaque, which clings to the teeth. The acids in plaque dissolve the enamel surface of the teeth, creating holes in the teeth called cavities.

Place the steps of the disease pathway provided in the correct order to test your understanding of the pathogenesis of syphilis.

1. Treponema pallidum enters the body through tiny abrasions or mucous membranes; a chancre appears at site of infection. 2. Bacteria multiply locally, then enter the bloodstream and spread throughout the body. 3. Secondary symptoms appear, including skin rashes and infectious mucous patches; fetus may become infected, developing congenital syphilis. 4. Latent, asymptomatic period occurs; bacteria disappear from blood, skin, and mucous membranes. 5. Tertiary symptoms may appear, including heart and blood vessel defects, gummas, strokes, and neurological symptoms and mental illness.

The pH at which most fungi thrive is

5.0

The compost pile temperature at which pathogens, but not thermophiles, are killed is about A. 20-30ºC. B. 55-66ºC. C. 62-75ºC. D. 90-100ºC.

55-66ºC. - achieving high temperatures is important to kill pathogens that might be present in the solid waste. Thermophilic organisms (45-70ºC) thrive at the high temperature, so they continue decomposing the material.

A single exposure to HPV results in infection _______ of the time. A. 10% B. 30% C. 60% D. 100%

60% Not in my textbook...

Explain the difference between a biotype and a serotype.

A biotype is the biochemical profile of a particular microorganism, while the serotype is the difference in its surface structures/antigens that will lead to separate immune responses form an organism (characterized by presence of different antibodies in the individual's serum).

Pyrogens are: A) fever-inducing substances. B) fever-inhibiting substances. C) phagocytosis-enhancing substances. D) complement activators. E) pus-producing substances.

A) fever-inducing substances.

Interferons function to make cells: A) prevent viral replication. B) lyse when exposed to virus. C) non-motile when infected with virus. D) resistant to phagocytosis. E) form endospores.

A) prevent viral replication.

The lactobacilli, in their role as normal flora of the vagina, help the vagina resist infection by contributing to

ACIDITY OF THE VAGINA

Which causes uncontrolled growth of plant tissue, resulting in a tumor?

AGROBACTERIUM

Coliforms

ALL OF THE ABOVE

Hydrogen sulfide

ALL OF THE ABOVE

Luminescence

ALL OF THE ABOVE

Members of the family Enterobacteriacaeae

ALL OF THE ABOVE

The genus of bacteria that is able to fix nitrogen and form heterocysts is

ANABAENA

Lactic acid bacteria such as Lactococcus

ARE OBLIGATED FERMENTERS

The methanogens

ARE PART OF THE ARCHEA & OXIDIZE HYDROGEN GAS TO PRODUCE METHANE

Cyanobacteria

ARE PROKARGYOTES

Wolbachia are found only in

ARTHROPODS

Movement of spirochetes occurs by means of structures called

AXIAL FILAMENTS

Infects corneas causes blindness

Acanthamoeba Protozoa (amebae)

nosocomial (hospital) infections respiratory pathogens

Acinetobacter gram - bacteria

In treating an oil spill, why might biostimulation be preferred over bioaugmentation? In fact, biostimulation is not generally favored as an approach. Biostimulation often leads to the generation of antibiotic—resistant microbes, which become a problem for other organisms in the environment. Bioaugmentation is the favored approach because adding nutrients to the area simply enhances the activities of the bacteria already present. Adding new microbes (bioaugmentation) poses some risk—when new microbes are added to an environment, the consequences of those additions on resident populations are unknown. Adding nutrients (biostimulation) is safer because once those nutrients are used up, the microbial populations are likely to return to their original levels. Biostimulation and bioaugmentation are always used simultaneously. This is done because neither one of these approaches works well individually. Adding bacteria to an environment will never be successful unless nutrients are also added to that environment. Bioaugmentation involves adding novel microbes to boost the populations of resident microbes. These novel microbes are highly genetically engineered to enhance their abilities to metabolize pollutants such as oil, so are very effective. Biostimulation merely involves adding nutrients, so all microbes are favored, even those not performing a useful function. Biostimulation involves adding novel microbes to boost the populations of resident microbes. These novel microbes are generally genetically engineered to enhance their abilities to metabolize pollutants such as oil, so are very effective. Bioaugmentation merely involves adding nutrients, so all microbes are favored, even those not performing a useful function.

Adding new microbes (bioaugmentation) poses some risk—when new microbes are added to an environment, the consequences of those additions on resident populations are unknown. Adding nutrients (biostimulation) is safer because once those nutrients are used up, the microbial populations are likely to return to their original levels.

The problem of not being able to cultivate most microorganisms has led to the use of FISH. FISH and 16s rRNA analysis. 16s rRNA analysis. microscopy with fluorescent dyes. All of these methods help identify microorganisms in the environment.

All of these methods

Cryptosporidium parvum may infect pigs. humans. dogs. cattle. All of the choices are correct.

All are correct p.657 - Cryptosporidium parvum is difficult to control because it has a wide host range, infecting domestic animals such as dogs, pigs and cattle. Feces from these animals, a well as from humans, can contaminate food and drinking water.

Viral central nervous system infections may result in: A. meningitis. B. encephalitis. C. poliomyelitis. D. rabies. E. All of the choices are correct.

All of the above can be caused by viruses.

Fungi are important because of their ability to

All of the above.

Indicator organisms are A. Coliforms B. Enterococci C. Suggestive of fecal contamination D. Used in water quality tests E. All of the choices are correct

All of the choices are correct

Which of the following is/are the modes of transmission for T. francisella? Via ingestion of contaminated meat By tick bites Through minor cuts or abrasions By inhalation All of these

All of these p.675/676 Correct

Lactic acid bacteria are used to ferment and produce foods from A.dairy. B.vegetables. C.meat. D.All of these can be fermented by lactic acid bacteria to make foods

All of these can be fermented by lactic acid bacteria to make foods

Which of the following is used to cause flocculation? Methane Charcoal Chlorine Aluminum potassium phosphate (alum) Natrium potassium phosphate (nalum)

Aluminum potassium phosphate (alum)

Which statement describes the relationship between ammonia oxidizers and nitrite oxidizers? Ammonium oxidizers use ammonium as an energy source, oxidizing it to nitrate. The nitrate produced then serves as the energy source for nitrite oxidizers. Ammonium oxidizers use ammonium as an energy source, reducing it to nitrate. The nitrate produced then serves as the energy source for nitrite oxidizers. Ammonium oxidizers use urea as an energy source, oxidizing it to nitrite. The nitrite produced then serves as the carbon source for nitrite oxidizers. Ammonium oxidizers use ammonium as an energy source, oxidizing it to nitrite. The nitrite produced then serves as the energy source for nitrite oxidizers. None of these explains the relationship—ammonia and nitrite are produced by different nutrient cycles which also involve sulfur reduction.

Ammonium oxidizers use ammonium as an energy source, oxidizing it to nitrate. The nitrate produced then serves as the energy source for nitrite oxidizers.

Which statement does NOT apply to water treatment? Advanced treatment of wastewater is often designed to remove ammonia, nitrates, and phosphates. Secondary treatments are designed to convert suspended solids into inorganic compounds and microbial biomass. The primary treatment of wastewater is a physical process designed to remove materials that settle out. An important goal of wastewater treatment is the increase of the BOD that stimulates microbial growth. Biosolids that result from anaerobic digestion of sludge can be used to improve soils and promote plant growth.

An important goal of wastewater treatment is the increase of the BOD that stimulates microbial growth. Goal is to DECREASE the BOD

The genus of bacteria that is able to fix nitrogen and form heterocysts is

Anaabaena

The genus of bacteria that is able to fix nitrogen and form heterocysts is

Anabaena.

Most bacteria associated with the teeth and gums are A) Aerobes. B) Anaerobes. C) Facultative anaerobes D) None of the above.

Anaerobes

Bacillus anthracis

Anthrax gram + positive rod endospore aerobic virulence factors: exotoxins enter cells and disrupt intracellular signaling, apoptosis capsule reservoir: soil signs: *eschars* (black, painless lesions) wool sorter's disease, GI disease treatment: antibiotics, antitoxins prevention: vaccine for livestock & at risk people 2001: 22 cases in US from spores in the mail

Bacillus anthracis

Anthrax is caused by _______ ________

Why are signs and symptoms not quickly resolved in a person with tetanus when they are given antibiotics? Antibiotics will prevent the production of additional tetanospasmin but will not repair nerves already damaged by the toxin. Clostridium tetani is an acid-fast organism; it thus has a slow generation time, so it takes several weeks for an antibiotic to be effective in killing the cells. Clostridium tetani is an acid-fast organism; it thus has a waxy cell wall that prevents antibiotics from entering the cell. The antibiotics are given prophylactically, to prevent secondary infections. Nerves damaged by tetanospasmin need time to repair. Clostridium tetani is resistant to most antibiotics; treatment thus involves several weeks of combined antibiotic therapy.

Antibiotics will prevent the production of additional tetanospasmin but will not repair nerves already damaged by the toxin. Correct

Which is not a component of innate immunity?

Antibody

A deep ground water source:

Aquifer

Skin and mucous membranes

Are the first line of innate immunity, act as physical barriers to infection AND contain antimicrobial secretions.

Human intestinal tract

Ascaris helminths (worms)

Which of the following helminthic diseases is a common infestation found in the southeastern United States? A) Enterobius vermicularis B) Ascaris lumbricoides C) Taenia saginata D) Trichuris trichiura E) Echinococcus granulosus

Ascaris lumbricoides

Aspergillosis respiratory infection

Aspergillus fungi

Fungal spores are a major cause of

Asthma

A cell infected by viruses may die due to the actions of interferons. The same result would occur WITHOUT interferon—any cell infected by a virus would diedirectly fromthevirus.Isthere any apparent benefit to the host organism from the interferon action? A) No; interferon is just an evolutionary leftover from a much earlier form of antiviral activity. It has no function now. Virus infections are always treated with medications. B) Yes; when the interferon acts on a virally infected cell, it shuts down virus replication. Without interferon, the virus will eventually kill the cell, but only after it has replicated many times. IFNs may kill the host cell, but they will also prevent it from being used to replicate virus. C) Yes; by killing host cells, the number of cells that are available targets for viral infection is limited. This is a good way of preventing viral infection. D) No; viruses will replicate in cells regardless of the effects of interferons, so their action of killing the cell has no benefit to the host organism during the infection process. E) Yes; IFN is needed to activate complement which stimulates otherwise phagocytes in the host so that they recognize and destroy not only the virus but also any infected cells. Without complement, all phagocytes are inert.

B

Given the name "leukocyte adhesion deficiency," which of the following steps in the inflammation process given below do you think is/are not carried out in a person with this condition? 1. Tight junctions between endothelial cells are disrupted, allowing fluid to leak from the vessels into the tissue. 2. The phagocytes bind to the endothelial cells and exit the blood vessel by a process called diapedesis. 3. Once in the tissues, phagocytic cells engulf and destroy any microbial invaders. 4. The increase of fluids in the tissues causes the swelling and pain associated with inflammation. 5. The diameter of local blood vessels increases due to the action of inflammatory mediators. A) 1 B) 2 C) 3 D) 1 and 2 E) 2, 3, and 5

B

Epstein-Barr virus may become latent in red blood cells. T cells. nerve cells. B cells.

B cells. p.678 - infects mouth/throat (pharyngitis), carried to lymph nodes and infects B cells (mononuclear cells) - infection can be Productive (virus replicates and kills the B cell) or Non-Productive (latent infection as provirus or plasmid).

Which is not a component of innate immunity? A) Skin B) Antibody C) Inflammation D) Fever E) Mucus membranes

B) Antibody - it's a part of adaptive immunity

Which statement about skin and mucous membranes is NOT correct? A) They are the first line of innate immunity. B) They are the first line of adaptive immunity. C) They act as physical barriers to infection. D) They contain antimicrobial secretions. E) They separate us from our environment.

B) They are the first line of adaptive immunity.

The complement pathway that requires antibodies to be activated is the: A) alternate pathway. B) classical pathway. C) properdin pathway. D) inflammatory pathway. E) lectin pathway.

B) classical pathway

An outbreak of viral gastroenteritis occurs in a pediatrics ward. Rotavirus is the most likely causative agent. T/F

True

The cyclic bout of fever and chills in malaria are caused by: A.liver cell lysisB.white blood cell lysisC.red blood cell lysisD.neurological involvementE.None of the choices is correct

C

What is the difference between 'bacteremia' and 'septicemia?' A. Bacteremia is an infection with bacteria. Septicemia is an infection with Septic protozoans. B. Bacteremia is the presence of living, multiplying bacteria in the bloodstream. Septicemia is the presence of endotoxins, but not necessarily of living microbial agents. C. Septicemia is the presence of living, multiplying bacteria in the bloodstream. Bacteremia is the presence of endotoxins, but not necessarily of living microbial agents. D. There is no difference-both terms denote the presence of living bacterial cells in the bloodstream.

Bacteremia is the presence of living, multiplying bacteria in the bloodstream. Septicemia is the presence of endotoxins, but not necessarily of living microbial agents

Next is Genital System Diseases

Bacterial Vaginosis (BV) - bacterial Vulvovaginal Candidiasis (VVC) fungal Staphylococcal Toxic Shock Syndrome (bacterial - toxin)

dysentary

Balantidium protozoa (ciliates)

The leukocyte that contains histamine is the

Basophil

An early contributor to the understanding of nitrogen fixation was Hooke. Pasteur. Winogradsky. Beijerinck

Beijerinck

The reference for taxonomic descriptions of bacteria is

Bergey's Manual of Systematic Bacteriology

Yersinia pestis is transmitted by fleas. The organism forms biofilms in the digestive tract of infected fleas, often blocking the tract. This prevents the flea from feeding properly, causing bacteria to be regurgitated into the bite wound that the flea has made. In this situation, the flea is a

Biological Vector

In humans, the stem cells from which all blood cells arise are found in the

Bone marrow

whooping cough

Bordetella gram - bacteria

Clostridium botulinum

Botulism, eat it gram + positive rod heat resistant anaerobic likes low acid and low salt endospores soil is reservoir, commonly ingested by humans causes *muscle paralysis* by interrupting acetylcholine action weakness, eyelid droop, speech/breathing/swallowing difficulty improper canning treatment is *antitoxin* and antibiotics

Brucellosis

Brucella gram - bacteria

Interferons, complement, lysozyme, and lactoferrin are all examples of: A) specific antimicrobial factors. B) immune enzymes. C) nonspecific antimicrobial factors. D) cytokines. E) antipeptides.

C) nonspecific antimicrobial factors.

Toll-like receptors (TLRs) bind molecules on pathogens. Why is this helpful to the immune response? a) TLRs are capable of directly lysing (destroying) the microbes, helping our immune responses by eliminating pathogens. b) It provides a highly specific response to very small and highly unique areas on an individual pathogenic microbe, providing the most specific and selective response possible. c) It provides a general response to broad categories of molecules/cells that should NOT be in our system, as we don't have these molecules on our own cells. d) These secreted molecules help bind pathogens and then direct them to receptors on the immune system cells that can eliminate them from our systems. TLRs are delivery mechanisms for the immune responses.

C. It provides a general response to broad categories of molecules/cells that should NOT be in our system, as we don't have these molecules on our own cells.

HIV typically attaches to: protein A. C3b. C5a. CD4.

CD4 CD4 cells - T helper cells, macrophages, dendritic cells - means they can be targeted by HIV.

Reticulate and elementary bodies are two forms of

CHLAMYDIA

When the center of a tubercle breaks down into a ________________, it gradually heals by calcification that forms a tuberculous cavity. A. Caseous necrosis B. Granuloma C. Primary lesion D. Tertiary E. Miliary tubercle

Caseous necrosis

Trachoma urethritis

Chlamydia gram - bacteria

The attraction of leukocytes to the area on inflammation is referred to as

Chemotaxis

The animal(s) frequently associated with Campylobacter jejuni is/are: A. turtles. B. iguanas. C. chickens. D. ducks. E. All of the choices are correct.

Chickens

All of the following are gram-negative rods that cause gastroenteritis EXCEPT: A) Clostridium perfringens. B) Escherichia coli. C) Salmonella typhi. D) Shigella spp. E) Yersinia enterocolitica

Clostridium perfringens. Clostridium perfringens is a Gram-positive, rod-shaped, anaerobic, spore-forming pathogenic bacterium of the genus Clostridium.

Which of the following may aid Staphylococcus in resisting phagocytosis? A. leukocidin B. coagulase C. granulation enzyme D. hemolysin

Coagulase

Diphtheria

Corynebacterium gram + bacteria

Corynebacterium diphtheriae

Corynobacteriaceae; Diphtheria; Gram-positive pleiomorphic

Cryptosporidiosis ("crypto") diarrhea

Cryptosporidium Protozoa (apicomplexa)

The "voices" of a cell, which carry messages, are: A) surface receptors. B) cytokines. C) platelets. D) antigens. E) adhesion molecules.

Cytokines - chemical messengers "any of a number of substances, such as interferon, interleukin, and growth factors, which are secreted by certain cells of the immune system and have an effect on other cells."

The "voices" of a cell are

Cytokines.

During apoptosis, a cell will: A) die because it is damaged and, as a result, cause an inflammatory response. B) self-destruct and, as a result, cause an inflammatory response. C) die because it is accidentally damaged, and, as a result, cause fever. D) self-destruct without causing an inflammatory response. E) divide to repair damaged tissue, leading to an inflammatory response.

D

Which activity of the virally invaded cell triggers production of interferon? A) Activation of rRNA B) Movement of nuclear proteins to the cytoplasm C) Production of glycolipids D) Production of dsRNA E) Production of dsDNA

D) Production of dsRNA

Bacteria are the most numerous soil inhabitants, but the biomass of fungi is greater. T/F

True

Advanced treatment of sewage A. is done to prevent nutrient enrichment. B. is done to prevent possible overproduction of algae and other organisms. C. involves the removal of phosphates and nitrogen compounds. D. All of the choices are correct.

D. All of the choices are correct. p.789 - advanced treatment is often designed to REMOVE AMMONIA, NITRATES AND PHOSPHATES - compounds that foster growth of algae and cyanobacteria in receiving waters (so remove so to PREVENT POSSIBLE OVERPRODUCTION OF ALGAE AND OTHER ORGANISMS). The concentration of these nutrients is normally low in receiving waters which limits the growth of photosynthetic organisms but add the nutrients and see them multiply, BOD increase and other forms of aquatic life threatened (so advanced treatment IS DONE TO PREVENT NUTRIENT ENRICHMENT of the receiving waters so that doesn't happen).

All blood cells originate from the: A) erythrocyte stem cell. B) leukocytic stem cell. C) eosinophilic stem cell. D) hematopoietic stem cell. E) osteocytes (bone cells).

D. hematopoietic stem cells - in the bone marrow

All of the following pertain to poliomyelitis except: A.Transmitted primarily by fecal contaminated waterB.Can be asymptomatic or mild with headache, sore throat, fever and nauseaC.Caused by Poliovirus (genus Enterovirus)D.If virus enters the central nervous system motor, neurons can be infected and destroyed E.Summer outbreaks in the U.S. have been recently increasing

E

Iron: A) is required by microorganisms. B) binds to lactoferrin. C) is necessary for the functioning of some enzymes. D) binds to transferrin. E) All of the answer choices are correct.

E - iron is one of the major components bacteria need for growth. Bind up the iron and stall the growth of invading microbes.

Which of the following is least likely to be detected by pattern recognition receptors? A) O antigen on Gram-negative bacteria B) Peptidoglycan in Gram-positive cells C) Flagellin and pilin D) Double-stranded viral RNA E) Enzymes involved in glycolysis

E) Enzymes involved in glycolysis

C3a and C5a are involved in: A) inflammation AND fever. B) interferon production AND attraction of phagocytes. C) properdin activation AND inflammation. D) attraction of phagocytes AND properdin activation. E) inflammation AND attraction of phagocytes.

E) inflammation AND attraction of phagocytes.

Toll like receptors..

Each recognize a specific "danger" molecule AND transmit a message to the cell's nucleus.

Ehrlichiosis

Ehrlichia gram - bacteria

Which of the following structures would be the best choice as a biological indicator to test autoclave operations?

Endopores formed by Geobacillus (Bacillus) stearothermophilus

amebic dysentery

Entamoeba Protozoa (amebae)

Neutrophils are the second phagocytic cell to respond to an infection.

False

T/F Fungus grows well on fruits and many vegetables due to their alkaline pH.

False

T/F Protozoans are eukaryotes and as such have a nucleus, mitochondria, and chloroplasts.

False

Methanogens often grow in association with

FERMATIVE BACTERIA

Each axial filament is made up of fibrils identical in structure to

FLAGELLA

Swarmer cells are

FORMED BY SHEATHED BACTERIA

Azotobacter

FORMS CYSTS

Approximately 30 percent of the body's immune system is located in the intestinal tract. T/F

False

Cyclosporiosis is transmissable from person to person. T/F

False

Egg white is rich in the antimicrobial lysosome. T/F

False

Fever often enhances bacterial survival during an infection.

False

Gram-positive organisms are more likely to cause fatal septicemias than other infectious agents. T/F

False

Interferon directly interacts with and destroys viruses.

False

Lyme disease is spread from human to human by mosquito vectors. T/F

False

Mast cells are only found in the blood.

False

Individuals entering the room of a patient with Legionnaires' disease should wear a mask to prevent acquiring the disease from exposure to the patient. T/F

False Ch 21 After Legionella pneumophila grows and multiplies in a building water system, water containing Legionella then has to spread in droplets small enough for people to breathe in. People can get Legionnaires' disease or Pontiac fever when they breathe in small droplets of water in the air that contain the bacteria. The bacteria don't spread from person to person. Most people exposed to the bacteria don't get sick.

Candle jars work well for cultivating anaerobic bacteria. T/F

False For capnophiles.

Dental caries can also be referred to as periodontal disease. T/F

False NOT the same thing.

T/F Algae have a vascular system very similar to that found in plants.

False.

Device used for growing mass fermenting cultures.

Fermenter

One of the strongest indications of infectious disease is

Fever

Chlamydia trachomatis

Gram negative pleomorphic rod 1 in 2 get it intracellular *elementary stage* metaboliically inactive; infectious *reticulate stage* actively dividing; non-infectious mostly asymptomatic in women signs/symptoms: lymphogranuloma, painful sex or urination sexual transmission or birth (ocular trachoma in babies) treatment: antibiotics untreated: infertility

Carbon dioxide, sulfur dioxide, methane, and nitrous oxide that could disrupt the temperature balance of the earth.

Greenhouse gas

Which organisms may cause red tide?

Gymnodinium.

Which of the following hepatitis viruses are transmitted by the fecal-oral route? HAV and HBV HAV and HCV HAV and HEV HAV, HBV, and HCV All hepatitis viruses

HAV and HEV Chart on p.651 Correct

Which of the following antigens are useful markers for hepatitis B infection?

HBsAg

What underlying infection should a physician immediately suspect when confronted with a patient with confirmedcryptococcal meningoencephalitis caused by Cryptococcus neoformans, and why? A. Influenza infection. The 2 viruses are often transmitted together in respiratory droplets. B. HIV infection. People with normal immune systems generally fend off this fungal infection before it becomes clinically symptomatic. C. Varicella zoster virus (chicken pox). During the incubation period for chicken pox, and during its latency, C. neoformans has an increased ability to colonize the respiratory tract. D. Syphilis. This sexually-transmitted illness depletes CD4+ helper T cells that are necessary to directly attack the fungal infection of C. neoformans.

HIV infection. People with normal immune systems generally fend off this fungal infection before it becomes clinically symptomatic.

Which of the following is a small Gram-negative rod requiring X-factor for growth? Escherichia coli. Treponema pallidum. Neisseria gonorrheae. Haemophilus ducreyi.

Haemophilus ducreyi Chancroid, bacterial STI = Haemophilus ducreyi - tastidious, pleomorphic, Gram-negative coccobacillus (short rod) that can be cultivated only on a rich medium such as chocolate agar.

Peptic ulcers stomach cancer

Helicobacter gram - bacteria

A breath test assaying for radioactive carbon dioxide may be used to indicate the presence of

Helicobacter pylori

Histoplasmosis respiratory infection

Histoplasma fungi

Slowly decaying organic litter from plant and animal tissues found in the soil

Humus

Select the TRUE statement regarding clostridial myonecrosis. Prevention of clostridial myonecrosis can be achieved with the DCTaP vaccine. Hyperbaric chambers may be used to treat patients with gangrene—high levels of O2 reduce anaerobic conditions in the tissues. Clostridium perfringens infects healthy tissue and grows easily in well oxygenated tissues. The causative agent produces α-toxin, an A-B toxin that destroys lecithin in host cell membranes, leading to cell lysis. C. perfringens is an endospore-forming organism, and produces large number of spores in wounds or cultures.

Hyperbaric chambers may be used to treat patients with gangrene—high levels of O2 reduce anaerobic conditions in the tissues.

How do fleas cause disease?

If ingested, they may transmit a tapeworm to children and may transmit Yersinia pestis.

Why does the rash of shingles occur in only a limited region, as opposed to the widespread rash of chickenpox? A) In chickenpox, the replicating virus originates from multiple infected nerve cells, while in shingles it is limited to the bloodstream. B) In shingles, the replicating virus originates from a single infected nerve cell, while in chickenpox it is spread throughout the bloodstream. C) In chickenpox, the virus spreads through the bloodstream to all parts of the body, while in shingles, the virus remains in the chest area after inhalation. D) The chickenpox virus is a double-stranded DNA virus that is able to infect any nucleated cells, while the shingles virus is a single-stranded RNA virus that only infects epidermal cells. E) This question is misleading because the rash of both chickenpox and shingles is widespread throughout the body.

In shingles, the replicating virus originates from a single infected nerve cell, while in chickenpox it is spread throughout the bloodstream. - virus replicates in nucleus of nerve cell and moves to skin, spreading locally. originates from single infected nerve cell rather than from bloodstream.

Why do Neisseria meningitidis, Streptococcus pneumoniae, and Haemophilus influenzae seldom cause meningitis in newborns? These pathogens are unable to colonize infants. Infants seldom come into contact with these pathogens. Infants are vaccinated against these pathogens. Infants are protected by maternal IgM against these pathogens. Infants are protected by maternal IgG against these pathogens.

Infants are protected by maternal IgG against these pathogens. p.701 - the common causes of meningitis in children (Neisseria meningitidis, Streptococcus pneumoniae, and Haemophilus influenzae) seldom cause meningitis in newborns because most mothers have antibodies (IgG) against them. These antibodies can cross the placenta and protect the baby until it is about 6 months old.

Endocarditis - Inflammation of the heart valves or lining of the heart chambers. Fungemia - Fungi circulating in the bloodstream. Gametocyte - Haploid cell that can be used for sexual reproduction. HIV (viral) set point - Viral load in a person with HIV disease after the immune system begins to respond to the virus and viral numbers stabilize. Human immunodeficiency virus, HIV - The virus causing a disease marked by gradual impairment of the immune system, culminating in AIDS. Reverse transcriptase - Enzyme that synthesizes double-stranded DNA complementary to an RNA template. Seroconversion - Change from serum that lacks specific antibodies to serum that has those antibodies. Subacute bacterial endocarditis - Slowly progressing infection of the internal surfaces of the heart. Transformation - Morphologic and physiologic changes making a cell cancerous resulting from infection of an animal cell by an oncogenic virus. Trophozoite - Vegetative feeding form of some protozoa. Disseminated intravascular coagulation, DIC - Devastating condition in which clots form in small blood vessels, leading to failure of vital organs.

Integrase - Enzyme encoded by HIV or another retrovirus that inserts the DNA copy of the viral genome into a host cell chromosome. Kaposi's sarcoma - Tumor arising from blood or lymphatic vessels. Paroxysm - A sudden increase in symptoms. Petechiae - Small purplish spot on the skin or mucous membrane caused by hemorrhage. Preventive vaccine - Vaccine that prevents a disease. Acquired immune deficiency syndrome, AIDS - A disease of the human immune system caused by infection with human immunodeficiency virus (HIV); the most advanced stage of infection with HIV. Acute retroviral syndrome, ARS - A stage of HIV disease following the incubation stage; often includes flulike symptoms. Bacteremia - Bacterial cells circulating in the bloodstream. Bubonic plague - Form of plague that typically develops when it is injected during the bite of an infected flea.

Choose the one correct statement about leptospirosis. a) Humans are the only reservoir. b) Most infections produce severe symptoms. c) Transmission is by the fecal-oral route. d) It can lead to unnecessary abdominal surgery. e) An effective vaccine is generally available for preventing human disease.

It can lead to unnecessary abdominal surgery. a) It is a zoonosis - animal resorvoirs b) leptospirosis infections are often asymptomatic c) contaminated urine is the main mode of transmission - humans contract leptospirosis from water, soil or food contaminated with infected animal urine. Person to person transmission does not seem to occur. e) mild cases resolve without treatment, though antibiotics can be given. Available vaccines are for animal use or for preventing disease in high risk individuals like farmers and vets.

How does coagulase help Staphylococcus aureus evade phagocytes? A) It coats the surface of the bacterial cells with collagen, a tissue binding protein. B) It coats the surface of bacterial cells with fibrin, a protein found in blood. C) It causes formation of small clots in capillaries, slowing progress of phagocytes to the infected area. D) It causes formation of small clots in capillaries, slowing progress of phagocytes to the infected area AND it coats the surface of bacterial cells with fibrin, a protein found in blood. E) It causes formation of small clots in capillaries, slowing progress of phagocytes to the infected area AND it coats the surface of the bacterial cells with collagen, a tissue binding protein.

It causes formation of small clots in capillaries, slowing progress of phagocytes to the infected area AND it coats the surface of bacterial cells with fibrin, a protein found in blood. Table 22.3 - coagulase - may slow progress of leukocytes into affected area by producing clots in the surrounding capillaries.

Coccidiumycosis is what type of disease and is caused by which organism?

It is a fungal disease and is caused by Coccidiodes sp.

Which of the statements regarding Naegleria fowleri is FALSE?

It is a small, Gram-negative, flagellated diplococcus.

Define mycelium.

It is a tangle of fungal hyphae.

Why do water-testing procedures look for coliforms rather than pathogens? - It is always somewhat dangerous to culture pathogens in a laboratory setting - people in the lab are at risk of contracting serious disease. It is much safer to test for coliforms which simply indicate that humans have been swimming in the water. - Pathogens are seldom found in water. Testing for coliforms is a way of determining whether there is any human normal microbiota contamination in that water, which should then be treated with chlorine. - Pathogens and coliforms are the same thing. Testing for one is also testing for the other. - It is not possible to test for all of the potential pathogens in water, so indicator organisms such as coliforms are used. These microbes are found in feces, so finding them suggests fecal contamination, and indicates a greater likelihood that intestinal pathogens are also present. - Pathogens are extremely fastidious in their nutrient requirements and require special growth conditions. Coliforms are easier to culture in a laboratory setting, so it makes sense to test for these rather than for actual pathogens.

It is not possible to test for all of the potential pathogens in water, so indicator organisms such as coliforms are used. These microbes are found in feces, so finding them suggests fecal contamination, and indicates a greater likelihood that intestinal pathogens are also present.

Why is Legionella pneumophila detected using immunofluorescence? A) It stains poorly with conventional dyes. B) It is Gram-non-reactive. C) It is an acid-fast bacterium. D) It lacks a peptidoglycan cell wall. E) It is a virus.

It stains poorly with conventional dyes. This is from Ch 21.

food poisoning

Listeria gram + bacteria

Which of the following microbes is (are) important to cheese-making?

LACTIC ACID BACTERIA & PROPINONIBACTERIUM

Which of the following colonize the vagina during childbearing years?

LACTOBACTERIA

Mycobacterium leprae

Leprosy is caused by ________ _____

Please select the statement that does NOT apply to strategies of bioremediation. Bioaugmentation involves adding microorganisms to contaminated material, complementing the resident population. Providing optimal bacterial growth conditions is likely to promote pollutant degradation. Biostimulation enhances pollutant degradation by enhancing growth of resident microbes in a contaminated site through providing additional nutrients. Low temperature, arid conditions and acidic pH will speed up the rate of pollutant degradation. Co-metabolism occurs when an enzyme produced by a microbe to degrade an added substrate in an environment degrades a pollutant as well.

Low temperature, arid conditions and acidic pH will speed up the rate of pollutant degradation. - it's an increase in temperature, near neutral pH and optimal moisture that will speed up the rate of pollutant degradation.

Plasmodium causes

Malaria

How would increased travel lead to increased spread of multicellular eukaryotic parasites?

Many eukaryotic parasites depend on vectors (often small blood-sucking insects) for transmission, so moving an infected individual into an area with new vectors and new susceptible humans would increase spread.

Phagocytes were first discovered and named by

Metchnikoff.

The complex resulting from complement activity that leads to cell lysis is the

Membrane attack complex.

Which of the following are referred to as mononuclear phagocytes?

Monocytes and macrophages

Mycobacterium tuberculosis

Mycobacteriaceae; tuberculosis; acid-fast bacteria with Gram-positive cells structure streptobacillus

Tuber leprosy

Mycobacterium gram + bacteria

Which of the following organisms is the causative agent of Hansen's disease (leprosy)?

Mycobacterium leprae

The Gram -positive rod that is also acid fast and is human pathogen is

Mycobacterium tuberculosis.

The Gram-positive rod that is also acid-fast and is a human pathogen is

Mycobacterium tuberculosis.

Which organism is/are acid fast?

Mycobacterium tuberculosis.

Fungal diseases are generally referred to as

Mycoses

Gonorrhea

Nieisseria gram - bacteria

The conversion of ammonia (NH4+) to nitrate (NO3-) occurs during A. Ammonification B. Photosynthesis C. Nitrogen fixation D. Nitrification E. Denitrification

Nitrification

Are all fungi detrimental (bad) for other organisms?

No-fungi are sometimes good, sometimes bad for other organisms. It really depends on which fungus you're talking about and the relationship it has with the other organism. Some fungi, for example, can form a symbiotic relationship with plant roots that increases their nutrient and water absorption. This is good.

Are all fungi detrimental (bad) for other organisms?

No—it depends on the fungus and the relationship it has with the other organism. Some fungi can form a symbiotic relationship with plant roots that increases their nutrient and water absorption. This is good.

Organisms that grow very slowly, are non-curable, are present in very small numbers, or are mixed with a number of other bacteria may still be identified using

PCR

Would you expect an individual with giardiasis who has diarrhea to be more likely to transmit the disease than an individual with giardiasis who does NOT have diarrhea? Why or why not? A. No. This illness is spread by respiratory droplets, so diarrhea as a symptom shouldn't matter for transmission of the disease.B. No. This illness is spread by sexual contact, so diarrhea as a symptom shouldn't matter for transmission of the disease. C. Yes. This illness is spread by the fecal-oral route, so presence of diarrhea as a symptom should dramatically increase the possibility of transmission of infection. D. Yes. This illness is spread by insects that feed on contaminated fecal matter, becoming infected themselves. The disease is spread to new individuals when these infected insects bite a susceptible person, transmitting the cysts of the protozoan. As such, diarrhea as a symptom would increase the risk of transmission through biting insects to new individuals. E. No. This illness is spread when people ingest cysts, and a person with severe diarrhea excretes primarily trophozoites; an asymptomatic person is more likely to excrete cysts and is therefore more infectious.

No. This illness is spread when people ingest cysts, and a person with severe diarrhea excretes primarily trophozoites; an asymptomatic person is more likely to excrete cysts and is therefore more infectious.

Which one of the following is NOT a zoonosis? cat-scratch disease Hantavirus pulmonary syndrome rabies tapeworm None of the answers is correct; all of these are zoonoses

None of the answers is correct; all of these are zoonoses

Interferons, complement, lysozyme, and lactoferrin are all examples of

Nonspecific antimicrobial factors.

Which strain of Escherichia coli may be involved in foodborne infection? HB101 O157:H7 M309 AB220 O7:H157

O157:H7

Which dinoflagellate kills fish by dispersing toxins directly into the water?

Pfiesteria

The cellular organelle responsible for the digestion of ingested infectious agents is the

Phagolysosome.

Malaria

Plasmodium Protozoa (apicomplexa)

Interleukins are..

Produced by leukocytes AND are protein molecules.

Which is not a characteristic of Helicobacter pylori? A. Gram negative B. Produces urease that buffers stomach acidity C. Produces entertoxin that causes diarrhea D. Lives in the stomach E. Curved rods

Produces entertoxin that causes diarrhea Helicobacter pylori is a short, curved, gram-negative, microaerophilic bacterium with multiple flagella that are unusual because they are covered by sheaths.

Which activity of the virally invaded cell triggers production of interferon?-

Production of dsRNA

Which of the signs and symptoms of Rocky Mountain spotted fever is not typical of other diseases that manifest in a similar way? A) Fever B) Rash C) Headache D) Joint pain E)Muscle pain

Rash RMSF rash starts on palms, soles and moves up extremities to the trunk. Opposite more other rashes.

Which of the following happens when food is placed in the refrigerator? A. Refrigeration eliminates the most resistant bacterial spores, especially those of the genera Bacillus and Clostridium B. Refrigeration eliminates pathogenic bacteria from milk; the process also lowers the total number of bacteria and thereby reduces the change of spoilage C. Refrigeration results in low oxygen levels which prevent the growth of facultative aerobes, limiting them to fermentative ATP production D. Refrigeration destroys many enzymes in the food product and prevents any further cellular metabolism from taking place E. Refrigeration reduces the rate of enzyme activity in a microorganism and thus lowers the rate of growth and reproduction

Refrigeration reduces the rate of enzyme activity in a microorganism and thus lowers the rate of growth and reproduction

Rickettsia rickettsii

Rocky Mountain Spotted Fever gram negative coccobacillus obligate intracellular parasite transmission: blood sucking arthropod as vector virulence: stimulates endocytosis to get into cytoplasm signs/symptoms: rash, "black measles", petechiae- subcutaneous hemorrhage severe- respiratory, CNS, GI, renal failure treatment: antibiotics prevention: check for ticks, wear long clothes

Which of the following statements regarding S. aureus and S. pyogenes is FALSE? A) S. aureus appears as clusters of spherical cells while S. pyogenes appears as chains of spherical cells. B) S. aureus stains purple with the Gram stain while S. pyogenes stains pink with this procedure. C) S. aureus is catalase positive and coagulase positive while S. pyogenes is catalase negative and coagulase negative. D) The Fc receptor of S. aureus is protein A while the Fc receptor of S. pyogenes is protein G. E) S. aureus is a facultative anaerobe while S. pyogenes is an obligate fermenter.

S. aureus stains purple with the Gram stain while S. pyogenes stains pink with this procedure. Both Staphylococus aureus and Streptococcus pyogenes are Gram positive so stain PURPLE.

In addition to S. aureus, impetigo may also involve: A) M. luteus. B) S. pyogenes. C) S. epidermidis. D) Pseudomonas species. E) Cutibacterium species

S. pyogenes. Causes pus-tonsiled strep throat too. Pyogenes - generation of pus.

Sphaerotilus and Leptothrix are examples of

SHEATHED BACTERIA

Neisseria gonorrhoeae

STI- "the clap" Gram - negative bacteria diplococci obligate parasite target population is 18-24 year olds IgA protease breaks down IgA antibodies antibiotic resistant human pathogen non-motile can get in the eyes of newborns OMP for attachment and getting into host cell

Thermus and Deinococcus

SURVIVE IN EXTREME CONDITIONS

foodborne illness typhoid fever

Salmonella gram - bacteria

Why should scarring of a fallopian tube raise the risks of an ectopic pregnancy? a. Scarring may slow the progression of the ovum (egg) from the fallopian tubes into the uterus. If it is fertilized by a sperm cell while it is stopped in the fallopian tube, it may implant there, resulting in an ectopic pregnancy. b. Scarring can completely block the ovum (egg) from being able to exit the fallopian tube to enter the uterus. It may implant in the wall of the fallopian tube (since it can't enter the uterus) and result in an ectopic pregnancy. c. Scarring may allow the sperm cells to enter the fallopian tubes and bind to the scar tissue. The inflammatory response generated when this occurs is termed an ectopic pregnancy. d. Scarring of the fallopian tubes results from undiagnosed bacterial infections. These bacteria can also make the lining of the uterus inhospitable to implantation of a fertilized egg. If the fertilized egg can't implant into the wall of the uterus, it may travel back into the fallopian tube and implant there instead, resulting in an ectopic pregnancy.

Scarring may slow the progression of the ovum (egg) from the fallopian tubes into the uterus. If it is fertilized by a sperm cell while it is stopped in the fallopian tube, it may implant there, resulting in an ectopic pregnancy. Slow the progression. Complete blockage of fallopian tube means no sperm gets in to fertilize anything. Correct

Fungi that are important for fermentation of fruits

Secrete degradative enzymes

Streptococcus agalactiae (Group B strep)

Streptococcaceae; neonatal sepsis, postpartum infection; Gram-positive cocci

Which statement regarding the treatment of tetanus is FALSE? TIG neutralizes tetanospasmin that is already attached to nerve tissue, preventing damage to that nerve. The person is given tetanus vaccine that results in the production of anti-tetanospasmin antibodies. TIG antibodies bind to circulating toxin molecules, neutralizing their effects and providing passive immunity. The patient is given muscle relaxants and supportive care, including being placed on a ventilator if needed. An antibacterial medication such as metronidazole is given to kill any actively multiplying bacterial cells.

TIG neutralizes tetanospasmin that is already attached to nerve tissue, preventing damage to that nerve. Hint - it doesn't. Correct

Pork/human tapeworm

Taenia solium helminths (worms)

Which is not involved in adaptive immunity?

Tear flow

What type of vaccine protects against mumps virus?

The MMR/MMRV is:

What is the difference between a trophozite and a cyst?

The ability to exist as either a trophozoite or a cyst is a characteristic of many protozoa.

Why is staphylococcal scalded skin syndrome sometimes fatal? A) The first line of defense is compromised, putting the patient at risk secondary bacterial infection. B) The first line of defense is compromised, putting the patient at risk of both dehydration and secondary bacterial infection. C) The causative organism is Gram-negative and may cause endotoxic shock. D) Exfoliatin can bind to cardiac cells, causing disruption of the heart rhythm and heart failure. E) The first line of defense is compromised, putting the patient at risk of severe dehydration.

The first line of defense is compromised, putting the patient at risk of both dehydration and secondary bacterial infection.

Why are such a high percentage of infant botulism cases associated with ingestion of honey? A. Bees cultivate C. botulinum bacteria within their hives as a food source, so it gets mixed in with the honey. B. The infant digestive tract is far more susceptible to the effects of botulism toxin (found in the honey in relatively high levels) than the adult digestive tract. C. The spores of C. botulinum produce a strong endotoxin. This endotoxin has little effect on adults due to their overall size. Infants, however, are significantly smaller (and therefore far more susceptible) than adults. D. There can be endospores from C. botulinum in honey. They may germinate in the intestines of infants following consumption of honey, leading to colonization and pathogenesis as the bacteria begin forming botulism toxin. Adults' normal intestinal microbiota will out-compete the new microbes, but very young infants may not yet have this normal flora established.

There can be endospores from C. botulinum in honey. They may germinate in the intestines of infants following consumption of honey, leading to colonization and pathogenesis as the bacteria begin forming botulism toxin. Adults' normal intestinal microbiota will out-compete the new microbes, but very young infants may not yet have this normal flora established.

Which of the following statements regarding tapeworms is FALSE?

They complete their life cycle in a single host.

Borrelia burgdorferi is a spirochete with a number of axial filaments. T/F

True

Which parasitic organism, when it crosses the placenta, results in intracerebral calcifications, chorioretinitis, microcephaly, hydrocephaly, and convulsions?

Toxoplasma gondii - is an obligate intracellular parasitic one-celled eukaryote (specifically an apicomplexan) that causes the infectious disease toxoplasmosis.

A mycorrhiza is a symbiotic association between roots and fungi. T/F

True

A wide variety of fungi spoil food including species of Rhizopus, Alternaria, Penicillium, Aspergillus, Botrytis. T/F

True

Which statement regarding CagA and VacA of H. pylori is INCORRECT? VacA is an A-B toxin that increases urea flow into the stomach. CagA interferes with cell signaling and promotes inflammation. VacA interferes with the function of T cells. CagA is delivered into host cells by a secretion system. Strains of H. pylori that produce CagA are associated with stomach cancer.

VacA is an A-B toxin that increases urea flow into the stomach. It's a membrane-damaging toxin

Epidemics of bacterial infections of the digestive system are transmitted by: A) Food. B) Water. C) Milk. D) The respiratory route. E) All of the above

Water

Cholera & gastroenteritis

Vibrio gram - bacteria

Which of the following media would you use to cultivate Vibrio cholerae?

Vibrio cholerae: curved, Gram-negative rod Halotolerant, can grow in alkaline conditions - can grow on most media with additional salt Google says: Thiosulfate citrate bile salts sucrose agar

Soft shell clams that live buried in the mud on tidal mudflats: Staphylococcus aureus Vibrio parahaemolyticus E. coli 0157:H7 Listeria monocytogenes Campylobacter

Vibrio parahaemolyticus

Vibrio cholerae

Vibrionaceae; epidemic cholera/diarrhea; Gram-negative vibrio

In rare cases, people with leptospirosis develop ______________ disease, which affects the liver and kidneys.

Weil's - jaundice, liver and kidney failure, hemorrhage in many organs and meningitis.

What is the purpose of coagulation in drinking water treatment? Pollutants such as lead are combined into clumps using toxic chemicals, for easy removal. To bind together pollutants, bacteria, and other organisms for removal by chemicals. Suspended material is combined using toxic chemicals for easy removal from water. When suspended material clumps, it sinks and can be easily removed. Antibodies are used bind bacteria into large masses that can be removed by filtration.

When suspended material clumps, it sinks and can be easily removed.

Your sister recently had her first child, a boy who she names Owen. Unfortunately, Owen seems to get sick really often, and after some investigation, he is diagnosed with an extremely rare genetic disorder called leukocyte adhesion deficiency (LAD). People with LAD suffer from repeated bacterial infections because their leukocytes have adhesion defects—they cannot adhere to the endothelium and as a result are unable to leave the blood vessels to move into tissues containing microbial invaders. Without this inflammatory response, the person experiences severe tissue infections and frequently has neutrophilia (high levels of neutrophils in the blood). You help your sister understand this disease, using your knowledge of the immune system. Given what you understand about inflammation, why do you think a person with LAD experiences neutrophilia (high numbers of neutrophils in the blood)? - When a person has an infection, their number of neutrophils increases. In a person with LAD, the neutrophils do not attach to endothelial cells, and thus cannot not leave the blood vessel to enter infected tissue. As a result, they accumulate in the blood. - When a person has an infection, their number of neutrophils always decreases significantly. In a person with LAD, there are so few neutrophils that these cells do not leave the blood vessel to enter infected tissue. As a result, they accumulate in the blood. - The inflammation response in a person with LAD is overactive. They produce an excess of phagocytic cells such as neutrophils and although some of these cells migrate into infected tissue, there are so many of these cells that the rest accumulate in the blood. - In a person with LAD, the neutrophils become "sticky" and adhere strongly to blood vessel walls. At the same time, endothelial cell junctions always tighten during inflammation, so the neutrophils cannot not leave the blood vessel and they accumulate in the blood. - None of these explains why a person with LAD suffers from neutrophilia.

When a person has an infection, their number of neutrophils increases. In a person with LAD, the neutrophils do not attach to endothelial cells, and thus cannot not leave the blood vessel to enter infected tissue. As a result, they accumulate in the blood.

Which one of the following statements about bacterial vaginosis is FALSE? It is the most common vaginal disease in women of childbearing age. Women with BV always experience inflammation of the vagina. Treatment of the male sex partners of patients with BV does not prevent recurrences. The vaginal microbiota shows a decrease in lactobacilli and an increase in anaerobic bacteria. In pregnant women, it is associated with premature delivery.

Women with BV always experience inflammation of the vagina. Not true because p.735 - about half of BV cases are asymptomatic.

The causative agent of plague is Vibrio cholerae. Staphylococcus aureus. Brucella abortus. Yersinia pestis.

Yersinia pestis. Y. pestis is a member of Enterobacteriaceae - is a facultative anaerobe, Gram negative rod that grows best at 28C, is non-motile and forms biofilms in flea guts

A high BOD value means a small number of viruses are present AND a large amount of oxygen has been used. a small amount of oxygen has been used AND a large amount of degradable organic matter is present. a large amount of degradable organic matter is present AND a small number of viruses are present. a large amount of oxygen has been used AND a large amount of degradable organic matter is present. a large amount of oxygen has been used AND a small amount of degradable organic matter is present.

a large amount of oxygen has been used AND a large amount of degradable organic matter is present. p.787 - high BOD values indicate that large amounts of degradable materials are present, resulting in the consumption of correspondingly large amounts of oxygen during biological degradation of the environment.

Toxoplasmosis

a parasite which is most commonly transmitted from animals to humans by contact with contaminated feces

The normal flora of the genital tract of women is: affected by estrogen levels .dependent on the activity of Lactobacillus. unchanging. typically composed of E. coli. affected by estrogen levels AND dependent on the activity of Lactobacillus.

affected by estrogen levels AND dependent on the activity of Lactobacillus. p.730 - The normal microbiota of the genital tract of women is affected by the action of estrgen hormones on the epithelial cells of the vaginal mucosa. - estrogen present -> glycogen deposited in cells - glycogen converted to lactic acid by lactobacilli, resulting in an acidic pH that inhibits the growth of many potential pathogens.. - lactobacilli also release hydrogen peroxide, an inhibitor of some anaerobic bacteria.

Agar is obtained from

algae

Lichens may be an association of

algae and fungus

chapter quiz - 3rd try

all correct

Antimicrobial chemical in garlic

allicin

Leprosy is also known as Hansen's disease. a disease of the blood. an infectious disease caused by a virus. acquired by droplet transmission. always fatal.

also known as Hansen's disease. Caused by Mycobacterium leprae bacteria acquired person to person or through nasal secretions not always fatal, nope disease of the nervous system (likes PNS)

In humans, the stem cells from which all blood cells arise are found in the: A) peripheral circulation. B) bone marrow. C) lymphatic vessels. D) lymph nodes. E) spleen.

bone marrow - hematopoietic stem cells

The conversion of ammonium to nitrate could be accomplished by the presence of

both Nitrosomonas and Nitrobacter

The purple sulfur and green sulfur bacteria

both use hydrogen sulfide as a source of electrons.

The central nervous system is made up of: A. brain and spinal cord. B. spinal cord and spinal nerves. C. sympathetic and parasympathetic system. D. brain and endocrine organs.

brain and spinal cord

Which animal is used as an early warning system for viral encephalitis? chickens hawks armadillos mice foxes

chickens

Which of the following is a bacterial product? A) Lactoferrin B) Transferrin C) Colicin D) Defensin E) Mucus

colicin - A colicin is a type of bacteriocin produced by and toxic to some strains of Escherichia coli. Colicins are released into the environment to reduce competition from other bacterial strains.

The accepted method of testing water supplies for the possible presence of pathogens is to determine the presence of A. Streptococci. B. coliforms. C. Staphylococci. D. Streptomyces.

coliforms

Modern approaches to evolutionary taxonomy often include

comparison of DNA and RNA

A group of interacting serum proteins that provide a nonspecific defense mechanism is: A) complement. B) interferon. C) glycoprotein. D) lysozyme. E) antibody.

complement

A frequent complication of scalded skin syndrome is: A) a bacterial secondary infection. B) secondary viral infections. C) dehydration. D) dehydration and a secondary bacterial infection. E) toxemia.

dehydration and a secondary bacterial infection.

Pediatric Giardia infections have greater consequences than infections in adults primarily as a result of _________________________. infection of associated digestive organs, such as the liver and pancreas dehydration from the frequent diarrhea interruption of dietary fat breakdown invasion of the intestinal mucosa

dehydration from the frequent diarrhea

Most initial cases of dengue fever are very mild or even asymptomatic. When a second infection with a different serotype of the five known dengue viruses occurs, - the person always develops dengue hemorrhagic fever (DHF) or dengue shock syndrome (DSS), which is quickly fatal. - the usual protection from a previous infection is reactivated and quickly eliminates the new virus type from the infected individual. - dengue hemorrhagic fever may develop, due to antibody-dependent enhancement that facilitates viral entry into macrophages. - the two viruses work together to achieve a much more serious infection that is always rapidly fatal.

dengue hemorrhagic fever may develop, due to antibody-dependent enhancement that facilitates viral entry into macrophages.

Organisms that may cause red tide are

dinoflagellates

The most common infectious disease of humans is the common cold. hepatitis A. halitosis. influenza. dental caries.

dental caries

Collections of bacteria that adhere to the surfaces of the teeth are called: dental caries. dental plaque. halitosis. periodontal disease.

dental plaque. Proteinaceous material from saliva adheres, creates thin film (PELLICLE) - Bacteria attach, create biofilm called dental PLAGUE - Mineral salts deposit over time, create dental calculus or TARTAR

Typically pathogenic Neisseria gonorrheae: secretes transferrin. destroys IgA. destroys IgM. is very immunogenic.

destroys IgA. The bacteria produces IgA protease, which destroys secretory IgA and seems to interfere with phagosome maturation. p.743

Movement of spirochetes occurs by means of structures called

endoflagella

Slow cooking for a long time and then storage at room temperature would tend to favor the growth of anaerobes AND endospore-formers. thermophiles AND anaerobes. mesophiles AND thermophiles. endospore-formers AND thermophiles. endospore-formers AND mesophiles.

endospore-formers AND mesophiles. mesophiles like 25-45 degrees Celsius

Bacteria survive adverse conditions by forming zygotes. capsules. endospores. glycocalyx.

endospores

The material released from bacteria that may lead to shock and death in septicemia is exotoxin. protein A. endotoxin. interferon.

endotoxin. = LPS in outer membrane of Gram negative bacteria.

What 2 functions do phagocytes serve in immune responses?

engulfment/destruction of foreign cells AND alerting the other cells of the immune system to an invader.

Which species of Plasmodium causes the most serious form of malaria? ovale malariae vivax falciparum

falciparum p.688 - P. falciparum infections are more sever than other types. Can infect ALL RBCs, not just young or old ones, and it encodes a virulence factor called pfEMP1 that causes RBCs to stick to capillary walls, blocking blood flow (if it happens in the brain -> cerebral malaria. Happens in placenta....).

Any desirable change in microorganisms impart to food

fermentation

Methanogen often grow in association with

fermentative bacteria.

Foods that have been altered due to the carefully controlled growth of microorganisms are called controlled. fermented. cooked. spoiled. preserved.

fermented

Leaf cutter ants farm

fungal gardens

Foods such as acidic fruits are often spoiled by fungi. protozoans. viruses. bacteria

fungi Bacteria can't hack it at pH less than 4.5. Acidic fruit? Fungi is your spoiler.

Which of the following organisms may grow at pH 2.2? Clostridium Streptococcus fungi lactic acid bacteria fungi AND lactic acid bacteria

fungi Even lemons can go moldy. White and green fungi = yum...not

Which of the following organisms may grow at pH 3.5? Streptococcus AND lactic acid bacteria Clostridium AND fungi fungi AND lactic acid bacteria lactic acid bacteria AND Clostridium fungi AND Streptococcus

fungi AND lactic acid bacteria

Mycorrhizas

fungi and plant roots growing together

In which of these organs does a carrier of typhoid bacilli maintain the bacteria? Gallbladder Liver Colon Peyer's patches Liver AND Peyer's patches

gallbladder Correct

Sexual reproduction in algae utilizes meiosis that results in the production of

gametes with half the amount of DNA as in the parental cells.

Nematodes may enter the

gastrointestinal tract or the blood

The toxins involved in intestinal infections typically A. kill cells by inhibiting protein synthesis. B. modify cell physiology resulting in increased secretion of water and electrolytes. C. modify cell physiology resulting in decreased secretion of water and electrolytes. D. kill cells by inhibiting DNA synthesis. E. kill cells by inhibiting protein synthesis AND modify cell physiology resulting in increased secretion of water and electrolytes.

kill cells by inhibiting protein synthesis AND modify cell physiology resulting in increased secretion of water and electrolytes. 2 types - enterotoxins and cytotoxins enterotoxins make cells lose water and electrolytes cytotoxins cause cell death

Vibrio cholerae and most salmonellae are: a. killed by acid conditions. b. stimulated by acid conditions. c. killed by low concentrations of salt. d. killed by neutral conditions.

killed by acid conditions. - neither is very resistant to stomach acid and so need more to be infective. I know Salmonella needs 10^11 cells.

Vibrio cholerae and most salmonellae are stimulated by acid conditions. acidophiles. killed by neutral conditions. killed by acid conditions. killed by low concentrations of salt.

killed by acid conditions. Correct

Haemophilus influenzae

meningitis is caused by __________ ________

Your sister recently had her first child, a boy who she names Owen. Unfortunately, Owen seems to get sick really often, and after some investigation, he is diagnosed with an extremely rare genetic disorder called leukocyte adhesion deficiency (LAD). People with LAD suffer from repeated bacterial infections because their leukocytes have adhesion defects—they cannot adhere to the endothelium and as a result are unable to leave the blood vessels to move into tissues containing microbial invaders. Without this inflammatory response, the person experiences severe tissue infections and frequently has neutrophilia (high levels of neutrophils in the blood). You help your sister understand this disease, using your knowledge of the immune system. Which of the following is NOT a leukocyte? - Neutrophil - B cell - Basophil - Macrophage - These are all leukocytes

they are all leukocytes

Staphylococcus aureus

*furncle* boils *carbuncle* several boils joined together *impetigo* bunch of skin bumps are caused by ____________ _______

Which of the following may be added to normal media to make it more selective for staphylococci? A. 0.5% HCl B. 7.5% salt C. 1.0% glucose D. 5.0% mannose

- 7.5% salt Staphylococci: SALT-TOLERANT, use nutrients and produce antimicrobial substances active against other Gram positive bacteria

The organism that grows anaerobically in dead or damaged tissue and produces dense colonies that are the color and size of sulfur particles is: - Escherichia coli. - Staphylococcus aureus. - Actinomyces israelii. - Pseudomonas aeruginosa.

- Actinomyces israelii. Actinomycosis (Lumpy Jaw) - slide 38 Grows as dense yellowish colonies called "sulfur granules," since color and size of particles of sulfur.

Which of the following is considered a function of skin? A) Regulation of body temperature B) Prevention of fluid loss C) Synthesis of vitamin D D) Production of cytokines E) All of the choices are correct.

- All of the choices are correct. A and B definitely, C I think so and at that point it has to be ALL.

Almost all bacterial intestinal infections may be attributed to: - Vibrio spp. - C. jejuni. - Salmonella spp - .enterobacteria. - All of the choices are correct.

- All the choices are correct I don't like the wording but they all do cause bacterial intestinal infections.

Which of the following organisms is not normally found on the skin? A) Staphylococci B) Diphtheroids C) Candida species D) Malassezia species E) Staphylococci AND diphtheroids

- Candida species Most microbial inhabitants in three groups • DIPHTHEROIDS: oily regions (forehead, upper chest, back) - Cutibacterium (Propionibacteria) most common: obligate anaerobes, grow within hair follicles • STAPHYLOCOCCI: salt-tolerant, use nutrients and produce antimicrobial substances active against other Gram positive bacteria • MALASSEZIA: tiny lipid-dependent yeasts

Which of the following gastrointestinal pathogens produce A-B toxins?

- Cholera toxin is A-B - Shigella toxin is A-B

Which of the following is an important function of the skin? A) Hold muscle to bone B) Manufacture blood cells C) Produce antibodies D) Control body temperature E) All of the answer choices are correct.

- Control body temperature Tough but flexible. - Serves as a blood reservoir, barrier to keep things in/out = primary defense along with mucus membranes. Skin: prevents entry, regulates body temperature, restricts fluid loss, senses the environment

?Which of the following E. coli serovars give rise to a disease similar to that caused by Shigella sp.? Enteroinvasive Enteropathogenic Enterotoxigenic Enterohemorrhagic All of the choices are correct.

- Enteroinvasive E. coli p. 642 - Enteroinvasive E. coli (EIEC) - these strains invade the intestinal epithelium, causing a disease similar to shegellosis. Correct

Only one serotype of mumps virus is known. Why is this important?

- I think: lets us make a vaccine for it

You are studying for your NCLEX exams with a group of friends. Each of you presents a case study to the other students in your group. Your friend Sue gives the following information in her case study: the patient is a child, who presents with a high fever (41oC) and a rash. The parents report that three days prior to the onset of the rash, the child had a runny nose, diarrhea, and conjunctivitis, all of which they thought indicated influenza. The child has not received any of the common recommended vaccines because until recently, he had been receiving chemotherapy for acute lymphoblastic leukemia that was diagnosed at 9 months. Rubeola can be effectively prevented with the MMR vaccine. The child in this case has not received any vaccines, because he has been receiving chemotherapy. Why wasn't he given the MMR vaccine? A) It is an attenuated vaccine and there is a small risk that the virus used in it may revert to virulence, causing serious illness in the immunocompromised patient. B) It is an inactivated vaccine and there is a small risk that the virus used in it may revert to virulence, causing serious illness in the immunocompromised patient. C) The MMR vaccine is known to cause complications such as autism, and this child already has enough problems with leukemia. D) It is a toxoid vaccine—these vaccines contain deactivated microbial toxins; since the child is severely immunocompromised, the toxin used in the vaccine will damage all of his cells. E) The patient is a child, so will not be able to tolerate any egg proteins that may be present in the MMR vaccine, which is produced in eggs.

- It is an attenuated vaccine and there is a small risk that the virus used in it may revert to virulence, causing serious illness in the immunocompromised patient.

Rat bite fever, characterized by fever, rash, and muscle aches, is caused by: - Afipia felis. - Bartonella (Rochalimaea) henselae. - Pasteurella multocida. - Streptobacillus moniliformis.

- Streptobacillus moniliformis. 2nd is cat scratch disease 3rd is in bite wounds Streptobacillary Rat Bite Fever slide 48 Streptobacillus moniliformis: facultatively anaerobic, Gramnegative pleomorphic rod; spontaneously develops L-forms lacking cell wall Correct

Please select the TRUE statement regarding Toll-like receptors (TLRs). - A number of different TLRs have been described, all of which are found in the phospholipid membranes of endosomes and phagosomes. - TLRs allow cells to detect patterns associated with microbes, indicating that the innate immune responses involve some specificity (although not the specificity of adaptive immunity). - TLRs represent a very important early-warning system for viral infections. They allow any virally infected cell to alert neighboring cells that a virus is present. - TLRs anchored in the membrane of endosomes generally detect components of the outermost layers of microbial cells, including lipopolysaccharide (LPS), lipoproteins, and flagellin. - All of these statements are true of TLRs.

- TLRs allow cells to detect patterns associated with microbes, indicating that the innate immune responses involve some specificity (although not the specificity of adaptive immunity).

How do cytokines function? - They are secreted by lysozomes into the phagosomes of neutrophils to enhance the killing of ingested microbes within those organelles. - They are secreted by one cell type, then work as a series of serum proteins to produce holes in the cytoplasmic membranes of microbes to directly lyse them. - They are secreted by microbes that have been engulfed by phagocytes, causing those cells to produce destructive enzymes and undergo apoptosis. - They are secreted by one cell type, then bind to a receptor on target cell causing a signal within that cell that turns on (or off) certain genes to achieve a response. - They are produced by several cell types and bind directly to microbes to enhance their chance of being ingested, in a process called opsonization.

- They are secreted by one cell type, then bind to a receptor on target cell causing a signal within that cell that turns on (or off) certain genes to achieve a response.

A localized collection of pus in a wound is termed a(n): leukocyte. dead tissue. granulation mound. abscess.

- abscess Localized collection of pus surrounded by inflamed tissue. • Pus: thick, yellowish fluid composed of living and dead leukocytes, tissue debris, and proteins • Abscesses form as result of body's defenses • Help localize infection, although cells may escape • Difficult to treat: no blood vessels; adjacent blood vessels often blocked by clots

In many people, the growth of C. acnes within hair follicles leads to: A) eczema. B) carbuncles. C) boils. D) acne. E) all of these.

- acne Cutibacterium acnes multiples in the sebum -> acne "Cutibacterium acnes (formerly Propionibacterium acnes) is the relatively slow-growing, typically aerotolerant anaerobic, Gram-positive bacterium (rod) linked to the skin condition of ACNE"

Most cases of Salmonella gastroenteritis have a(n): A. water source. B. human source. C. plant source. D. animal source.

- animal source

Diphtheroids: A. are part of the normal flora of the skin. B. are responsible for body odor. C. include P. acnes. D. include Malassezia spp. E. are part of the normal flora of the skin, are responsible for body odor AND include P. acnes.

- are part of the normal flora of the skin, are responsible for body odor AND include P. acnes. Normal skin flora, 3 groups: diphtheroids, staphylococcus (G+), Malassezia yeasts - Acne vulgaris - Cutibacterium (Proprionibacteria) acnes is a diphtheroid.

Gas gangrene is so named due to the formation of: A. carbon dioxide. B. oxygen. C. hydrogen. D. carbon monoxide. E. carbon dioxide AND hydrogen.

- carbon dioxide AND hydrogen. • Severe pain, swelling, and thin bloody or brownish fluid leaks from wound; may look frothy (gas bubbles) slide 35 - Organisms multiply using tissue breakdown products, release HYDROGEN AND CARBON DIOXIDE, which accumulate

Which is deemed the most serious staphylococcal skin infection? A. tinea versicolor B. folliculitis C. furuncles D. carbuncles

- carbuncles Folliculitis (pimples) -> Furuncle (boils) -> Carbuncle - May worsen to form carbuncle, a large area of redness, swelling, pain, draining pus Staphylococcus aureus caused folliculitis.

A protein associated with a more virulent form of Staphylococcus is: A) leukocidin. B) coagulase. C) mannose. D) streptokinase. E) catalase.

- coagulase p.577 "Staphylococcus aureus can be distinguished from most other staphylococci because it produces COAGULASE and CLUMPING FACTOR. Coagulates causes a large clot to form in plasma...both coagulase and clumping factor are important virulence factors for S. aureus. Other staphylococcal strains, such as Staphylyococcus epidermidis, cause disease infrequently and lack the genes for coagulase and clumping factor."

Epidemics related to bacterial infection of the digestive system are typically caused by: A) biological vectors. B) contaminated food and water. C) unpasteurized milk. D) the respiratory route. E) exposure to contaminated soil

- contaminated food and water

Part of the ability of S. mutans to result in dental caries depends on its ability to: invade plaque and dissolve the gums. convert sucrose to lactic acid. convert proteins to sugars. attach to the gums.

- convert sucrose to lactic acid.

In humans the most common type of anthrax disease is: A. cutaneous anthrax resulting from the introduction of spores into a break in the skin. B. pulmonary anthrax from inhalation of spores attached to dust. C. gastrointestinal anthrax resulting from consumption of undercooked food contaminated with spores. D. tick borne anthrax resulting from direct introduction of vegetative Bacillus anthracis to the blood stream of a host.

- cutaneous anthrax resulting from the introduction of spores into a break in the skin.

The chains of an antibody molecule are bonded to one another by: - disulfide bonds. - hydrogen bonds. - ionic bonds. - oxygen bonds.

- disulfide bonds

This chemical compound, typically added to drinking water, makes enamel more resistant to dissolving in acid. calcium chlorine chloramine fluoride

- flouride

Wound healing can be slowed by: A. normal flora. B. antiseptic ointments. C. foreign matter D. sweat.

- foreign matter Dirt, dead tissue, foreign matter...

A striking feature of Clostridium tetani that differentiates it from other pathogenic species of Clostridium is its ability to: - produce a greenish pigment. - form spores. - form terminally located, spherical spores. - grow aerobically.

- form terminally located, spherical spores. slide 28 Clostridium tetani: anaerobic, Gram-positive, rod-shaped; forms spherical endospore at one end of cell; has swarming growth that quickly spreads over media surface Correct

All of the following are required for tooth decay except: A) Sucrose. B) Glucose. C) Capsule-forming bacteria. D) Acid-producing bacteria. E) None of the above.

- glucose

The nodular, red, translucent surface material of a healing wound is called: - soluble skin. - irritated scab. - granulation tissue. - abscess.

- granulation tissue Wound healing • Begins with outgrowth of connective tissue (fibroblasts) and capillaries from surface to yield GRANULATION TISSUE • Fills space created by wound • Shrinks, is converted to collagen (scar tissue) • Covered by skin or mucous membrane Correct

Both Staphylococcus aureus and Streptococcus pyogenes: - are rod shaped - cause necrotizing fasciitis -- have fibronectin binding proteins - have exotoxin A

- have fibronectin binding proteins Correct

Tetanus vaccine contains: - inactivated bacteria. - inactivated spores. - live bacteria. - inactivated tetanospasmin.

- inactivated tetanospasmin. Tetanus and diphtheria - toxoid (inactivated) vaccine.

A wound created by the drag of a knife across skin can be classified as: A. puncture. B. incised. C. lacerated. D. contused.

- incised • Incisions: produced by knife or other sharp object

Many childhood diseases caused by viral infections of the upper respiratory tract can usually be diagnosed by: A) inspection of the rash. B) the type of cough. C) the type of fever. D) the incubation period. E) the gender of the patient.

- inspection of the rash.

Most of the normal microbiota of the digestive system are found in the: a. large intestine b. mouth c. small intestine d. stomach e. stomach and small intestine

- large intestine stomach and large intestine are acidic places to be. • Bacteria make up about one-third of fecal weight, reach concentrations of 1011 cells per gram

The popular name for tetanus is: - hydrophobia. - lockjaw. - whooping cough. - consumption.

- lockjaw slide 27

The Rickettsial disease that killed Howard Ricketts and Stanislaus Prowazek was: A) louse-borne typhus. B) tick-borne typhus. C) yellow fever. D) bubonic plague. E) unidentified.

- louse-borne typhus Ricketts studied Rocky Mountain spotted fever (RMSF) but was unable to grow it on lab media . He went on to study similar LOUSE-BORNE TYPHUS which he got and dies from. - Stanislaus Prowazek, studying same disease, met same fate at nearly the same age - Louse-borne typhus named Rickettsia prowazekii - Agent of RMSF named Rickettsia rickettsii

Toll-like receptors are found on: - lymphocytes. - mast cells. - macrophages. - neutrophils.

- macrophages

Complement: - may be activated through three pathways. - disrupts the cytoplasmic membrane of invading bacteria and foreign cells. - is part of the specific defense system. - is a group of blood proteins. - may be activated through three pathways, disrupts the cytoplasmic membrane of invading bacteria and foreign cells, AND is a group of blood proteins.

- may be activated through three pathways, disrupts the cytoplasmic membrane of invading bacteria and foreign cells, AND is a group of blood proteins.

A young mother brings her three-month old baby into the doctor's office where you work as an RN. The baby is miserable and the mother reports that he has developed a fever, which is why she has brought him to the doctor. She tells you that the child is particularly upset when she changes his diaper and that he seems to have diaper rash, which she attributes to the fact that her son's diaper isn't always changed promptly enough at the day-care facility he goes to daily. In fact, she has recently switched him to another facility. You remove the child's diaper and immediately see that he has a very red rash on his buttocks and genital area. There is also red, scaly skin in the area where his diaper touches his thighs. You suspect that this may be more than just a typical case of diaper rash. The mother asks you whether her child needs penicillin. You tell her: A) yes—diaper rash is a serious infection, and should be treated promptly with this antibiotic. B) yes—penicillin interrupts protein synthesis, so will kill any type of cell, including a fungus. C) no—penicillin targets peptidoglycan synthesis, but C. albicans is a fungus, so has a chitin cell wall. D) no—penicillin is no longer an effective antibiotic for treating any type of infection. A different antibiotic is needed. E) no—killing C. albicans with an antibiotic increases the risk of endotoxic shock.

- no—penicillin targets peptidoglycan synthesis, but C. albicans is a fungus, so has a chitin cell wall.

Which of the following correctly lists the stages of tooth decay? A) plaque accumulation, destruction of enamel, advancement of decay through enamel, decay in dentin, decay in tooth pulp B) decay in dentin, plaque accumulation, destruction of enamel, advancement of decay through enamel, decay in tooth pulp C) plaque accumulation, destruction of enamel, decay in tooth pulp, advancement of decay through enamel, decay in dentin D) destruction of enamel, advancement of decay through enamel, plaque accumulation, decay in dentin, decay in tooth pulp E) destruction of enamel, plaque accumulation, advancement of decay through enamel, decay in dentin, decay in tooth pulp

- plaque accumulation, destruction of enamel, advancement of decay through enamel, decay in dentin, decay in tooth pulp

Interleukins are: - involved in directly killing tumor cells. - produced by leukocytes AND protein molecules. - produced by leukocytes. - protein molecules. - important in both innate and adaptive immunity.

- produced by leukocytes AND protein molecules.

The protein produced by S. aureus that interferes with phagocytosis is: A. protein M. B. collagen. C. protein A. D. capsular protein.

- protein A. S. aureus strains have many different virulence factors • Nearly all have cell wall component protein A; many synthesize polysaccharide capsule; Both interfere with phagocytosis - Protein A binds the Fc portion of antibodies Protein M is S. pyogenes - strep throat Collagen is part of connective tissue - gives the skin elasticity.

An important feature of many wounds that may lead to more serious problems is that they are: - well aerated. - relatively anaerobic - well fed. - sterile.

- relatively anaerobic. Reasons for this include: - crush injury - presence of dirt/foreign matter in wound - aerobes using up available oxygen Many wounds RELATIVELY ANAEROBIC, allow growth of obligate anaerobes such as Clostridium tetani • Wounds with extensive tissue damage, contaminated with dirt, or small but deep punctures • Multiple species may also produce anaerobic conditions as facultative anaerobes use up available oxygen Correct

You are studying for your NCLEX exams with a group of friends. Each of you presents a case study to the other students in your group. Your friend Sue gives the following information in her case study: the patient is a child, who presents with a high fever (41oC) and a rash. The parents report that three days prior to the onset of the rash, the child had a runny nose, diarrhea, and conjunctivitis, all of which they thought indicated influenza. The child has not received any of the common recommended vaccines because until recently, he had been receiving chemotherapy for acute lymphoblastic leukemia that was diagnosed at 9 months. The signs and symptoms of the patient suggest that he has: A) rubella. B) rubeola. C) varicella. D) shingles. E) candidiasis.

- rubeola. Measles = rubeola Signs and Symptoms - FEVER, RUNNY NOSE, cough, and SWOLLEN, RED, WEEPY EYES - Fine red RASH appears within a few days, lasts ~1 week

An effective means used early in the 19th century to clear water of the majority of bacteria was the use of chlorine iodine sand filters alcohol

- sand filters sand filtration (since the 1920s) - Later, Koch showed that this kind of filtration yields clear water and removes most bacteria. p.786

You are studying for your NCLEX exams with a group of friends. Each of you presents a case study to the other students in your group. Your friend Sue gives the following information in her case study: the patient is a child, who presents with a high fever (41oC) and a rash. The parents report that three days prior to the onset of the rash, the child had a runny nose, diarrhea, and conjunctivitis, all of which they thought indicated influenza. The child has not received any of the common recommended vaccines because until recently, he had been receiving chemotherapy for acute lymphoblastic leukemia that was diagnosed at 9 months. The rubeola virus damages the respiratory mucous membranes of an infected person, putting them at risk of: A) secondary bacterial infections such as pneumonia and otitis media. B) primary bacterial infections such as pneumonia and otitis media. C) type I hypersensitivities such as asthma and anaphylaxis. D) autoimmune disease such as leukocyte adhesion deficiency. E) All of the answer choices are correct.

- secondary bacterial infections such as pneumonia and otitis media. Rubeola (measles) - • Often secondary infections lead to EARACHES, bacterial PNEUMONIA; caused by S. aureus, S. pneumoniae, S. pyogenes, H. influenzae

Most of the normal microbiota of the digestive system are found in the: A) mouth. B) stomach. C) stomach and small intestine. D) small intestine and large intestine. E) accessory structures

- small intestine and large intestine

The fungal disease that may be associated with sphagnum moss is: - candidiasis. - actinomycosis. - cat scratch fever. - sporotrichosis.

- sporotrichosis. First is yeast that causes thrush and yeast infections. Second is Lumpy jaw (bacterial), third is a zoonosis of cats - bartonellosis. Last is fungal, associated with vegetation so yes.

The bacteria that appear to maintain balance between the members of the normal flora and play a vital role in limiting colonization by pathogens are: A. staphylococci. B. diptheroids. C. Candida spp. D. Malassezia spp. E. streptococci

- staphylococci. Staphylococci: salt-tolerant, use nutrients and produce antimicrobial substances active against other Gram positive bacteria

S. pyogenes associated with invasive disease characteristically have: A. leukocidins. B. coagulase. C. streptococcal pyrogenic exotoxin A. D. streptococcal pyrogenic exotoxin B. E. streptococcal pyrogenic exotoxins A AND B.

- streptococcal pyrogenic exotoxins A AND B.: slide 18 Group A Streptococcal "Flesh-Eating Disease" Virulent strains of Streptococcus pyogenes produce damaging streptococcal pyrogenic exotoxins (SPEs) and enzymes - Exotoxin A: superantigen that causes toxic shock - Exotoxin B: protease that destroys tissue

The exotoxin produced by C. tetani is: - tetanoxin. - exotetanus. - tetanospasmin. - endospasmin.

- tetanospasmin slide 28/29 Exotoxin tetanospasmin released from cells - A-B toxin - Plasmid-encoded Exotoxin tetanospasmin is an A-B toxin - B portion attaches to receptors on motor neurons - A portion taken up by endocytosis, is carried to neuron cell body in spinal cord - Prevents release of neurotransmitter from inhibitory neurons, so muscles contract without control Correct

Which of the following is NOT true for both hepatitis B and hepatitis C infection? The virus can cause a chronic disease. The disease is prevented by vaccination. The infection is transmitted through blood and body fluids. Diagnosis can be achieved through testing the blood for antibodies.

- the disease is prevented by vaccination There is a vaccine for HepB but there is no vaccine available for preventing HepC. It is recommended to be vaccinated for HepA and HepB to help prevent a combination of infections that might severely damage the liver.

Hepatitis A spreads via: the respiratory route. blood transfusion. body fluids. the fecal-oral route.

- the fecal-oral route Body fluids is Hepatitis B Blood transfusion is Hepatitis C. - Hepatitis C is the most chronic blood borne infection in the US.

Typhoid fever differs from salmonellosis in that in typhoid fever: A) the microorganisms multiply within macrophages. B) the incubation period is much longer. C) the symptoms are due to infection of the gallbladder. D) infection is spread only in the feces of domesticated animals. E) infection occurs due to ingestion of endospores

- the incubation period is much longer. Thyphoid fever and paratyphoid fever are ENTERIC FEVERS - systemic diseases that originate in the intestines. - caused by specific serotypes of Salmonella enterica and spread from person to person through fecal-oral transmission. - colonize the intestines, cross the mucus membrane via M cells, multiply inside macrophages and then are carried in the bloodstream to locations body-wide. INCUBATION TIME = 1-4 WEEKS Salmonellosis INCUBATION TIME IS 2 HOURS TO 6 DAYS

Rubella, rubeola, and varicella-zoster are all only acquired via: A) the gastrointestinal route. B) the respiratory route. C) wounds. D) blood transfusions. E) arthropods.

- the respiratory route Problem with this question is that the portal(s) of entry for varicella-zoster virus is/are: - skin AND respiratory route.

The portal(s) of entry for varicella-zoster virus is/are: A) the skin. B) the respiratory tract. C) the genitourinary tract. D) the genitourinary tract AND the gastrointestinal tract. E) the skin AND the respiratory tract.

- the skin AND the respiratory tract. "Transmitted by respiratory secretions and skin lesions"

You are studying for your NCLEX exams with a group of friends. Each of you presents a case study to the other students in your group. Your friend Sue gives the following information in her case study: the patient is a child, who presents with a high fever (41oC) and a rash. The parents report that three days prior to the onset of the rash, the child had a runny nose, diarrhea, and conjunctivitis, all of which they thought indicated influenza. The child has not received any of the common recommended vaccines because until recently, he had been receiving chemotherapy for acute lymphoblastic leukemia that was diagnosed at 9 months. In rare cases, the rubeola virus leads to rapid breathing, shortness of breath, and dusky skin color; these are signs and symptoms of: A) viral meningitis. B) otitis media. C) strep throat. D) viral pneumonia. E) subacute sclerosing panencephalitis.

- viral pneumonia. These are respiratory signs/symptoms - rapid breathing, SOB, the cyanosis from lack of O2 that makes me think pneumonia instead of the others. - Subacute sclerosing panencephalitis (SSPE) is a progressive neurological disorder of children and young adults that affects the central nervous system (CNS). It is a slow, but persistent, viral infection caused by defective measles virus. - Subacute sclerosing panencephalitis is a progressive, usually fatal brain disorder occurring months to usually years after an attack of measles. It causes mental deterioration, myoclonic jerks, and seizures. Is why we need to keep up on vaccinations - so we don't see more cases of this down the road.

If an individual in the infectious stage of chickenpox comes into contact with a person who had chickenpox as a child, that second person: A) will be unaffected by the contact. B) will develop a second case of chickenpox. C) will develop shingles. D) will develop a second case of chickenpox OR will develop shingles. E) will remain healthy if they receive zoster immune globulin.

- will be unaffected by the contact.

After being bitten by an infected tick, transfer of the rickettsial organism occurs: A) within 5 minutes. B) within 20 minutes. C) within 4-10 hours. D) immediately. E) after 7 days.

- within 4-10 hours Transmitted by tick bite - 4-10 hours of feeding

The toxin implicated in C. perfringens toxicity is: - tetanospasmin. - exoenzyme S. - alpha-toxin. - endoenzyme T.

-alpha-toxin slide 34 Releases α-toxin, an enzyme that destroys lecithin in host cell membranes, resulting in cell lysis

The circulation of an agent in the bloodstream is given a name ending in -ase. -ing. -emia. -ation.

-emia.

Would you expect an individual with giardiasis who has diarrhea to be more likely to transmit the disease than an individual with giardiasis who does NOT have diarrhea?Why or why not? A. No-this illness is spread by respiratory droplets, so diarrhea as a symptom shouldn't matter for transmission of the disease. B. No-this illness is spread by sexual contact, so diarrhea as a symptom shouldn't matter for transmission of the disease. C. Yes-this illness is spread by the fecal-oral route, so presence of diarrhea as a symptom should dramatically increase the possibility of transmission of infection. D. Yes-this illness is spread by insects that feed on contaminated fecal matter, becoming infected themselves. The disease is spread to new individuals when these infected insects bite a susceptible person, transmitting the cysts of the protozoan. As such, diarrhea as a symptom would increase the risk of transmission through biting insects to new individuals.

. Yes-this illness is spread by the fecal-oral route, so presence of diarrhea as a symptom should dramatically increase the possibility of transmission of infection.

Hydrogen sulfide

1) is produced when sulfur compounds are used as terminal electron acceptors. 2) may react with iron to produce black precipitate. 3) is produced by Desulfovibrio. 4) has a rotten egg smell.

Place the steps of the disease pathway provided in the correct order to test your understanding of the pathogenesis of gonorrhea.

1. Neisseria gonorrhoeae attaches to mucous membranes of genitalia, mouth, rectum, or eyes. 2. Primary infection in women is cervix. 3. Outer covering of liver is infected in women when gonococci enter abdominal cavity from infected fallopian tubes. 4. Infection of fallopian tubes or testicular tubules causes scarring, which may lead to ectopic pregnancy (women) or infertility (men and women). 5. Bacteria enter the bloodstream and infect heart and joints.

Please match the statement to the stage of syphilis that it most accurately describes to test your understanding of syphilis. 1. Hard, painless chancre at site of spirochete entry 2. Fever, headache, and sore throat, followed by lymphadenopathy and a peculiar red or brown rash on all skin surfaces, including the palms and the soles 3. Neurosyphilis can be present as well as gummas 4. Syphilis spirochete passes through placenta to infect fetus

1. Primary syphilis 2. Secondary syphilis 3. Tertiary syphilis 4. Congenital syphilis

Most large cities have required the pasteurization of milk and milk products since ________. 1800 1850 1920 1985 1900

1900

Please select the correct sequence of steps involved in the creation of a dead zone in a water body. 1. Heterotrophic microbes metabolize the organic compounds synthesized by primary producers. 2. Water becomes hypoxic and toxic algal blooms may occur; animals in the area either flee or die. 3. Water accumulates nutrients as it runs through agricultural, industrial, and urbanized regions. 4. Dissolved O2 is consumed by metabolic activity of heterotrophs. 5. Excess nitrate and phosphate in nutrient-rich water causes algae and cyanobacteria to flourish. 5; 3; 4; 2; 1 3; 5; 1; 4; 2 3; 1; 4; 2; 5 1; 3; 4; 2; 5 2; 1; 4; 3; 5

3;5;1;4;2

Your brother is an enthusiastic hiker. He returns from a walking vacation and tells you that although he had a good time, he ran into a bit of trouble on one of the days he was away because he had got lost in the woods for several hours. Although he took enough food for the day, your brother ran out of water. Luckily, he came across a small stream and was able to fill up his water bottle and to finish his hike without any further problems. Your brother mentions as an aside that he must have eaten something bad while he was away, because he has had pretty bad diarrhea since he got back. He tells you he doesn't feel too bad, but that he has been having what he describes as "explosive diarrhea." He also complains of abdominal pain. You urge him to go to the doctor because you suspect that he may have something more than just food-related diarrhea. Which statement about rotavirus gastroenteritis is false? Most cases of the disease occur in infants and children. There are no vaccines available for preventing rotavirus gastroenteritis. Rotaviruses are transmitted by the fecal-oral route. Rotaviruses mainly infect the epithelial cells in the upper part of the small intestine .Deaths related to the disease are usually caused by dehydration Giardiasis involves: treatment with mitochondria-targeting medications. production of resistant, infectious cysts. a protozoan that uses fimbriae for attachment.

5 of these... When searching this, spell enthusiastic as 'enthusiatic'. The false one is : There are no vaccines available for preventing rotavirus gastroenteritis. Giardiasis involves: production of resistant, infectious cysts.

Your brother is an enthusiatic hiker. He returns from a walking vacation and tells you that although he had a good time, he ran into a bit of trouble on one of the days he was away because he had got lost in the woods for several hours. Although he took enough food for the day, your brother ran out of water. Luckily, he came across a small stream and was able to fill up his water bottle and to finish his hike without any further problems. Your brother mentions as an aside that he must have eaten something bad while he was away, because he has had pretty bad diarrhea since he got back. He tells you he doesn't feel too bad, but that he has been having what he describes as "explosive diarrhea." He also complains of abdominal pain. You urge him to go to the doctor because you suspect that he may have something more than just food-related diarrhea. There are several stages in the life cycle of G. lamblia. Which is the correct sequence? 1. Trophozoites multiply in the intestine. 2. Mature cysts or trophozoites are released in feces. 3. Cysts pass through the stomach to the lower small intestine. 4. Dehydration in the large intestine stimulates formation of cysts. 5. Cysts enter the mouth by ingestion of contaminated food or water. 6. Trophozoites are released from the cysts. 5, 3, 6, 1, 4, 2 2, 5, 6, 1, 3, 4 5, 2, 3, 1, 6, 4 1, 2, 3, 4, 5, 6 2, 3, 1, 5, 4, 6

5, 3, 6, 1, 4, 2 Correct

Why is it difficult to treat P. aeruginosa wound infections?

? Only a few antimicrobial medications effective • Established infections difficult to treat

Antibiotics are given to a person with gas gangrene to:

? Clostridial myonecrosis = Gas Gangrene Treatment. If gas gangrene is suspected, treatment must begin immediately. High doses of antibiotics, typically penicillin and clindamycin, are given, and all dead and infected tissue is removed surgically. - the antibiotic is to kill the bacteria itself - Clostridium perfringins - Gram-positive, rod-shaped, anaerobic, spore-forming pathogenic bacterium of the genus Clostridium.

Mycoplasma genitalium infections frequently go undiagnosed for all of the following reasons EXCEPT: p.744 - no widely available detection methods are available - infections are often asymptomatic - when signs and symptoms do occur, they can be similar to those of other bacterial STIs.

?? Mycoplamsa genitalium is an emerging cause of genital tract infections in men and women worldwide. p.744 - it cannot be detected by gram stain. - very difficult to culture. Fastidious and grow very slowly, generation time of at least 16 hrs and takes months to grow in vitro. - infections are frequently asymptomatic. - signs and symptoms are similar to those of chlamydia and gonorrhea.

A common causative agent of acute endocarditis is: A. Staphylococcus aureus B.Listeria monocytogenesC.Candida albicansD.Neisseria gonorrheaE.All of the choices are correct.

A

C3b is involved in: A) opsonization. B) interferon production. C) properdin activation. D) endotoxin production. E) inflammation.

A

Production of a neurotoxin that prevents acetylcholine release from motor neurons at neuromuscular junctions is a characteristic of: A.Clostridium botulinumB.Clostridium perfringensC.Clostridium difficileD.Clostridium tetaniE.All of the choices are correct.

A

The causative organism for mumps is: A. Paramyxovirus B.CorynebacteriumC.Streptococcus pyogenesD.VibrioE.Morbillivirus

A

Which of the following are enzymes found in neutrophil granules, saliva, and milk that react with hydrogen peroxide to form antimicrobial compounds? A) Peroxidases B) Lactoferrins C) Transferrins D) Peroxidases AND lactoferrins E) Lactoferrins AND transferrins

A

Which of the following is the correct sequence of events in the interferon response? 1. IFN induces synthesis of inactive antiviral proteins (iAVPs). 2. Virus enters host cell (cell 1) and replicates, producing long dsRNA. 3. Activated AVPs induce apoptosis of infected cell. 4. dsRNA induces synthesis and secretion of IFN to neighboring cells. 5. Entry of virus into cell armed with iAVPs (cell 2) activates these proteins. A) 2, 4, 1, 5, 3 B) 1, 2, 3, 4, 5 C) 3, 1, 2, 5, 4 D) 5, 4, 1, 2, 3 E) 2, 1, 4, 3, 5

A

Which of the following are referred to as mononuclear phagocytes? A) Monocytes and macrophages B) Lymphocytes and basophils C) Mast cells and eosinophils D) Basophils and eosinophils E) Neutrophils and macrophages

A - The mononuclear phagocyte system includes monocytes, macrophages/histiocytes, and dendritic cells. Macrophages/histiocytes are tissue-based cells that have an important phagocytic function and produce a variety of bioactive substances that play roles in inflammation and fibrosis.

Why might an individual with an STD need to be checked for other STDs, even though he/she has no symptoms of any others? A. Several STD-causing pathogens are known to 'piggy back' on each other, being carried into the genital tract together. B. A diagnosis of an STD implies that the individual engaged in unprotected sexual intercourse. Several STDs are largely asymptomatic, so it is wise to test for a variety of typical STDs when the patient admits to or shows signs of engaging in such risky behavior. C. They don't need to be tested-all STDs are highly obviously symptomatic, so there is no need to test for other diseases than the one currently manifesting itself in the patient. D. It's a public health issue-it's best to know who is positive for what in the population of sexually-active individuals. This lets healthcare professionals act to best safeguard the population as a whole from the range of STDs.

A diagnosis of an STD implies that the individual engaged in unprotected sexual intercourse. Several STDs are largely asymptomatic, so it is wise to test for a variety of typical STDs when the patient admits to or shows signs of engaging in such risky behavior.

Macroscopic algae possess a special structure that acts as an anchor and is commonly called

A holdfast

Why is puerperal fever not regarded as a sexually transmitted disease? What spreads the disease? A. A physician that doesn't observe proper sterile technique may introduce bacteria from the environment into the genitourinary tract of a woman during delivery of a child, inducing puerperal fever several days later. B. The trauma of the delivery of the placenta through the vaginal tract after the fetus introduces bacteria directly into the bloodstream, resulting in puerperal fever. C. A woman's own normal fecal microbiota can occasionally cause puerperal fever if it contaminates the genitourinary tract following delivery. This can especially occur after an episiotomy procedure (cutting between the vaginal opening and the rectum). D. A physician that doesn't observe proper sterile technique may introduce bacteria from the environment into the genitourinary tract of a woman during delivery of a child, inducing puerperal fever several days later AND a woman's own normal fecal microbiota can occasionally cause puerperal fever if it contaminates the genitourinary tract following delivery. This can especially occur after an episiotomy procedure (cutting between the vaginal opening and the rectum).

A physician that doesn't observe proper sterile technique may introduce bacteria from the environment into the genitourinary tract of a woman during delivery of a child, inducing puerperal fever several days later AND a woman's own normal fecal microbiota can occasionally cause puerperal fever if it contaminates the genitourinary tract following delivery. This can especially occur after an episiotomy procedure (cutting between the vaginal opening and the rectum).

Which is not involved in adaptive immunity? A) Tear flow B) Antibody production C) T cells D) B cells E) Tear flow AND B cells

A) Tear flow

Your first patient of the day is complaining of pain in the area of his new tattoo. When you examine the tattoo, you note that the area around it is firm, red, and swollen. There is no evidence of pus, and your patient has a normal pulse, and no fever. You tell your patient that you have seen this type of response to a tattoo before, and that it is called a granulomatous reaction. You give him more information on the condition. You tell your patient that tattoo ink can sometimes cause the skin reaction he is experiencing; red ink in particular can cause this because it commonly contains mercury and/or other heavy metals. Your patient has developed a granuloma at the site of his tattoo. When he asks you what a granuloma is, you tell him: A) a collection of macrophages, giant cells, and T cells. B) a collection of neutrophils, giant cells, eosinophils and red blood cells. C) a mass of macrophages, activated complement proteins, and keratin. D) an acute inflammation response involving mast cells and dendritic cells. E) a collection of microbes, keratin, and white blood cells.

A) a collection of macrophages, giant cells, and T cells.

What is the consequence of fever? A) inhibits bacterial growth AND speeds up the body's reactions B) enhances bacterial growth AND speeds up the body's reactions C) inhibits bacterial growth AND triggers complement activation D) speeds up the body's reactions AND triggers complement activation E) enhances bacterial growth AND triggers complement activation

A) inhibits bacterial growth AND speeds up the body's reactions

Fever: A) inhibits bacterial growth AND speeds up the body's reactions. B) enhances bacterial growth AND speeds up the body's reactions. C) inhibits bacterial growth AND triggers complement activation. D) speeds up the body's reactions AND triggers complement activation. E) enhances bacterial growth AND triggers complement activation.

A) inhibits bacterial growth AND speeds up the body's reactions.

Factors that work generically against any foreign substance entering the host are described as: A) innate immunity. B) specific immunity. C) irregular immunity. D) immune metabolism. E) adaption.

A) innate immunity.

A. The outermost layer of skin. B. In Lyme disease the nervous system signs and symptoms appear 2-8 weeks post-infection. C. A boil; a localized skin infection that penetrates into the subcutaneous tissue, usually caused by Staphylococcus aureus. D. Mucus-secreting epithelial cell. E. A bacterial toxin that causes sloughing of the outer epidermis. F. A localized collection of pus within a tissue. G. A common skin condition in which pores on the face, neck, chest, or back become plugged and the surrounding skin becomes inflamed. H. Fungus that lives on the skin and can be responsible for disease of the hair, nails, and skin. I. Plasma-clotting virulence factor of Staphylococcus aureus that serves as an identifying characteristic. J. Slight changes in a viral surface antigen render antibodies made against the previous version only partially protective. K. A protein in the cell wall of Group A streptococci that is associated with virulence. L. Lesions of the oral cavity caused by measles virus that resemble a grain of salt on a red base. M. Multidrug-resistant Mycobacterium tuberculosis. N. An nflammation of the lungs accompanied by filling of the air sacs with fluids such as pus and blood. O. A worldwide epidemic. P. A post-streptococcal sequela thought to be due to circulating immune complexes. Q. Condition resulting from the reactivation of the varicella-zoster virus. R. Test for tuberculosis involving the intracutaneous injection of 0.1 mL of PPD into the forearm; relies on a delayed-type hypersensitivity reaction triggered by past infection, significant exposure to tuberculosis. S. A tough layer of dead cells and debris accumulated on an epithelial surface. T. Caused by Mycoplasma pneumoniae.

A. epidermis B. Early disseminated infection C. Furuncle D. goblet cell E. exfoliatin F. abscess G. acne H. dermatophyte I. coagulase J. antigenic drift K. M protein L. Koplik spots M. Multidrug-resistant TB, MDR-TB N. pneumonia O. Pandemic P. Rhuematic fever Q. Shingles R. Tuberculin skin test, TST S. pseudomembrane T. walking pneumonia

Gastroenteritis A.Acute inflammation of the stomach and intestines; often applied to the syndrome of nausea, vomiting, diarrhea, and abdominal pain. Jaundice E.Yellow color of the skin and eyes caused by buildup of bilirubin in the blood. Microvilli D.Tiny cylindrical projections from luminal surfaces of cells such as those lining the intestine; increases surface area of the cell. Myonecrosis C.Also known as gas gangrene. Potassium iodide B.Effective as a non-antibiotic treatment for infection with Sporotrichosis.

Abrasion C.Occurs where the epidermis is scraped off. Bile E.Yellow-colored fluid produced by the liver that aids in the absorption of nutrients from the intestine. Collagen B.Fibrous support protein found in skin, tendons, scars, and other tissues. Contusion D.Produced by a blow that crushes tissue. Cyst A.Dormant resting protozoan cell characterized by a thickened cell wall.

Yellow fever is transmitted by ticks. fleas. Anopheles mosquitoes. Aedes mosquitoes.

Aedes mosquitoes. Bunch of viruses from mosquitoes - Yellow fever - main vector = Aedes aegypti mosquito - Dengue fever - main vector = Aedes aegypti and Aedes albopictus - Chikungunya - Aedes aegypti and Aedes albopictus mosquitoes - Zika - Aedes aegypti and Aedes olbopictus (less frequently) Anopheles mosquitoes carry Plasmodium for Malaria. Fleas is Yerisinia pestis and plague. Ticks is Lyme disease - but Ebola and Marburg - fruit bats reservoir

Found in grains spoiled by Aspergillus

Aflatoxin

Which disease is INCORRECTLY matched with its arthropod vector?

African sleeping sickness- sand fly

You go hiking in the woods with some of your classmates. A few days after the hike, your friend Alice calls to tell you she is not feeling well and that she is going to skip soccer practice that evening. She says she has a weird rash on her leg and wonders if she was bitten by a spider while your were hiking. She sends you a photo of the rash. You are in your final semester of nursing school and have recently studied disease of the cardiovascular system. You think that the rash that Alice has is a sign of Lyme disease, and offer to take her to the urgent care to get medical help. While you are waiting to see a clinician, you review answer Alice's questions about Lyme disease.What causes the characteristic erythema migrans of Lyme disease? Borrelia bergdorferi is an encapsulated organism. The capsule of these cells is composed of hyaluronic acid that causes a widespread inflammatory response in the skin of the host that manifests as a rash. Borrelia bergdorferi cells propel themselves from host cell to host cell using actin tails, in a manner similar to Shigella cells. As they move, the bacteria destroy the host cells, causing the characteristic rash. After being introduced into the skin by the bite of an infected mosquito, the Gram-positive bacteria migrate outward in a corkscrew-like manner; the NAGs and NAMs in their cell wall causes an inflammatory reaction. After a person acquires the causative bacteria by eating contaminated meat, the bacteria migrate to the skin in a corkscrew-like motion, killing host cells as they move and causing the bull's-eye rash. After being introduced into the skin by the bite of an infected tick, the Gram-negative bacteria migrate outward in a circular manner; the LPS in their outer layer causes an inflammatory reaction.

After being introduced into the skin by the bite of an infected tick, the Gram-negative bacteria migrate outward in a circular manner; the LPS in their outer layer causes an inflammatory reaction. p.672 Correct

Explain how nutrient-rich runoff can cause waters to become hypoxic. The nutrients in the runoff often include toxic chemicals that kill anaerobes such as cyanobacteria. Aerobes flourish, depleting the amount of oxygen present. This consumption of oxygen leads to a hypoxic state in the area. Larger animals cannot survive in the hypoxic area. Algae and cyanobacteria flourish on the nutrients in the runoff. Heterotrophic microbes then flourish on the organic molecules produced by these organisms, using oxygen in the process and leading to a hypoxic state in the area. Larger animals cannot survive in the hypoxic area. It doesn't. This claim is a scare tactic used by environmental extremists to prevent farmers and individuals from fertilizing their crops and lawns. There is no induction of a hypoxic state in bodies of water where runoff occurs. Algae and cyanobacteria flourish on the nutrients in the runoff, using oxygen in the process. This consumption of oxygen leads to a hypoxic state in the area. Larger animals cannot survive in the hypoxic area. The nutrients in the runoff often include chemicals that spontaneously oxidize in the presence of oxygen. They tend to deplete the amount of oxygen present in an area simply by being present themselves. This consumption of oxygen leads to a hypoxic state in the area. Larger animals cannot survive in the hypoxic area.

Algae and cyanobacteria flourish on the nutrients in the runoff. Heterotrophic microbes then flourish on the organic molecules produced by these organisms, using oxygen in the process and leading to a hypoxic state in the area. Larger animals cannot survive in the hypoxic area.

Why is it important to learn about rabies when only a few cases occur in the United States each year? A. The US isn't the entire world. Rabies might be more common in other areas, and it's important to understand due to its lethality. B. The only reason why rabies isn't common in the US is due to our lack of interaction with animal carriers. We could very easily have an outbreak on our hands if we don't know the warning signs to watch for that we've learned about by studying the virus and disease. C. Rabies can never be completely eliminated-it has too many different animal reservoirs/carriers. As such, we need to remain constantly aware of how to protect human populations, both by preventing and treating the illness. We can only do that by learning about it. D. All of the above are correct.

All of the above are correct.

Which is (are) true of coenzymes? They are organic molecules. They may bind to a number of different enzymes. They are synthesized from vitamins. They transfer atoms from one molecule to another. All of the answer choices are correct.

All of the answer choices are correct.

Preservation of food to limit microbial survival and growth includes A. High temperature and pressure B. Pasteurization C. Refrigeration and freezing D. Irradiation E. All of the choices are correct

All of the choices are correct

Virulence factors of S. pyogenes include: A. Polysaccharides on the cell wall B. Lipoteichoic acid C. Spiky M-proteins D. Capsule to provide adherence E. All of the choices are correct

All of the choices are correct

Which of the following may be used by prokaryotes, under anaerobic conditions, as terminal electron acceptors? carbon dioxide sulfate nitrate nitrite All of the choices are correct.

All of the choices are correct

Naturally occurring microbial chemicals may include benzoic acid. lysozyme. allicin. peroxidase. All of the choices are correct

All of the choices are correct benzoic acid - in cranberries lysozyme - in egg whites allicin - in garlic peroxidase -Peroxidase is an enzyme found in a wide variety of organisms, from plants to humans to bacteria. Its function is to break down hydrogen peroxide (H2O2), which is one of the toxins produced as a byproduct of using oxygen for respiration. (The fact that it's toxic is what makes hydrogen peroxide useful in first aid kits.

Which is true of listeriosis? A. It is a foodborne disease. B. It may result in meningitis. C. It is usually asymptomatic in healthy people. D. It is caused by an organism that can grow at refrigerator temperatures. E. All of the choices are correct.

All of the choices are correct p.702 - Meningitis is the most common result of listeriosis, a foodborne disease caused by Listeria monocytogenes. - motile, non-spore forming, facultative anaerobe, Gram positive rod that can grow at 4 C even on vacuum-packed refridgerated food - epidemics - GI route - can cross the placenta - abscesses in fetus

Untreated gonorrhea in males may lead to sterility. urinary tract infections. prostatic abscesses. orchitis. All of the choices are correct.

All of the choices are correct p.741 - In men, an inflammatory response to the infection can cause scar tissue formation that partially obstructs the urethra, slowing urination and creating a predisposition to UTIs. The infection may spread to the prostate gland and the testes, producing prostatic abscesses and orchitis (inflammation of the testes). If scar tissue blocks the tubes that carry the sperm, or if testicular tissue is destroyed by the infection , infertility can result.

Limiting the activity of microbes in food can be accomplished by cold storage. lowering the pH. growth inhibiting substances. pasteurization. All of the choices are correct.

All of the choices are correct.

Sludge A. is a byproduct of sewage treatment. B. may be a source of pollution. C. takes up space in a land fill. D. All of the choices are correct

All of the choices are correct.

The antimicrobial aspect(s) of the skin is/are: A. dead layers. B. saltiness. C. acidity. D. antimicrobial peptides. E. All of the choices are correct.

All of the choices are correct.

Treponema pallidum -is the organism that causes syphilis -is a spirochete -can be more easily viewed with dark field illumination -has become less virulent over time

All of the choices are correct.

Wastewater treatment decreases the amount of phosphate. biodegradable carbon. pathogens. ammonia and nitrate. All of the choices are correct.

All of the choices are correct.

Central nervous system (CNS) diseases caused by fungus may occur in: A. cancer patients. B. diabetics. C. AIDS patients. D. immunosuppressed people. E. All of the choices are correct.

All of the choices are correct. - in immunocompromised individuals

Which is true of the Clostridium botulinum toxin? A. It is heat-sensitive. B. It is a neurotoxin. C. It blocks nerve to muscle signal transmission. D. It is a two-part toxin. E. All of the choices are correct.

All of the choices are correct. Botulinum toxin - C. botulinum is G+ so is an exotoxin - heat sensitive - is a neurotoxin - attaches to motor neurons and stops transmission of signals to the muscles, producing paralysis - is an A-B toxin - B on motor nerve endings. A in nerve cell where it inactivates proteins that regulate the release of neurotransmitters - stops muscle contraction = Flaccid paralysis

Which is true about C. botulinum? A. It is anaerobic. B. It may form spores. C. It produces a neurotoxin. D. It does not grow well below pH 4.5. E. All of the choices are correct.

All of the choices are correct. Botulism is not a nervous system disease but its key symptom is paralysis. Clostridium botulinum - G+, anaerobic, endospore-forming, rod shaped bacterium. The endospore can germinate in favorable conditions = nutrient-rich, anaerobic environment, a pH above 4.5 and a temperture above 4 C. Botulinum toxin (types A, B and E) is a neurotoxin and causes paralysis.

Treatment of gas gangrene may involve _________. A. wound debridement B. antitoxins C. hyperbaric chambers D. All of the choices are correct.

All of the choices are correct. Clostridium perfringens -> Clostridial Myonecrosis Treatment and Prevention • Prompt surgical removal of all dead and infected tissue = DEBRIDEMENT • Antibiotics, HYPERBARIC oxygen treatment • No available vaccine Makes ALPHA TOXIN so ANTITOXIN, too

The normal flora of the lower urethra may show: Lactobacillus. Staphylococcus. Corynebacterium. Bacteroides. All of the choices are correct.

All of the choices are correct. p.7.29 - the lower urethra has a normal resident microbiota that includes species of Lactobacillus, Staphylococcus, Corynebacterium, Haemophilus, Streptococcus and Bacteroides.

Bacterial cystitisis: a common nosocomial disease. is typically caused by E. coli. may occur through the use of a catheter. is unusual in men under 50. All of the choices are correct.

All of the choices are correct. p.730/31 - is a frequent HAI - usually originates from the normal intestinal microbiota - more than 80% of cases are caused by specific uropathogenic strains of E. coli. - hospitalized patients, and people with long-standing bladder catheters are often chronically infected - catheters allow for formation of biofilms that let bacteria reach bladder - UTIs are unusual in men until about age 50 when the prostate enlarges

Which of the following is important in the treatment of potential infections by Clostridium tetani? A. Proper care of wounds contaminated with soil B. Prophylactic use of antitoxin C. Administration of penicillin D. All of the choices are important in the treatment of potential infections by Clostridium tetani.

All of the choices are important in the treatment of potential infections by Clostridium tetani. Clostridium tetani: anaerobic, GRAM POSITIVE, rod-shaped; forms spherical endospore at one end of cell; swarming growth quickly spreads over media surface Treatment and Prevention • Human tetanus immune globulin (TIG) injection - Antibodies bind to free toxin molecules, provide passive immunity, but do not neutralize attached tetanospasmin or repair nerve damage - Muscle relaxants, supportive care necessary • CLEANING WOUND, antibiotics, vaccine • VACCINATION with tetanus toxoid (inactivated tetanospasmin) PREVENTS - Table 23.3 says TETANUS ANTITOXIN

Why is it important to continue vaccinating children in the United States against measles, even though it is now a rare disease in the Western Hemisphere? A) A rare complication of measles is subacute sclerosing panencephalitis (SSPE) that is characterized by progressive brain degeneration, and generally results in death within 2 years. B) Measles that occurs during pregnancy increases the risk of miscarriage, premature labor, and low-birth-weight babies. C) Occasionally, the measles virus causes viral pneumonia, with rapid breathing, shortness of breath, and dusky skin color from lack of adequate O2 in the blood. D) Encephalitis is a rare but serious complication of measles that sometimes results in permanent brain damage, with mental disability, deafness, and epilepsy. E) All of these complications are reasons that the measles vaccine should be continued in the United States.

All of these complications are reasons that the measles vaccine should be continued in the United States.

What is a definitive host in the life cycle of a parasite?

An organism in which sexual reproduction or the adult form of a parasite occurs.

Influenza virus can exhibit constant mutation of viral glycoproteins, called ________, or __________, a more serious phenomenon caused by the exchange of a viral gene with that of another influenza virus strain. A. Antigenic drift, antigenic shift B. Antigenic shift, antigenic drift C. Antigenic drift, antigenic drift D. Antigen resistance, antigen cooperation E. None of these

Antigenic drift, antigenic shift drift is a small change, shift is big and serious.

Which of the following is NOT a characteristic of meningococcus? A. Antigenic variation prevents the development of a vaccine. B. It uses pili for attachment to mucosal epithelium. C. It is a diplococcus. D. It is an encapsulated bacterium. E. It can vary its surface antigens.

Antigenic variation prevents the development of a vaccine. WRONG - I checked the PP and the textbook and it has to be A. Neisseria miningitidis - Gram negative encapsulated diplococcus - most serious infections are due to serotypes A, B, C, W and Y - inhaled in airborne droplets attach by pili to mucus membranes and then multiply. Like N. gonorrheae, N. meningitidis can vary some of its antigens. Hello, there are 2 vaccines p.700

Normal microbiota..

Are the organisms that typically reside on your body AND protect against infection by pathogens.

Please select the FALSE statement regarding arthropod vectors.

Arthropods are very specific and always bite only one type of host.

This organism commonly contaminates grains, corn, and peanuts and produces a potent mycotoxin called aflatoxin A. Aspergillus flavis B. Pneumocystis (carinii) jiroveci C. Fusarium species D. Rhizopus species E. Geotrichum candidum

Aspergillus flavis

Soy sauce is made by fermentation of soybeans and wheat by - lactic acid bacteria. - Saccharomyces. - Aspergillus. - lactic acid bacteria AND Saccharomyces.

Aspergillus. Soy sauce is made by inoculating equal parts of cooked soybeans and roasted cracked wheat with a culture of either Aspergillus oryzae or Aspergillus sojae. The mixture (called KOJI) stands for several days while carbs and proteins in soybeans are broken down - making a yellow-green liquid full of fermentable sugars, peptides and amino acids. This is added to a solution of 18% NaCl (brine) where salt-tolerant microbes grow and produce flavor changes over time. Microorganisms involved in this stage of fermentation: lactobacilli, pediococci, and yeasts. The brine sits for 8-12 months, then the liquid soy sauce is removed and the remaining solids are used as animal feed.

What are A/E lesions, produced by some strains of E. coli?

Attachment and Effacing Lesions - makes the cells of the intestine made a pedestal under the bacteria. Is part of how it attaches/colonizes.

All of the following pertain to Neisseria meningitidis except: A. Virulence factors include a capsule, pili, endotoxin, and IgA protease B.Common cause of bacterial pneumoniaC.Causes the most serious form of acute meningitisD.Causes formation of petechiaeE.More easily transmitted in schools, day care facilities, dorms, and military barracks

B

When microbes are introduced into normally sterile body sites, or when tissues are damaged, inflammation occurs. The purpose of this is to contain a site of damage, localize the response, eliminate the invader, and restore tissue function. Select the correct sequence of events for an inflammation response. 1. The phagocytes then produce other adhesion molecules that strengthen their attachment to the endothelial cells. 2. Tight junctions between endothelial cells are disrupted, allowing leakage from the vessels into the tissue. 3. Endothelial cells of the blood vessels produce adhesion molecules that loosely "grab" phagocytic cells. 4. In response to various chemoattractants, the phagocytes leave the blood vessels and move into the surrounding tissues. 5. The diameter of local blood vessels increases due to the action of inflammatory mediators, slowing blood flow in the area. A) 1, 2, 3, 4, 5 B) 5, 2, 3, 1, 4 C) 3, 2, 4, 1, 5 D) 1, 3, 5, 2, 4 E) 4, 5, 1, 2, 3

B

Which is true of viral conjunctivitis? A. Caused by Chlamydia trachomatis or Neisseria gonorrhea B.Caused by adenovirusesC.Caused by MoraxellaD.Must be treated with topical and oral antibioticsE.Has a mucopurulent, milky discharge

B

Your first patient of the day is complaining of pain in the area of his new tattoo. When you examine the tattoo, you note that the area around it is firm, red, and swollen. There is no evidence of pus, and your patient has a normal pulse, and no fever. You tell your patient that you have seen this type of response to a tattoo before, and that it is called a granulomatous reaction. You give him more information on the condition. You go on to explain to your patient that his skin and mucous membranes are protected from microbes by a number of antimicrobial substances. You inadvertently make a mistake during this explanation. Which of the following definitions is INCORRECT? A) Lysozyme—enzyme that degrades the peptidoglycan layer of the bacterial cell wall. B) Defensins—short chains of carbohydrates that have strong antimicrobial properties. C) Peroxidases—enzymes found in neutrophils, saliva, and milk that react with H2O2 to form antimicrobial compounds. D) Transferrin—an iron-binding protein found in blood and tissue fluids that deprives microbes of Fe, inhibiting their growth. E) Perspiration—source of salty residue on the skin that inhibits all microbes except those that are salt-tolerant.

B

Which drugs interfere with the action of an HIV enzyme needed for final assembly and maturation of the virus? A.Reverse transcriptase inhibitorsB.Protease inhibitorsC.Fusion InhibitorsD.Integrase inhibitorsE.All of the choices are correct

B They stop the virions from maturing CORRECT!!!

What two functions do phagocytes serve in immune responses? A) Production of antibodies AND engulfment/destruction of foreign cells. A) Production of antibodies AND engulfment/destruction of foreign cells. B) Engulfment/destruction of foreign cells AND alerting the other cells of the immune system to an invader. C) Alerting the other cells of the immune system to an invader AND serving as a physical barrier against microbial invasion. D) Production of antibodies AND serving as a physical barrier against microbial invasion. E) Alerting the other cells of the immune system to an invader AND production of antibodies. C) Alerting the other cells of the immune system to an invader AND serving as a physical barrier against microbial invasion. D) Production of antibodies AND serving as a physical barrier against microbial invasion. E) Alerting the other cells of the immune system to an invader AND production of antibodies.

B) Engulfment/destruction of foreign cells AND alerting the other cells of the immune system to an invader.

How are macrophages and neutrophils similar, and how are they different? A) Macrophages and neutrophils both circulate in the blood. Macrophages are phagocytic cells while neutrophils are granulocytes but not phagocytes. B) Macrophages and neutrophils are both phagocytic cells. Macrophages reside in the tissues while neutrophils typically circulate in the blood. C) Macrophages and neutrophils are both abundant cell types in tissues. Macrophages are phagocytic cells while neutrophils are not. D) Macrophages and neutrophils are both phagocytic cells. Macrophages are granulocytes while neutrophils are effector cells. E) Macrophages and neutrophils are both leukocytes. Macrophages are granulocytes and neutrophils are lymphocytes.

B) Macrophages and neutrophils are both phagocytic cells. Macrophages reside in the tissues while neutrophils typically circulate in the blood.

The body's own cells do not trigger the alternative pathway of complement system activation. Why is this? A) Host cells produce convertase 3b which changes C3b into C3a that does not activate the complement pathway. B) Molecules in host cell membranes bind regulatory proteins that will inactivate any C3b molecules that attach to the membrane. C) C3b triggers the alternative pathway in prokaryotes but not in eukaryotes. D) Molecules in host cell membranes bind regulatory proteins that will stimulate the binding of C3b molecules to bacterial cells. E) Molecules in host cell membranes bind any MACs, deactivating them and preventing the activation of the alternative pathway.

B) Molecules in host cell membranes bind regulatory proteins that will inactivate any C3b molecules that attach to the membrane.

The four cardinal signs of inflammation are: A) Flare, wheals, fever, cough B) Redness, heat, swelling, pain C) Rash, pus, heat, rubor D) Heat, pain, vesicles, fever E) Heat, pain, redness, fainting

B) Redness, heat, swelling, pain

The oxygen consuming property of a wastewater sample is designated by the term biological nitrogen demand. lagooning. lagooning ANF stabilization. stabilization. biochemical oxygen demand.

BOD - biochemical oxygen demand

Staphylococcus aureus

BOILS gram + positive cocci in clusters bacteria facultative anaerobe withstands salt & high temp antibiotic treatment virulence factors: *coagulase* coagulates blood plasma *hyaluronidase* digests host CT *hemolysins* lyse RBCs diseases: *furncle* boils *carbuncle* several boils joined together *impetigo* bunch of skin bumps *osteomyelitis* bones *bacteremia* *endocarditis* *food intoxication* *scalded skin syndrome* *toxic shock syndrome*

Treponema and Borrelia

BOTH SPIROCHETES

Presenters on the morning news channel that you watch alert consumers to a food recall. A particular brand of ice cream has tested positive for Listeria monocytogenes, an organism that causes asymptomatic or relatively mild disease in otherwise healthy individuals, but can be problematic in pregnant women. Your sister is expecting her first child, so you call her immediately and tell her about the recall. You fill her in on what you know about this organism and the disease it causes, having just learned about it yourself in your pre-nursing microbiology class. Your sister has questions that you are able to answer. Why is listeriosis so important in pregnant women even though it usually causes them few symptoms? Listeriosis causes significant signs and symptoms in pregnant women and may prove fatal because they cannot take medication while pregnant. Bacteremia commonly occurs, resulting in infection of the fetus; miscarriage, stillbirth, or infection of the newborn can result. Listeriosis suppresses the woman's immune system; she fails to make IgG that would protect her fetus. Listeriosis in a pregnant woman usually leads to untreatable meningitis. Neither she nor her fetus is able to survive the infection. Infants born with listeriosis are sensitized against the foods in which the causative agent is found, such as soft cheeses. They can never eat these foods.

Bacteremia commonly occurs, resulting in infection of the fetus; miscarriage, stillbirth, or infection of the newborn can result.

What is the difference between 'bacteremia' and 'septicemia?' Bacteremia is an infection with bacteria. Septicemia is an infection with Septic protozoans. Bacteremia is the presence of living, multiplying bacteria in the bloodstream. Septicemia is the presence of endotoxins, but not necessarily of living microbial agents. Septicemia is the presence of living, multiplying bacteria in the bloodstream. Bacteremia is the presence of endotoxins, but not necessarily of living microbial agents. There is no difference-both terms denote the presence of living bacterial cells in the bloodstream.

Bacteremia is the presence of living, multiplying bacteria in the bloodstream. Septicemia is the presence of endotoxins, but not necessarily of living microbial agents.

Other microorganisms besides coliforms that have been used as indicators of fecal contamination may be all of the following EXCEPT Bacteroides. Clostridia, Enterococci AND bacteriophages. Clostridia. bacteriophages. Enterococci.

Bacteroides. p.794

Brucellosis may also be known as Bang's disease. undulent fever. Hansen's disease. rabbit fever. Bang's disease AND undulent fever.

Bang's disease AND undulent fever. p. 676 - Because Bang discovered the cause of cattle brucellosis and the fevers are rucurrent over weeks or months.

Cat Scratch Disease

Bartonella gram - bacteria

You are looking after your sister's cat Singa. While playing one evening, Singa accidentally bites you instead of the toy mouse you are holding. The bite wound is very small but is in the fleshy part of your hand. You wash the wound, which is not bleeding much, and put a band aid on it. The next day, your whole hand is red, swollen, and painful, and some lymph nodes in your arm are enlarged and tender. You call your sister, who is an RN. She urges you to go to the doctor, telling you that animal bites can sometimes be infected, and that if that is the case, you will need treatment before the infection gets any worse.The PA tells you that a disease called cat scratch disease can also be contracted through a cat bite. The organism that causes this disease is ________, and that the disease is characterized by ________. Streptobacillus moniliformis; vomiting, diarrhea and rash Pasteurella multocida; rash and joint pain Bartonella henselae; local lymph node enlargement Pasteurella multocida; local lymph node enlargement Bartonella henselae; peliosis hepatis

Bartonella henselae; local lymph node enlargement

Above this is all correct (I know because they were mine.)

Below - did my best to make sure.

A good range of temperature to store food is A. Below 4º C and above 60º C B. 20º-80º C C. Below -10º C and above 100º C D. 0º-50º C E. Below 10ºC and above 80º C

Below 4º C and above 60º C

What is the difference between biostimulation and bioaugmentation? Biostimulation is a bioremediation process designed to remove pollutants from an environment. Bioaugmentation is a method used in laboratories to promote the growth of fastidious microbes. Biostimulation generally occurs in situ and is less disruptive while bioaugmentation is performed off site using a bioreactor, a large tank designed to accelerate microbial processes. Bioaugmentation enhances the growth of local microbes in a contaminated site by providing additional nutrients while biostimulation relies on activities of microorganisms added to the contaminated material, complementing the resident population. Biostimulation is a method of degrading environmental pollutants while bioaugmentation is a method of supplementing microbial populations in landfills and composters. Biostimulation enhances the growth of local microbes in a contaminated site by providing additional nutrients while bioaugmentation relies on activities of microorganisms added to the contaminated material, complementing the resident population.

Biostimulation enhances the growth of local microbes in a contaminated site by providing additional nutrients while bioaugmentation relies on activities of microorganisms added to the contaminated material, complementing the resident population.

Which of the following about bacterial cystitis is false? a) About one-third of all women will have it at some time during their life. b) Catheterization of the bladder markedly increases the risk of contracting the disease. c) Individuals who have a bladder catheter in place indefinitely risk bladder infections with multiple species of intestinal bacteria at the same time. d) Bladder infections occur as often in men as they do in women.

Bladder infections occur as often in men as they do in women. SO MUCH more common in women.

Your brother is an enthusiatic hiker. He returns from a walking vacation and tells you that although he had a good time, he ran into a bit of trouble on one of the days he was away because he had got lost in the woods for several hours. Although he took enough food for the day, your brother ran out of water. Luckily, he came across a small stream and was able to fill up his water bottle and to finish his hike without any further problems. Your brother mentions as an aside that he must have eaten something bad while he was away, because he has had pretty bad diarrhea since he got back. He tells you he doesn't feel too bad, but that he has been having what he describes as "explosive diarrhea." He also complains of abdominal pain. You urge him to go to the doctor because you suspect that he may have something more than just food-related diarrhea. The signs and symptoms of giardiasis vary among people. Which of the following is your brother unlikely to experience with his infection? Bloody, frothy urine Jaundice and malabsorption Indigestion and gas These all may be experienced with giardiasis. Nausea and vomiting

Bloody, frothy urine

The causative organism of whooping cough is: A. Streptococcus pyogenes B. Staphyloccus aureus C. Streptococcus pneumoniae D. Bordetella pertussis E. Haemophilus influenzae

Bordetella pertussis

Lyme disease borreliosis

Borrelia gram - bacteria

You go hiking in the woods with some of your classmates. A few days after the hike, your friend Alice calls to tell you she is not feeling well and that she is going to skip soccer practice that evening. She says she has a weird rash on her leg and wonders if she was bitten by a spider while your were hiking. She sends you a photo of the rash. You are in your final semester of nursing school and have recently studied disease of the cardiovascular system. You think that the rash that Alice has is a sign of Lyme disease, and offer to take her to the urgent care to get medical help. While you are waiting to see a clinician, you review answer Alice's questions about Lyme disease. Lyme disease is a zoonosis with humans being an accidental host. What does this mean? Borrelia bergdorferi is transmitted to animals from humans AND the bacterium cannot complete its life cycle in humans. Borrelia bergdorferi is found predominantly in zoo animals AND the tick vector cannot complete its life cycle in humans. Borrelia bergdorferi is found predominantly in zoo animals AND the bacterium cannot complete its life cycle in humans. Borrelia bergdorferi is found predominantly in zoo animals AND humans are not the normal host for the tick vector. Borrelia bergdorferi is transmitted to humans from animals AND humans are not the normal host for the tick vector.

Borrelia bergdorferi is transmitted to humans from animals AND humans are not the normal host for the tick vector. Correct Borrelia burgdorferi - bacteria- Gram negative, microaerophilic spirochete with many plamsids and a chromosome that is linear and present in multiple copies

The causative agent of Lyme disease is: a. Rickettsia rickettsi. b. Rickettsia prowazeki. c. Borrelia burgdorferi. d. Dermacentor andersoni.

Borrelia burgdorferi. Borrelia burgdorferi: large, Gram-negative, microaerophilic spirochete with multiple copies of linear chromosome - Also contains numerous circular and linear plasmids with genes usually found on bacterial chromosomes

Select the TRUE statement regarding staphylococci and staphylococcal wound infections. Staphylococcus epidermidis produces several virulence factors, including exotoxins. Some S. epidermisis strains produce superantigens that activate many helper T cells, causing a cytokine storm. Staphylococcus aureus forms biofilms but lacks virulence factors and is not very pathogenic. S. aureus and S. epidermidis are both coagulase-positive, encapsulated organisms. Both HA-MRSA and CA-MRSA are resistant to multiple antibiotics, including sulfa drugs and tetracyclines.

Both HA-MRSA and CA-MRSA are resistant to multiple antibiotics, including sulfa drugs and tetracyclines.

You are looking after your sister's cat Singa. While playing one evening, Singa accidentally bites you instead of the toy mouse you are holding. The bite wound is very small but is in the fleshy part of your hand. You wash the wound, which is not bleeding much, and put a band aid on it. The next day, your whole hand is red, swollen, and painful, and some lymph nodes in your arm are enlarged and tender. You call your sister, who is an RN. She urges you to go to the doctor, telling you that animal bites can sometimes be infected, and that if that is the case, you will need treatment before the infection gets any worse.You are given a prescription for both a penicillin derivative AND a β-lactamase inhibitor. Why are you given these medications? Both P. multicoda and B. henselae produce β-lactamases. They are thus treated with β-lactamase-inhibiting medications. In addition, skin microbiota such as S. epidermidis respond to penicillins. Both P. multicoda and B. henselae respond to penicillin derivatives. In addition, some bite wounds are infected with strains of β-lactamase-producing Streptobacillus moniliformis. The causative organisms of cat bite infections and bartonellosis are not well characterized. For this reason, penicillin derivatives and inhibitors of these medications are given synergistically. Both P. multicoda and B. henselae respond to penicillin derivatives. In addition, some bite wounds are infected with strains of β-lactamase-producing Staphylococcus aureus. Both P. multicoda and B. henselae are encapsulated, so respond well to penicillin derivatives. In addition, some bite wounds are infected with strains of β-lactamase-producing Staphylococcus epidermidis.

Both P. multicoda and B. henselae respond to penicillin derivatives. In addition, some bite wounds are infected with strains of β-lactamase-producing Staphylococcus aureus. p.616 Correct

You are cleaning out your pantry and find that your bread has spoiled. Which organism caused the spoilage, and why? Bread is spoiled by bacteria because it does not have enough water available for mold growth. Pseudomonas aeruginosa—this bacterium is the primary cause of spoilage in grain products. Bread is spoiled by mold because it does not have enough water available for bacterial growth. Bread is spoiled by both bacteria and mold simultaneously—these organisms always act synergistically in spoilage. Bread is spoiled by mold because there is too much water present for bacterial growth.

Bread is spoiled by mold because it does not have enough water available for bacterial growth.

Which of the following is/are able to survive phagocytosis? Brucella sp. Staphylococcus aureus Francisella tularensis Mycobacterium tuberculosis Brucella sp, Francisella tularensis AND Mycobacterium tuberculosis

Brucella sp, Francisella tularensis AND Mycobacterium tuberculosis Brucellosis - Brucella sp. prevent phagosome-lysosome fusion Tularemia - Francisella tularemia escape from the phagosome TB - Mycobacterium tuberculosis - mycolic acids in the bacterial cell wall prevent phagosome-lysosome fusion, bacteria leaves the phagosome, multiplies in cytoplasm. p.553

Meliodosis

Burkholderia gram - bacteria

How does F. tularensis avoid the immune system? By escaping from the macrophage phagolysosome AND by producing streptokinases. By having altered surface components AND by destroying lymphocytes. By synthesizing a large polysaccharide capsule AND by escaping the macrophage phagolysosome. By having altered surface components AND by escaping the macrophage phagolysosome. By synthesizing a large polysaccharide capsule AND by degrading complement component C3b.

By having altered surface components AND by escaping the macrophage phagolysosome. p.676 Correct

Why would the use of a known, reliable starter culture improve the safety of fermented meat products? a) We need to be very careful which microbes are introduced into a food product. Some microbes can interact with microbes already present in the food, producing toxins and toxic byproducts that might harm human beings. b) By using pure starter cultures, we KNOW what we're adding in, and how much. We can control the microbial fermentation precisely to best prepare the food product. c) It doesn't. All fermented meat products are inherently unsafe since we can't determine what fermentation products are left behind in the food. Some of them may be toxic. d) It doesn't. Every fermenting microbe ferments the same way, producing the same products. Whatever we start with, the outcome is the same.

By using pure starter cultures, we KNOW what we're adding in, and how much. We can control the microbial fermentation precisely to best prepare the food product.

All of the following are true of acute necrotizing ulcerative gingivitis (ANUG) except: A.Most destructive of the periodontal diseasesB.Involves Treponema spirochete and other anaerobic bacteriaC.Very communicableD.Associated with severe pain, bleeding, abscessed gums, and necrosisE.Due to poor oral hygiene, altered host defenses, or prior gum disease

C

Please select the correct sequence for the steps of phagocytosis: 1. Conditions in the phagosome change, increasing its antimicrobial activities. 2. The phagocytic cell binds microbial invaders and engulfs them, internalizing them in a phagosome. 3. Phagocytic cells are recruited to the site of an injury by chemoattractants. 4. Within the phagolysosome, various factors work together to destroy an engulfed invader. 5. The phagosome fuses with enzyme-filled lysosomes, forming a phagolysosome. A) 1, 2, 3, 4, 5 B) 2, 4, 3, 5, 1 C) 3, 2, 1, 5, 4 D) 1, 3, 5, 2, 4 E) 1, 5, 2, 4, 3

C

Which of the following cytokines is most antiviral in its action? A) Interleukin-1 B) Interleukin-2 C) Interferon D) Tumor necrosis factor E) Colony-stimulating factor

C

Which of the following is a viral hemorrhagic fever? A. Cat Scratch fever B.Rocky Mountain spotted feverC. Yellow fever D.Rabbit feverE.Q fever

C

Which of the following locations pertaining to the GI tract has a large commensal population of microorganisms? A. Liver B.PancreasC.Large intestineD.Small intestineE.Salivary glands

C

Your first patient of the day is complaining of pain in the area of his new tattoo. When you examine the tattoo, you note that the area around it is firm, red, and swollen. There is no evidence of pus, and your patient has a normal pulse, and no fever. You tell your patient that you have seen this type of response to a tattoo before, and that it is called a granulomatous reaction. You give him more information on the condition. Your patient doesn't understand all of the cell names that you have used. You educate him on the role of each cell type. One cell type that plays a role in adaptive immunity rather than innate immunity. Which cell is this? A) Neutrophils—type of phagocytic cell in the blood that quickly move to infected tissues, where they engulf and destroy invading microbes. B) Macrophages—type of phagocytic cell that resides in tissues, engulfing invaders and debris, and producing pro-inflammatory cytokines. C) Dendritic cells—antigen-presenting cells that "show" antigens to T cells, playing an essential role in their activation. D) Eosinophils—primary role is to rid the body of parasitic worms; also involved in allergy. E) Mast cells—tissue cells that are important in the inflammatory response and immediate allergic reactions.

C

Each of the following is true for Listeria monocytogenes except: A.Resistant to cold B.Possesses flagella C.Penetrates alveolar macrophages D.Can result in septicemia E.Can be controlled by viral lysis

C p703 - Listeria monocytogenes is a motile, non-spore forming, facultatively aerobic G+ rod that can grow at 4 C (in your fridge) A yes, B yes (I put B and got wrong), C = gets in by GI tract so probably C, D. Does enter bloodstream so perhaps and E. they spray bacteriophages during packaging to lyse the bacteria so is true.

Phagocytes were first discovered and named by A) Pasteur. B) Koch. C) Metchnikoff. D) Lister. E) Hooke.

C) Metchnikoff. - he was studying starfish larvae

Which of the following are phagocytic cells found in the human body? A) Erythrocytes AND neutrophils B) Neutrophils AND megakaryocytes C) Neutrophils AND macrophages D) Megakaryocytes AND leukocytes E) Macrophages AND erythrocytes

C) Neutrophils AND macrophages - erythrocytes are red blood cells, megakaryocytes are where platelets come from, leukocytes means all the white blood cells and not all WBCs are phagocytes.

Which statement about complement proteins is FALSE? A) They may be activated through three different pathways. B) They disrupt the cytoplasmic membrane of invading bacteria and foreign cells. C) They are part of the specific immune defense system. D) They are a group of blood proteins produced by the liver. E) They are part of the innate immune response.

C) They are part of the specific immune defense system.

The presence of long double-stranded RNA (> 30 bp): A) indicates infection by any virus. B) indicates exposure to mutagens. C) induces synthesis of interferon. D) indicates infection by a retrovirus. E) indicates complement activity.

C) induces synthesis of interferon.

Which of the following mimics the infection caused by Neisseria gonorrhea? C. trachomatis M. pneumonia T. pallidum E. coli

C. trachomatis Chlamydial infections are the most common bacterial STIs. Mostly asymptomatic, signs and symptoms similar, infertility, PID, neonatal conjunctivitis/pneumonia - just like with gonorrhea.

Which of the following is/are obligate intracellular parasites?

CHLAYMYDIA & RICKETTISA

A physician is attempting new therapies for HIV patients who are suffering from an impaired immune response. He decides to try using a recombinant form of colony-stimulating factor cytokine (CSF). Why? - CSF is a strong inducer of antiviral activities in our cells, and may help our immune system fight off the effects of an HIV infection for a longer period of time. - CSF will drive up the production of lactoferrin and transferrin, strong antiviral compounds produced in our mucus membrane secretions that trap and destroy viruses. - CSF is a cytokine that allows cells to communicate. If levels of CSF are elevated, HIV-infected cells can alert other healthy cells so that they produce protective IFNs. - CSF will help to stimulate the production of new lymphocytes, the very cells that are affected by HIV. This may help to keep the patients' immune responses "normal" for a period of time. - CSF will hyperstimulate the activities of the macrophages, leading to aggressive ingestion and successful destruction of HIV-infected cells.

CSF will help to stimulate the production of new lymphocytes, the very cells that are affected by HIV. This may help to keep the patients' immune responses "normal" for a period of time.

earliest oxygenic phototrophs are thought to be

CYANOBACTERIA

Chicken shish kebab prepared on a camping trip: Staphylococcus aureus Vibrio parahaemolyticus E. coli 0157:H7 Listeria monocytogenes Campylobacter

Campylobacter

Foodborne intestinal disease

Campylobacter gram - bacteria

A young mother brings her three-month old baby into the doctor's office where you work as an RN. The baby is miserable and the mother reports that he has developed a fever, which is why she has brought him to the doctor. She tells you that the child is particularly upset when she changes his diaper and that he seems to have diaper rash, which she attributes to the fact that her son's diaper isn't always changed promptly enough at the day-care facility he goes to daily. In fact, she has recently switched him to another facility. You remove the child's diaper and immediately see that he has a very red rash on his buttocks and genital area. There is also red, scaly skin in the area where his diaper touches his thighs. You suspect that this may be more than just a typical case of diaper rash. Diaper rash (dermititis) can be caused by a number of factors. A common microbial cause is the fungus ________. A) Malassezia furfur B) Candida albicans C) Microsporum gypseum D) Cutibacterium acnes E) Streptococcus pyogenes

Candida albicans

A young mother brings her three-month old baby into the doctor's office where you work as an RN. The baby is miserable and the mother reports that he has developed a fever, which is why she has brought him to the doctor. She tells you that the child is particularly upset when she changes his diaper and that he seems to have diaper rash, which she attributes to the fact that her son's diaper isn't always changed promptly enough at the day-care facility he goes to daily. In fact, she has recently switched him to another facility. You remove the child's diaper and immediately see that he has a very red rash on his buttocks and genital area. There is also red, scaly skin in the area where his diaper touches his thighs. You suspect that this may be more than just a typical case of diaper rash. In the laboratory, rubella virus, VZV, rubeola virus and Rickettsia rickettsii must be cultivated in host cells. Candida albicans does not need to be cultured in cells—why? A) Candida albicans is not an obligate intracellular parasite. B) This fungus is an aquatic organism that requires fresh water for culture. C) Candida species are yeasts and require a medium such as bread for culture. D) Candida albicans is an obligate intracellular parasite. E) Yeasts are autotrophs and are thus able to grow independent of other organisms.

Candida albicans is not an obligate intracellular parasite. Nope, it's a yeast - a fungus.

Which statement regarding canning and pasteurization is TRUE? Canning and pasteurization are essentially the same process—one is used on fresh food while the other is used on preserved food; both result in sterilization of the product. Pasteurization uses a process that destroys all microorganisms that can grow under normal storage conditions; the resulting product is commercially sterile. Canning significantly decreases the number of spoilage microbes and eliminates pathogens; however, viable organisms still remain. Canning and pasteurization both use a process that destroys all microorganisms that can grow under normal storage conditions; the resulting products are commercially sterile. Canning and pasteurization both significantly decrease the number of spoilage microbes and eliminates pathogens; however, viable organisms still remain. Canning uses a process that destroys all microorganisms that can grow under normal storage conditions; the resulting product is commercially sterile. Pasteurization significantly decreases the number of spoilage microbes and eliminates pathogens; however, viable organisms still remain.

Canning uses a process that destroys all microorganisms that can grow under normal storage conditions; the resulting product is commercially sterile. Pasteurization significantly decreases the number of spoilage microbes and eliminates pathogens; however, viable organisms still remain. WRONG!!! Pasteurization uses a process that destroys all microorganisms that can grow under normal storage conditions; the resulting product is commercially sterile. Canning significantly decreases the number of spoilage microbes and eliminates pathogens; however, viable organisms still remain.

Which of the following pertains to typhoid fever? A) It is acquired via ingestion of contaminated meat. B) Enterotoxin spreads via the blood. C) It is caused by several different species of Salmonella. D) Causative microorganism multiplies in patient phagocytes. E) It is also called bacillary dysentery

Causative microorganism multiplies in patient phagocytes. - colonize the intestines, cross the mucus membrane via M cells, MULTIPLIES INSIDE MACROPHAGES and then are carried in the bloodstream to locations body-wide.

Why can an infection in the brain's ventricles usually be detected in spinal fluid obtained from the lower back (lumbar region)? A. Cerebrospinal fluid (CSF) originates in the ventricles, but percolates over and around the brain and spinal cord. B. There is a high degree of vascularity that exists, allowing easy transfer of bacteria in the cerebrospinal fluid (CSF) and the blood. C. There is a set of lymph nodes specifically in place to drain the cerebrospinal fluid (CSF) from the ventricles. They are housed in the lumbar region of the spinal column, adjacent to the spinal cord. D. This is an incorrect statement. Since the central nervous system (CNS) is a protected site due to the blood-brain barrier, there can never BE an infection in the ventricles.

Cerebrospinal fluid (CSF) originates in the ventricles, but percolates over and around the brain and spinal cord.

Why can an infection in the brain's ventricles usually be detected in spinal fluid obtained from the lower back (lumbar region)? When ventricles are infected, the brain reacts by pumping the causative pathogens into the CFS so that they can be removed by the immune system. There is a set of lymph nodes specifically in place to drain the cerebrospinal fluid (CSF) from the ventricles. They are found in the lumbar region of the spinal column. There is a high degree of vascularity that exists, allowing easy transfer of bacteria in the cerebrospinal fluid (CSF) and the blood. Cerebrospinal fluid (CSF) originates in the ventricles, but percolates over and around the brain and spinal cord. This is an incorrect statement. Since the central nervous system (CNS) is a protected site due to the blood-brain barrier, there can never BE an infection in the ventricles.

Cerebrospinal fluid (CSF) originates in the ventricles, but percolates over and around the brain and spinal cord.

The paroxysms that occur in cyclic 48 to 72 hour episodes in a malaria patient are: A. Urinary frequency and pain and vaginal discharge B. Bloody, mucus-filled stools, fever, diarrhea and weight loss C. Fever, swollen lymph nodes and joint pain D. Chills, fever, and sweating from erythrocytic lysis E. Sore throat, low grade fever and swollen lymph nodes

Chills, fever, and sweating from erythrocytic lysis Cold, Hot and Wet stages.

Which of the following contains bacteriochlorophyll?

Chromatium, Thiospirillum, Thidictyon.

Two people are brought to the Emergency Department where you work. The first patient reports fever, abdominal pain, vomiting, and diarrhea. The second patient appears extremely ill. Her family tells you that she also had gastrointestinal signs and symptoms a few days ago. She has now developed severe skin blistering, has a very high fever and is only semi-conscious. When you question the first patient and the family of the second patient, you discover that they know each other and that they have all recently returned from a beach vacation in a major city on the U.S. Gulf Coast. The group tells you that the enjoyed a raw oyster dinner on the last day of their summer vacation. They think that both patients are probably just allergic to seafood, but given the severity of the signs and symptoms of your patients, you suspect that they may have Vibrio vulnificus infection. Lab tests confirm your suspicion. You give the families information on this disease.You find out that the seriously ill patient has an underlying condition. Which of the following likely predisposed her to the more severe infection? Cystitis Asthma Glomerulonephritis Osteoporosis Cirrhosis

Cirrhosis Liver problem. p.674 Correct

Which of the following is/are treatment(s) for advanced periodontal disease? Cleaning out gingival crevices, minor gum surgery, AND antibiotic therapy. Cleaning out gingival crevices AND removing plaque and tartar. Removing plaque and tartar. Brushing and flossing, filling cavities with amalgam, AND antibiotic therapy. Removing plaque and tartar AND minor gum surgery.

Cleaning out gingival crevices, minor gum surgery, AND antibiotic therapy.

Other microorganisms besides coliforms that have been used as indicators of fecal contamination may be A. Clostridia. B. Enterococci. C. bacteriophages. D. Bacteroides. E. Clostridia, Enterococci AND bacteriophages

Clostridia, Enterococci AND bacteriophages - but NOT bacteroides.

Bacteria that may form endospores include

Clostridium and Bacillus

A 37-year-old man is admitted to the hospital where you work with dizziness, dry mouth, blurred vision, and slurred speech. In addition, he reports abdominal symptoms, including pain, nausea, vomiting, and diarrhea. The patient's family tells you that he was recently at a church potluck, where he ate bread, streak, homemade tomato-beef stew and home-canned green beans. Based on the patient's signs and symptoms, the physician with whom you are working suspects botulism, and the patient is immediately given botulinum antitoxin. He develops breathing difficulty and is placed on a mechanical ventilator.A stool sample from the patient is cultured, and confirms the presence of Clostridium botulinum. In addition, the stool sample as well as leftover stew and green beans are tested for botulinum toxin—toxin is present in the green beans and in the stool sample, but not in the stew. These results confirm the diagnosis of foodborne botulism. You are one of the RNs taking care of the patient—he remains in the hospital for several weeks, before being discharged. Please select the statement that does NOT apply to botulism. The causative agent is an obligate anaerobe. Vegetative Clostridium cells release botulinum toxin, a potent neurotoxin. Boiling home-canned foods for 10 minutes will destroy any botulinum toxin present. The causative agent is Clostridium botulinum. Clostridium endospores can germinate in food with an acidic pH.

Clostridium endospores can germinate in food with an acidic pH. Not true - this is why acidic foods don't need such stringent canning procedures - Clostridium endospores are inhibited by low/acidic pH. Clostridium botulinum - Gram positive, anaerobe, motile, endospore former.

Enterobacteriaceae Family

Coliforms; ferment lactose Large family of small, non-spore-forming Gram-negative rods Many members inhabit soil, water, decaying matter, and are common occupants of large bowel of animals including humans. Most frequent cause of diarrhea through enterotoxins Enterics, along with Pseudomonas sp., account for almost 50% of nosocomial infections.

You go for your regular Pap test and are informed by your OBGYN that you are positive for HPV16. You are surprised because you have not noticed any signs or symptoms of infection. Your doctor explains that sexually transmitted HPV strains are among the most common of the STI agents, and that while some HPV strains cause warty growths of the external and internal genitalia, other strains cause non-warty lesions of mucosal surfaces such as the uterine cervix. These strains are of concern because they are a major factor in the development of cervical cancer. You ask your doctor a number of questions regarding HPV and HPV infection. I know HPV is sexually transmitted, but my male partner knows he is HPV positive, so always wears a condom whenever we have sex. I am not sure how I contracted this virus. How can this be explained? - Condoms are never an effective way of preventing transmission of viruses. People who avoid HPV infections take daily doses of prophylactic antibiotics as long as they are sexually active. - Condoms are never an effective way of preventing transmission of viruses. People who avoid HPV infections take daily doses of antiviral medications as long as they are sexually active. - Condoms do not provide complete protection against HPV because the virus can be transmitted by exposure to areas not covered by the condom. - Only latex condoms prevent the transmission of HPV because there are medications in the latex that destroy viruses. Other types of condoms lack these medications. - Condoms prevent the transmission of bacterial pathogens but do not protect against viral diseases because viruses are small enough to get through the latex.

Condoms do not provide complete protection against HPV because the virus can be transmitted by exposure to areas not covered by the condom.

Giardia intestinalis is found worldwide and is one of the most common intestinal parasites in the U.S. Which of the following factors contributes to the high incidence of infection, particularly in urban and suburban settings in the U.S? Contamination of fruits and vegetables Ability to infect a wide range of mammals Extremely resistant cysts Contamination of public drinking water sources

Contamination of public drinking water sources?? Giardia does have cysts that survive stomach acid... Contamination of fruits and veggies is Cyclosporiasis. wide host range is Cryptospiridium.

Corneal implants have been implicated in a few cases of: A.botulism. B. trypanosomiasis. C. Creutzfeldt-Jakob. D. scrapie.

Creutzfeldt-Jakob prion disease, a type of Transmissible Spongiform Encephalophathy. Most cases occur as Creutzfeldt-Jakob disease (CJD). - occurs in people over 45, median age 28 - has been transmitted from human to human through corneal transplants, contaminated surgical instruments and injections of human hormone replacements.

Which of the following statements regarding microbial identification is false? Culture techniques are an accurate way of determining which members in a microbial community are most common. Studying the genome of one organism can give insights into the characteristics of another organism. Metagenomics involves comparison of gene sequences in DNA from environmental samples to known gene sequences in databases. FISH can be used to distinguish subsets of prokaryotes that contain a specific nucleotide sequence. 16S rRNA sequences can be used to determine taxonomic similarity among prokaryotes.

Culture techniques are an accurate way of determining which members in a microbial community are most common.

Each of the following is associated with pneumonic plague except: A.Can progress to a septicemiaB.Patient has fever, headache, nausea, pneumoniaC. Transmitted by respiratory droplets D.Severe internal hemorrhagingE.Caused by Yersinia pestis

D

Which is incorrect about warts? A.Are transmitted by direct contact or fomitesB.Include deep plantar warts of soles of the feetC.Freezing and laser surgery can be used for removalD.Are frequently cancerousE.Caused by human papillomaviruses (HPV)

D

Your first patient of the day is complaining of pain in the area of his new tattoo. When you examine the tattoo, you note that the area around it is firm, red, and swollen. There is no evidence of pus, and your patient has a normal pulse, and no fever. You tell your patient that you have seen this type of response to a tattoo before, and that it is called a granulomatous reaction. You give him more information on the condition. Your patient wonders what giant cells are and how they form. You explain that giant cells: A) are very large cells with a single nucleus; they are formed by the fusion of many neutrophils during a chronic inflammatory response. B) are macrophages that have large extensions in their cytoplasmic membranes which both increases their size and enhances their phagocytic ability. C) are any cells in body that have a diameter of greater than 10-3 m. D) are very large cells with multiple nuclei; they are formed by the fusion of many macrophages during a chronic inflammatory response. E) cells that have engulfed a large number of particles that get stored inside the cell, causing it to absorb water and swell.

D

Your sister recently had her first child, a boy who she names Owen. Unfortunately, Owen seems to get sick really often, and after some investigation, he is diagnosed with an extremely rare genetic disorder called leukocyte adhesion deficiency (LAD). People with LAD suffer from repeated bacterial infections because their leukocytes have adhesion defects—they cannot adhere to the endothelium and as a result are unable to leave the blood vessels to move into tissues containing microbial invaders. Without this inflammatory response, the person experiences severe tissue infections and frequently has neutrophilia (high levels of neutrophils in the blood). You help your sister understand this disease, using your knowledge of the immune system. Which of the following do you think would be a possible way to cure a person with LAD? A) Giving the person inflammatory mediators daily to enhance the vasodilation process during inflammation. B) Giving the person a single blood transfusion with blood containing healthy leukocytes. C) Keeping the person on multiple antimicrobial medications for their whole life to fight any potential infections. D) Giving the person a hematopoietic stem cell transplant—this would replace the person's defective leukocytes with healthy leukocytes. E) Keeping the person in a completely microbe-free environment so that they are never at risk of contracting a microbial infection.

D

Please identify the INCORRECT definition regarding innate immunity. A) First line defenses—the barriers that separate and shield the interior of the body from the surrounding environment (skin and mucous membranes). B) Sentinel cells—host cells that help the immune system detect signs of invasion by producing cytokines when their PRRs recognize an invading microbe. C) Inflammatory response—coordinated innate response involved in containing a site of damage, localizing the response, eliminating the invader, and restoring tissue function. D) Innate lymphoid cells—a group of lymphocytes that have increased specificity in their mechanism of antigen recognition (example: natural killer (NK) cells). E) RIG-like receptors—a group of pattern recognition receptors within cells that recognize molecules associated with infecting viruses.

D) Innate lymphoid cells—a group of lymphocytes that have increased specificity in their mechanism of antigen recognition (example: natural killer (NK) cells). Natural killer cells, or NK cells, are a type of cytotoxic lymphocyte critical to the innate immune system. ... NK cells are known to differentiate and mature in the bone marrow, lymph nodes, spleen, tonsils, and thymus, where they then enter into the circulation. RIG-I-like receptors are a type of intracellular pattern recognition receptor involved in the recognition of viruses by the innate immune system.

Your patient had abdominal surgery three days ago. She seemed to be doing well after the surgery, but today the skin around the surgical incision is red and swollen. In addition, your patient has a high fever and is complaining of muscle aches. She also has a rash and has diarrhea. You realize that her wound has become infected, despite your best efforts as a nurse, and you are concerned that she has developed toxic shock syndrome. You explain the details of this type of infection to the patient's family.You explain general mechanisms of antibiotic resistance to your patient's family. Which of the following mechanisms is not one used by S. aureus? A) Some bacteria are able to make minor structural changes in the cellular target of a drug. This can prevent the medication from binding to that target, thereby protecting the organism from its effects. B) Some bacteria produce enzymes that chemically modify a specific medication, interfering with its function. An example is the β-lactamases that inactivate different β-lactam antibiotics. C) Some bacteria use efflux pumps to transport antimicrobials and other damaging compounds out of the cell. Sometimes the pumps are structurally altered, conferring resistance to several different antimicrobials simultaneously. D) Some bacteria can make changes in porin proteins of the outer membrane and can therefore prevent certain antimicrobials from entering the cell's cytoplasm. By stopping entry of an antimicrobial, an organism avoids its effects. E) Resistance in S. aureus may involve any of these mechanisms.

D) Some bacteria can make changes in porin proteins of the outer membrane and can therefore prevent certain antimicrobials from entering the cell's cytoplasm. By stopping entry of an antimicrobial, an organism avoids its effects. Used by G-. S. aureus is G+.

How do some pathogens prevent complement activation or avoid the effects of activated complement? A) Some pathogens produce C5a peptidase that destroys complement C5a, a chemoattractant that recruits phagocytes to the area of infection. B) Some pathogens hijack the host's regulatory membrane proteins that inactivate C3b, preventing the triggering of the alternative activation pathway. C) Some pathogens produce C5a peptidase that causes destruction of phagoctyes recruited to the area of infection. D) Some pathogens hijack the host's regulatory membrane proteins that inactivate C3b, preventing the triggering of the alternative activation pathway AND some pathogens produce C5a peptidase that destroys complement C5a, a chemoattractant that recruits phagocytes to the area of infection. E) Some pathogens hijack the host's regulatory membrane proteins that inactivate C3b, preventing the triggering of the alternative activation pathway AND some pathogens produce C5a peptidase that causes destruction of phagoctyes recruited to the area of infection.

D) Some pathogens hijack the host's regulatory membrane proteins that inactivate C3b, preventing the triggering of the alternative activation pathway AND some pathogens produce C5a peptidase that destroys complement C5a, a chemoattractant that recruits phagocytes to the area of infection.

Syphilis, an STI, was once treated by intentionally infecting the patient with the parasite that causes malaria, a disease characterized by repeated boutsof fever, shaking,andchills. Why might this treatment cure syphilis? A) Malaria parasites produce strong antibacterial compounds (since they're eukaryotic in nature-they are trying to eliminate their competition for resources). This helps to eliminate ALL bacteria in and on the human body for a short period of time. B) Malaria parasites track down and feed upon ALL bacterial cells in the human body as a part of their life cycle. This makes them a "natural antibiotic" of sorts, and highly effective at clearing the bacterial infection of syphilis. C) One of the side effects of malarial infection is a massive overproduction of macrophages. This drives up the ability to ingest and destroy any microbe, including the bacterium that causes syphilis. D) The effect of driving up the body temperature for periods of time can shut down the temperature-sensitive replication of the bacterium that causes syphilis. This gives the immune system time to eliminate it properly. E) The bacteria that cause syphilis track down and destroy the protozoan that causes malaria. By doing this, the bacteria are "occupied" and do not cause damage to the human host.

D) The effect of driving up the body temperature for periods of time can shut down the temperature-sensitive replication of the bacterium that causes syphilis. This gives the immune system time to eliminate it properly.

Toll-like receptors: A) are cytokines, are embedded in cellular membranes, AND are part of adaptive immunity. B) each recognize a specific "danger" molecule AND are cytokines. C) are embedded in cellular membranes AND are part of adaptive immunity. D) each recognize a specific "danger" molecule AND are embedded in cellular membranes. E) each recognize a specific "danger" molecule AND are part of adaptive immunity.

D) each recognize a specific "danger" molecule AND are embedded in cellular membranes.

Which describes toll-like receptors? A) cytokines, embedded in cellular membranes, AND are part of adaptive immunity B) recognize a specific MAMP AND are cytokines C) embedded in cellular membranes AND are part of adaptive immunity D) each recognize a specific MAMP AND are embedded in cellular membranes E) each recognize a specific MAMP AND are part of adaptive immunity

D) each recognize a specific MAMP AND are embedded in cellular membranes

Which of the following statements about landfills is FALSE? Dangerous levels of methane gas can accumulate. The excavated site has a plastic liner to prevent wastes from leaching into groundwater. Degradation of wastes is rapid and inexpensive. Recycling greatly reduces the amount of wastes sent to landfills. All of the answer choices are true.

Degradation of wastes is rapid and inexpensive.

The major vector of Rocky Mountain spotted fever in the western U.S. is: a. Rickettsia rickettsi. b. Rickettsia prowazeki. c. Borrelia burgdorferi. d. Dermacentor andersoni.

Dermacentor andersoni. A is the bacteria that causes RMSF B is the cause of louse-borne typhus C is the cause of lyme disease D is the type of tick that carries the bacteria - the vector. Table 22.7 a zoonosis transmitted by the bite of an infected tick, usually Dermacentor species.

What type of algae are used as filters?

Diatoms are algae whose silicon dioxide-containing shells are useful economically as filters.

Normal microbiota: A) are the organisms that typically reside on and in your body. B) protect against infection by pathogens. C) enhance infection by pathogens. D) play no role in affecting pathogen growth. E) are the organisms that typically reside on and in your body AND protect against infection by pathogens.

E

Spongiform encephalopathies are: A.Associated with abnormal, transmissible, protein in the brainB.Chronic, fatal infections of the nervous systemC.Caused by prionsD.Creutzfeldt-Jacob disease, scrapie, and bovine spongiform encephalopathyE.All of the choices are correct

E

The normal biota of the CNS consists of: A. Neisseria meningitidis B. Herpes simplex I C. Herpes simplex II D.Streptococcus agalactiaeE. The CNS has no normal biota

E

What features of the respiratory system protect us from infection? A.Nasal hairsB.CiliaC.MucusD.MacrophagesE.All of the choices are correct

E

Which is incorrect about rabies? A.Wild populations of bats, skunks, raccoons, cats and canines are primary reservoirsB.Is a zoonotic diseaseC.Symptoms include anxiety, agitation, muscle spasms, convulsions, and paralysisD.Transmission can involve bites, scratches and inhalationE.Average incubation in human is 1 week

E

Which of the following are lymphocytes? A) B cells B) T cells C) NK cells D) B cells AND T cells E) B cells, T cells, AND NK cells

E

Your first patient of the day is complaining of pain in the area of his new tattoo. When you examine the tattoo, you note that the area around it is firm, red, and swollen. There is no evidence of pus, and your patient has a normal pulse, and no fever. You tell your patient that you have seen this type of response to a tattoo before, and that it is called a granulomatous reaction. You give him more information on the condition. Your patient questions why he has developed a granuloma. You explain to him that granulomas: A) are formed to recruit macrophages and neutrophils to an area of persistent inflammation such as that observed with a tattoo. B) result from activation of complement proteins; once complement C3 is activated, it causes a cascade of activation of C4 - C25. These activated proteins concentrate to form the granuloma. C) are immune cells that are normally synthesized to fight virus infections. In your patient's case, these cells were formed in response to ink particles. D) are proteins that recognize compounds unique to microbes or tissue damage; in your patient's case, these are made as a result to the tissue damage caused by the tattoo. E) are formed as an attempt by the body to wall off and contain persistent antigens - in his case, the red tattoo ink (red ink contains mercury compounds).

E

How do cytokines function? A) They are produced by several cell types and bind directly to microbes to enhance their chance of being ingested, in a process called opsonization. B) They are secreted by lysozomes into the phagosomes of neutrophils to enhance the killing of ingested microbes within those organelles. C) They are secreted by one cell type, then work as a series of serum proteins to produce holes in the cytoplasmic membranes of microbes to directly lyse them. D) They are secreted by microbes that have been engulfed by phagocytes, causing those cells to produce destructive enzymes and undergo apoptosis. E) They are secreted by one cell type, then bind to a receptor on target cell causing a signal within that cell that turns on (or off) certain genes to achieve a response.

E) They are secreted by one cell type, then bind to a receptor on target cell causing a signal within that cell that turns on (or off) certain genes to achieve a response.

The low molecular weight protein produced by animal cells in response to viral infections is: A) complement. B) lysozyme. C) histamine. D) interferon. E) transferrin.

E) transferrin

Steak tartare (minced raw beef): Staphylococcus aureus Vibrio parahaemolyticus E. coli 0157:H7 Listeria monocytogenes Campylobacter

E. coli 0157:H7

Pinworm

Enterobius helminths (worms)

Which of the following are mismatched? A. propionic acid-Swiss cheese B. brewer's yeast-S. carlbergensis C. glycocalyx-milk ropiness D. botulinum-neurotoxin E. starter culture-Pseudomonas

Estarter culture-Pseudomonas Wound and spoilage bacteria that turns things green as a starter culture? Just no. The point of the starter culture is to get the ball rolling for fermentation. B - that's the yeast Saccharomyces and this one is used in the production of Lager bear. (I don't drink alcohol, do I have to know this?!)

Amoebic dysentery and bacillary dysentery differ in the A) Mode of transmission. B) Appearance of the patientʹs stools .C) Etiologic agent. D) All of the above. E) None of the above.

Etiologic agent

The connection between unpasteurized milk and brucellosis in humans was made by Pasteur. Evans. Fleming. Ehrlich. Koch.

Evans

Which statement regarding Staphyloccocus aureus is FALSE? A) MRSA strains that can be traced to hospitals and clinics are referred to as HA-MRSA (hospital-acquired MRSA). B) CA-MRSA strains have a group of genes that codes for a leukocyte-destroying leukocidin. C) Evidence proves that CA-MRSA strains are more virulent than HA-MRSA because they produce leukocidin. D) MRSA strains are resistant to nearly all β-lactam antibiotics except ceftaroline, a new cephalosporin. E) Some hospitals screen patients for MRSA when they are discharged so they do not take a MRSA strain home with them.

Evidence proves that CA-MRSA strains are more virulent than HA-MRSA because they produce leukocidin. From Hair Follicle Infections slide: - Most MRSA originally hospital-acquired (HA-MRSA), but strains now widespread in healthy carriers, yield community-acquired (CA-MRSA) infections - Many hospitals now screen patients to limit spread

What virulence factory of P. aeruginosa inhibits host cell protein synthesis?

Exotoxin A - stops host cell protein synthesis

T/F Protozoans are eukaryotes and as such will always possess a nucleus, mitochondria, and chloroplasts.

False

T/F Protozoans are usually multicellular and found in arid environments.

False

T/F Slime molds and water molds are types of fungi.

False

Aflatoxin is a potent carcinogen produced by certain bacteria. T/F

False - Aspergillus flavus (fungus) grows on peanuts and other grains, producing AFLATOXIN, a potent carcinogen monitored by the FDA. What makes it false is that aflatoxin is produced by a fungus, not bacteria. p.808

Septicemia is only caused by Gram-negative bacteria. T/F

False - It can be cause by any bacteria that gets into the bloodstream. More Gram positives cause sepsis but cases from Gram negatives are worse.

The compost pile temperature at which pathogens, but not thermophiles, are killed is about 20-30ºC. T/F

False 55 to 66 degrees Celsius

T/F - Antibody and antigen are held to one another by covalent bonds

False Antibodies bind antigens through weak chemical interactions, and bonding is essentially non-covalent. Electrostatic interactions, hydrogen bonds, van der Waals forces, and hydrophobic interactions are all known to be involved depending on the interaction sites. - the chains in antibodies are held together by disulfide bonds

Foodborne intoxication requires the ingestion of living organisms. T/F

False Foodborne intoxication involves ingesting toxins and foodborne infection is ingesting the living cells.

High BOD values reflect small amounts of degradable organic matter in a sample of wastewater or other material. T/F

False High BOD values reflect large amounts of degradable organic matter in a sample of wastewater or other material.

Clostridium tetani is a highly invasive pathogen that causes tetanus, characterized by spastic paralysis. T/F

False Is localized, not invasive.

The MMR vaccine is used to protect against measles, mange, and rubella. T/F

False MMR = Measles, Mumps, Rubella.

All Staphylococci are coagulase positive. T/F

False Most, including Staphylococcus epidermiditis are coag-negative. Correct

Actinomycosis is a fungal disease. T/F

False Originally thought to be a fungal disease. Causative Agent • Most commonly Actinomyces israelii: Gram-positive, filamentous, branching, slow-growing anaerobe

Protease inhibitors are useful in curing AIDS. T/F

False Protease inhibitors stop viral encoded protease from cutting polypeptides made during viral genome replication into individual proteins. - No cure for AIDS yet.

High concentrations of nucleic acids in the diet may lead to high levels of uric acid in the blood. T/F

False Proteins might but false.

Yellow fever is caused by a protozoan infection. T/F

False but malaria is!

Only females should receive the 9vHPV vaccine. T/F

False no cure but the symptoms can be treated - Gardasil 9 - boys and girls ages 9-12 - 2 doses and up to age 26 in women, 21 in men - 3 doses for age 15 and older..

You are reviewing for your nursing school entry exam with your study group. Each person in the group agrees to cover one disease. You select toxoplasmosis for your presentation. Toxoplasmosis can be a serious problem for those who are immunosuppressed, as well as for fetuses. Even in healthy people, the immune system may not clear the organism, resulting in a latent infection that can reactivate when immunity declines. Immunosuppressed patients who test positive for antibody to T. gondii always have an active infection. True False

False p.716 - Immunosuppressed patients who test positive for antibody to T. gondii are presumed to have a latent infection and are given prophylactic medication.

Which of these descriptions does NOT apply to the pathogenesis of meningitis? Vasodilation disrupts the blood-brain barrier and allows neutrophils and proteins to enter the CSF. Vasodilation from inflammation results in accumulation of fluid, causing brain swelling and nerve damage. Inflammation impedes normal outflow of CSF, causing it to accumulate and press the brain against the skull. Inflammation results in the formation of small blood clots that block capillaries and interrupt blood supply, leading to cell death. Fever causes the blood pressure to increase significantly, forcing lymph fluid into the meninges and causing brain swelling.

Fever causes the blood pressure to increase significantly, forcing lymph fluid into the meninges and causing brain swelling.

Fever allows the body to fight microbial invaders by which of the following mechanisms? - Fever prevents microbes with lower optimum temperatures from growing, giving the immune system time to eliminate those cells. - Fever increases the inflammatory response, but reduces the production of interferons unless the invader is a dsRNA virus. - A moderate fever reduces the inflammatory response but stimulates phagocytic activity in neutrophils and eosinophils. - Fever increases the multiplication of lymphocytes and enhances the release of substances that attract basophils that then engulf the microbial invader. - Fever plays all of these roles in the body.

Fever prevents microbes with lower optimum temperatures from growing, giving the immune system time to eliminate those cells.

What is the likely fate of a fish living in a eutrophic stream? Fish will first thrive as they have more food from the increase in photosynthetic organisms; as they deplete the dissolved oxygen, however, they will die. Fish will first thrive as they have more oxygen from the increase in photosynthetic organisms; eutrophic streams are low in nutrients however, so they will quickly die. Eutrophic streams are always anoxic. This means that only anaerobic fish will be able to survive in a eutrophic environment. Eutrophic streams have high levels of carbon dioxide. Fish will be able to use this for aerobic respiration, so will thrive in this environment. Eutrophic streams have low levels of nutrients in them. Only fish that feed off other fish will be able to survive in these streams.

Fish will first thrive as they have more food from the increase in photosynthetic organisms; as they deplete the dissolved oxygen, however, they will die.

Which are not arthropods?

Flukes

What are not arthropods?

Flukes.

What leads to organ failure in sepsis? Helper T cells and dendritic cells undergo apoptosis, causing immunosuppression that results in organ rejection. Uncontrolled inflammation that leads to neutrohpils releasing damaging toxins and lysosomal enzymes. Formation of small clots in capillaries, impairing blood flow to organs and causing hypoxia and cell death. Accumulation of endotoxins released from Gram-negative bacteria in the organs, causing tissue damage and cell death. Organisms from an infected site generate biofilms that break apart, leading to emboli and organ destruction.

Formation of small clots in capillaries, impairing blood flow to organs and causing hypoxia and cell death. p.668 Correct

The genomes of free-living spirochaetes are larger than those living in animal hosts. Why might this be so?

Free-living spirochetes will need genes to code for additional proteins to synthesize or obtain their own food from the environment around them. Parasitic spirochetes obtain nutrients from the animal host, and may not need to move towards those nutrients either.

Which organisms returned carbon dioxide to the atmosphere and nitrogen to the soil?

Fungi and bacteria.

Respiratory Tract Infections sinusitis

Fusarium Fungi

vaginitis

Gardnerella gram + bacteria

A vaccine to provide active immunity to serum hepatitis is prepared from: A) Viruses grown in tissue culture. B) Genetically modified yeast. C) Pooled gamma globulin D) Viruses grown in embryonated eggs. E) Viral particles in patients.

Genetically modified yeast

Which of the following are diseases caused by protozoa?

Giardiasis, malaria, AND toxoplasmosis.

Please select the definition regarding aspects of the digestive system that is INCORRECT. Dysentery—diarrhea characterized by fecal pus and blood. Cirrhosis—liver scarring that interferes with function. Gingivitis—inflammation of the teeth and gums. Dysbiosis—imbalance of the intestinal normal microbiota. Cariogenic—causing dental caries.

Gingivitis—inflammation of the teeth and gums. gingivitis is swelling and redness of the gums

Paralytic shellfish poisoning occurs when humans eat shellfish that have fed on

Gonyaulax spp.

Which of the following are most susceptible to complement lysis?

Gram negative bacteria

Which of the following organisms is most likely to be responsible for periodontal disease? A) Gram-positive cocci B) Gram-positive rods C) Gram-negative cocci D) Gram-negative

Gram-positive rods CHECK THIS ONE OUT

Your patient had abdominal surgery three days ago. She seemed to be doing well after the surgery, but today the skin around the surgical incision is red and swollen. In addition, your patient has a high fever and is complaining of muscle aches. She also has a rash and has diarrhea. You realize that her wound has become infected, despite your best efforts as a nurse, and you are concerned that she has developed toxic shock syndrome. You explain the details of this type of infection to the patient's family.Wound infections are commonly caused by Staphylococcus species, common inhabitants of the nostrils and the skin. These organisms are: - Acid-fast, pyrogenic, obligate aerobes. - Gram-positive, pyogenic, obligate aerobes. - Gram-positive, pyrogenic, facultative anaerobes. - Gram-positive, pyogenic, facultative anaerobes. - Gram-negative, pyogenic, facultative anaerobes.

Gram-positive, pyogenic, facultative anaerobes. Staphylococcus - surgical wounds - pus... - not acid fast - not pyrogenic = cause fever - Gram positive - stains purple - is a facultative anaerobe (many, many wounds anaerobic) Correct

What is the purpose of grinding yard waste before putting it into composting piles? Grinding yard waste is a method for introducing degradative microbes into the mix, needed for composting. Grinding the material breaks down some of the components that microbes cannot digest, such as cellulose. Grinding the material increases the surface area available for microbial action, so that the material can be degraded more efficiently. Grinding the matter introduces large amounts of oxygen into the material, which is essential for microbial degradation of matter. Bacteria are microscopic organisms and can only break down very small pieces of matter, which grinding produces.

Grinding the material increases the surface area available for microbial action, so that the material can be degraded more efficiently.

Heliobacter pylori

Heliobacteraceae; peptic ulcers, gastritis, stomach cancer; Gram-negative spirilum

All blood cells originate from the..

Hematopoietic stem cell.

Which of the following type of hepatitis is/are associated with liver cancer or cirrhosis? Hepatitis A Hepatitis B Hepatitis C Hepatitis B AND hepatitis C Hepatitis A AND hepatitis B

Hepatitis B AND hepatitis C The table on p.651

The most common chronic blood-borne infection in the U.S. is: A. hepatitis A B. hepatitis B C. hepatitis C D. hepatitis D

Hepatitis C

What does circulating HBeAg indicate about people with chronic HBV infection? Antibodies against this antigen confer immunity to HBV. High levels of this antigen are associated with increased risk of liver cancer. Low levels of this antigen are associated with increased risk of liver cancer. IgM against this antigen indicate active viral replication. High levels of this antigen are associated with decreased risk of liver cancer.

High levels of this antigen are associated with increased risk of liver cancer.

Human Herpesvirus 1 (HHV-1)

Human herpes simplex virus 1 (oral herpes)

Human Herpesvirus 3 (HHV-3)

Human herpes simplex virus 3 (chicken pox/shingles)

Gonococci are parasites of: cows. iguanas. sheep. humans. cows AND sheep.

Humans Neisseria gonorrhoeae (gonococcus, GC) is well adapted to grow within the human host. Like other STI agents, it is a human-specific pathogen that survives poorly in the environment.

Select the TRUE statement regarding clostridial myonecrosis. Prevention of clostridial myonecrosis can be achieved with the DCTaP vaccine. C. perfringens is an endospore-forming organism, and produces large number of spores in wounds or cultures. Hyperbaric chambers may be used to treat patients with gangrene—high levels of O2 reduce anaerobic conditions in the tissues. Clostridium perfringens infects healthy tissue and grows easily in well oxygenated tissues. The causative agent produces α-toxin, an A-B toxin that destroys lecithin in host cell membranes, leading to cell lysis.

Hyperbaric chambers may be used to treat patients with gangrene—high levels of O2 reduce anaerobic conditions in the tissues. Correct

You are looking after your sister's cat Singa. While playing one evening, Singa accidentally bites you instead of the toy mouse you are holding. The bite wound is very small but is in the fleshy part of your hand. You wash the wound, which is not bleeding much, and put a band aid on it. The next day, your whole hand is red, swollen, and painful, and some lymph nodes in your arm are enlarged and tender. You call your sister, who is an RN. She urges you to go to the doctor, telling you that animal bites can sometimes be infected, and that if that is the case, you will need treatment before the infection gets any worse. The PA goes on to explain that cat bite infections and bartonellosis (cat scratch disease) differ in that

I put the whole thing further up

When Lyme disease was first being investigated, the observation that frequently only 1 person in a household was infected was a clue leading to the discovery that the disease was spread by arthropod bites. Why was this so? A. Mosquitoes (an example of arthropods) are never inside a house. They are strictly outdoor animals, so they couldn't spread the infection inside the household. B. Mosquitoes (an example of arthropods) only bite once in their life cycle. As such, they can only transmit the illness once before they die. Even if an infected mosquito was inside a house, it could therefore only infect one human. C. If the infection is spread by the bite of an arthropod, it wouldn't spread easily by respiratory secretions, direct contact, or sexual contact between individuals within the family. D. Arthropods lose their mechanical ability to bite a human after a single bite, much like certain bees that lose their stinger after a single sting. This prevents them from transmitting the infection to more than one individual in a household.

If the infection is spread by the bite of an arthropod, it wouldn't spread easily by respiratory secretions, direct contact, or sexual contact between individuals within the family. A, B and D have nutty statements in them, so no.

When Lyme disease was first being investigated, the observation that frequently only one person in a household was infected was a clue leading to the discovery that the disease was transmitted by arthropod bites. Why was this so? Mosquitoes (an example of arthropods) are never inside a house. They are strictly outdoor insects, so they couldn't spread the infection inside the household. If the infection is spread by the bite of an arthropod, it wouldn't spread easily by respiratory secretions, direct contact, or sexual contact between individuals within the family. Mosquitoes (an example of arthropods) only bite once in their lifetime. As such, they can only transmit the illness once before they die. Even if an infected mosquito was inside a house, it could only infect one human. Arthropods lose their mechanical ability to bite a human after a single bite, much like certain bees lose their stinger after a single sting. This prevents them from transmitting the infection to more than one individual in a household. These are all valid reasons that support the theory that Lyme disease must be transmitted by arthropod bites.

If the infection is spread by the bite of an arthropod, it wouldn't spread easily by respiratory secretions, direct contact, or sexual contact between individuals within the family. Read them. The rest are stupid. Correct

Please select the TRUE statement regarding impetigo. A) Signs and symptoms of impetigo result from inflammation in the keratinized outer epidermal layer. B) The causative organism of impetigo is Cutibacterium acnes. C) Patients with impetigo experience fever, pain, and malaise. D) Impetigo is an example of a pyoderma—a superficial skin disease characterized by pus production. E) Treatment of impetigo requires administering at least two different antibiotics.

Impetigo is an example of a pyoderma—a superficial skin disease characterized by pus production. "Most common type of PYODERMA, skin infection characterized by PUS production resulting from infection of insect bite, burn, scrape, or other wound" Streptococcal Impetigo is caused by: Streptococcus pyogenes, also Staphylococcus aureus; both have group A carbohydrate

Which one of the following statements about bacterial vaginosis is false? a) It is the most common vaginal disease in women of childbearing age. b) In pregnant women, it is associated with premature delivery. c) Inflammation of the vagina is a constant feature of the disease. d) The vaginal microbiota shows a significant decrease in lactobacilli and a marked increase in anaerobic bacteria. e) The cause is unknown.

Inflammation of the vagina is a constant feature of the disease. Not true because p.735 - about half of BV cases are asymptomatic.

How does Ascaris lumbricoides, an intestinal parasite, cause choking and pulmonary symptoms in some people?

Ingested Ascaris larvae penetrate intestinal capillaries, from where they are carried to the lungs, causing coughing and shortness of breath.

Which of the following is NOT a route by which bacteria that cause infective endocarditis may gain access to the bloodstream? Ingestion Brushing teeth Trauma Dental procedures Kidney infection

Ingestion p.667 - The bacteria the cause infective endocarditis may enter the bloodstream: - from another infected body site - during dental procedure - during tooth brushing - from trauma Correct

Factors that work generically against any foreign substance entering the host are described as

Innate immunity

Skin and mucous membranes are mostly involved in

Innate immunity

Which of the following cytokines is most antiviral in its action?

Interferon

How does protein A help Staphylococcus aureus evade phagocytes? A) It binds the Fc portion of antibodies, interfering with opsonizaton and phagocytosis. B) It binds the Fab portion of antibodies, interfering with opsonizaton and phagocytosis. C) It coats the bacterial cell, destroying LPS and preventing recognition by phagocytes. D) It makes holes in the cytoplasmic membrane of phagocytes, killing them. E) It degrades complement protein C5a, preventing phagocyte recruitment and attachment.

It binds the Fc portion of antibodies, interfering with opsonizaton and phagocytosis. Acts like Streptococcus pyogenes's protein G. Table 22.3 - protein A - binds to Fc portion of antibody, thereby interfering with opsonization.

Presenters on the morning news channel that you watch alert consumers to a food recall. A particular brand of ice cream has tested positive for Listeria monocytogenes, an organism that causes asymptomatic or relatively mild disease in otherwise healthy individuals, but can be problematic in pregnant women. Your sister is expecting her first child, so you call her immediately and tell her about the recall. You fill her in on what you know about this organism and the disease it causes, having just learned about it yourself in your pre-nursing microbiology class. Your sister has questions that you are able to answer.Which of these statements concerning the causative agent of listeriosis is TRUE? It is a Gram-negative thermophile. It is uncommon in natural waters and vegetation. It can cause meningitis during the first month of life. It is usually transmitted by the respiratory route. Most strains of L. monocytogenes are resistant to penicillin.

It can cause meningitis during the first month of life. - notile, non-spore forming, facultative anaerobe, G+ rod that can grow at 4 C. - It is widespread in natural waters and vegetation *babies infected at birth usually develop meningitis after an incubation period of 1-4 wks. True p.703 - in epidemics the portal of entry is the Gi tract - most strains remain susceptible to penicillin

Rowley Pharmaceutical company produces a drug that promotes new blood vessel growth. Is there any application for this drug in wound treatment? A. It could be very beneficial-one of the biggest problems with wound infections is their largely anaerobic nature. The blood supply to the area is usually compromised, helping to create the anaerobic environment. Certain pathogenic microbes then thrive in this environment, creating very serious wound infections. B. It could be disastrous! Part of the reason abscesses lack blood flow is to keep toxins and bacteria from spreading to other areas of the body. Increasing blood flow to the area will provide a highway for these bacteria to seed other organ systems! C. It could be dangerous-wounds need to cut off blood supply to prevent the patient from bleeding to death. Increasing blood vessel development in the wound site might cause the patient to bleed out. D. There would be no beneficial effect-the immune response is already in place in the wound site, so increasing blood cell delivery to the area wouldn't increase or decrease the rate of wound healing.

It could be very beneficial-one of the biggest problems with wound infections is their largely anaerobic nature. The blood supply to the area is usually compromised, helping to create the anaerobic environment. Certain pathogenic microbes then thrive in this environment, creating very serious wound infections.

Why is Rickettsia rickettsii difficult to grow in culture? A) It is a double-stranded DNA virus. B) It is an obligate intracellular parasite. C) It is resistant to multiple antibiotics. D) It is an obligate anaerobe. E) It is Gram-negative.

It is an obligate intracellular parasite. Rickettsia rickettsii: tiny Gram-negative non-motile coccobacilli that are OBLIGATE INTRACELLULAR BACTERIA - Difficult to grow in culture.

Pick the one false statement about vulvovaginal candidiasis. a) It often involves the external genitalia. b) It is readily transmitted by sexual intercourse. c) It is caused by a yeast present among the normal vaginal microbiota in about one-third of healthy women. d) It is associated with prolonged antibiotic use. e) It involves increased risk late in pregnancy.

It is readily transmitted by sexual intercourse. p.736 - fungal infection. Like BV, it seems to occur after a disruption of the normal microbiota. Associated with many sexual partners or a new partner, but can occur in the absence of sexual intercourse.

What is a dimorphic fungi?

It may grow as mycelia or yeast and are often associated with disease in humans.

Pediculus humanus causes what disease?

It only uses humans as a host and can transmit a bacterial disease.

Which of the following statements about interferon is incorrect?

It only works on a few specific types of virus.

Describe Gonyaulax species.

It produces a non-protein neurotoxin and is a dinoflagellate.

What is the epidemiological significance of shingles? A) It shows that, as a latent viral infection, there is always a possible reservoir available to reinfect new susceptible individuals. B) It shows that we must always be vigilant against this deadly and highly infectious secondary infection in elderly and immunocompromised individuals. C) It shows that, as a chronic viral infection, individuals infected are always infectious to others around them, even when they do not show outward symptoms. D) It shows that our fight for long-term eradication of varicella zoster virus will be a long one, depending on immunizing all newly born individuals until all the people who had ever had contracted the illness have died. E) It shows that, as a latent viral infection, there is always a possible reservoir available to reinfect new susceptible individuals AND it shows that our fight for long-term eradication of varicella zoster virus will be a long one, depending on immunizing all newly born individuals until all the people who had ever contracted the illness have died.

It shows that, as a latent viral infection, there is always a possible reservoir available to reinfect new susceptible individuals AND it shows that our fight for long-term eradication of varicella zoster virus will be a long one, depending on immunizing all newly born individuals until all the people who had ever contracted the illness have died. I know the second part reads nutty but go with it.

Why is it possible to prevent rabies with vaccine given after exposure to the virus? The virus replicates extremely slowly. It is possible to prevent the disease by vaccinating while the viral load is very low. It takes some time for the virus to invade the CNS and travel to the peripheral nerves. Vaccination before this occurs will prevent the disease. Rabies vaccine contains anti-rabies antibodies. These bind to the rabies and virus and destroy it before it is able to cause any neuronal damage. People receive a rabies vaccine in their childhood. Giving a vaccine after exposure to the virus quickly boosts their memory cell populations, protecting them. It takes some time for the virus to invade the peripheral neurons and travel to the CNS. Vaccination before this occurs will prevent the disease.

It takes some time for the virus to invade the peripheral neurons and travel to the CNS. Vaccination before this occurs will prevent the disease.

Former president Ronald Reagan once commented at a press conference that the best way to combat the spread of AIDS in the US was to prohibit everyone from having sexual contact for 5 years. What would be the success of such a program if it were possible to carry out? A) It would 'cure' the population of HIV/AIDS. Everyone that had it would die off, and no new cases would occur, since sexual contact is the only mode of transmission for this virus. B) It would be a failure-this 'program' wouldn't eliminate IV drug use or transmission of HIV from mother to child. Sexual transmission isn't the ONLY mode of transmission for this virus. C) It would be a failure-many HIV positive individuals survive for longer than 5 years, even without treatment (sometimes without even realizing they're positive). Once sexual contact resumed, sexual transmission would also resume. D) It would be a failure-this 'program' wouldn't eliminate IV drug use or transmission of HIV from mother to child. Sexual transmission isn't the ONLY mode of transmission for this virus, AND many HIV positive individuals survive for longer than 5 years, even without treatment (sometimes without even realizing they're positive). Once sexual contact resumed, sexual transmission would also resume.

It would be a failure-this 'program' wouldn't eliminate IV drug use or transmission of HIV from mother to child. Sexual transmission isn't the ONLY mode of transmission for this virus, AND many HIV positive individuals survive for longer than 5 years, even without treatment (sometimes without even realizing they're positive). Once sexual contact resumed, sexual transmission would also resume. A is stupid, B and C are both good and D = B+C.

A pathologist claims it's much easier to determine the causative agent of meningitis than an infection of the skin or intestines. Is this true? Why or why not? A. It's not true-an infection is an infection, and it's always difficult to track down the exact causative agent. The organ system involved in the illness has nothing to do with identification of the causative agent. B.It's true-because the central nervous system (CNS) is generally microbe free, whereas the skin and intestines have a diverse normal flora. Separating out a pathogenic causative agent from the normal microbiota in these areas would be much harder to do than in the central nervous system (CNS). C.It's not true-the ventricles of the brain can provide a direct route for microbes to enter the central nervous system (CNS) from the bloodstream/extracellular fluid. Therefore, it would be just as likely to find a pathogenic microbe in the central nervous system (CNS) as it would be in the skin or intestines. D. It's true-there are only 1 or 2 microbes that can cause meningitis, yet there are hundreds (if not thousands) that can cause skin/intestinal infections. This makes it much easier to determine the causative agent for a case of meningitis.

It's true-because the central nervous system (CNS) is generally microbe free, whereas the skin and intestines have a diverse normal flora. Separating out a pathogenic causative agent from the normal microbiota in these areas would be much harder to do than in the central nervous system (CNS).

The researcher who did much of the early work on Clostridium tetani was: A. von Behring. B. Koch. C. Kitasato. D. Pasteur.

Kitasato • Studied tetanus (lockjaw) in Robert Koch's laboratory • Spores of Clostridium tetani found in soil and dust • Kitasato determined that C. tetani is obligate anaerobe, devised method of cultivating • Discovered animals injected with C. tetani developed tetanus, but no cells found elsewhere in body • Showed bacterial toxin responsible for disease Correct

An important diagnostic sign of measles is: A) Koplik's spots. B) giant cells. C) fever. D) swollen lymph nodes. E) red, weepy eyes.

Koplik's spots • Koplik spots of mucous membranes identifying feature - check the tongue.

Which organisms are used to make salt pickles? A. Lactobacillus species and Leuconostoc mesenteroides B. Propionibacterium and Spirulina C. Leuconostoc mesenteroides and Spirulina D. Streptococcus lactis and Lactobacillus E. Pediococcus cerevisiae and Lactobacillus plantarum

Lactobacillus species and Leuconostoc mesenteroides

Leptospirosis

Leptospira gram - bacteria

Treponema pallidum is similar in shape to: Escherichia coli. Staphylococcus aureus. Leptospira interrogans. herpes virus.

Leptospira interrogans. Treponema pallidum - syphilis - STI - very slender, highly motile (endoflagellum with corkscrew motion) spirochete that can't be seen using a gram stain. Leptospira interrogans - Leptospirosis - UTI, bacteria gets to urinary tract via bloodstream - slender, aerobic G- spirochete with hooked ends and more than 250 antigenic types

Which of the following genitourinary tract infections is a zoonosis?

Leptospirosis

Which of the following genitourinary tract infections is a zoonosis? Leptospirosis M. genitalium infection Cystitis Syphilis Genital herpes

Leptospirosis

A disease in which the urinary system is infected from the bloodstream is known as cystitis. pyelonephritis. candidiasis. leptospirosis.

Leptospirosis - cystitis is bladder infection, causative agents generally reach the bladder by moving up the urethra. - pyelonephritis is kidney infection that can cause kidney failure - more serious condition that develops with bacterial cystitis - candidiasis is a fungal genital system disease - Leptospirosis - zoonosis (animal urine) - bacteria enters the body through a mucus membrane or wound and is then carried to the urinary system by the bloodstream.

Which S. aureus virulence factor is not correctly matched with its function? A) Capsule—avoiding phagocytosis. B) Hyaluronidase—facilitates spread in tissues. C) Protein A—interferes with opsonization. D) α-Toxin—makes holes in host cell membranes. E) Leukocidin—kills erythrocytes.

Leukocidin—kills erythrocytes. Leukocidin kills leukocytes (WBCs).

Artisanal cheese prepared with raw milk: Staphylococcus aureus Vibrio parahaemolyticus E. coli 0157:H7 Listeria monocytogenes Campylobacter

Listeria monocytogenes

Presenters on the morning news channel that you watch alert consumers to a food recall. A particular brand of ice cream has tested positive for Listeria monocytogenes, an organism that causes asymptomatic or relatively mild disease in otherwise healthy individuals, but can be problematic in pregnant women. Your sister is expecting her first child, so you call her immediately and tell her about the recall. You fill her in on what you know about this organism and the disease it causes, having just learned about it yourself in your pre-nursing microbiology class. Your sister has questions that you are able to answer.How would L. monocytogenes meningitis be diagnosed and treated? Lumbar puncture; antibiotics Brain biopsy; acyclovir Antibiogram results; antibiotics Gram staining; acyclovir Signs and symptoms; phage therapy

Lumbar puncture; antibiotics

Two weeks following a hiking trip in which several ticks were noticed and numerous mosquitoes were biting, a 25-year-old man notices a rash shaped like a red ring on the back of his thigh. He was not concerned because he had no other symptoms. Three months later he noticed significant pain in his elbows and his knees. What is the most likely cause of his illness? A. West Nile encephalitis B. Lyme disease C. Rocky Mountain Spotted Fever D. Ehrlichiosis

Lyme disease - the rash in a red ring.

Arachae are typically found living in extreme environments. And exception to this are

METHANOGENS

How would increased travel lead to increased spread of multicellular eukaryotic parasites?

Many eukaryotic parasites depend on vectors (often small blood-sucking insects) for transmission-so moving an infected individual into an area with new populations of vectors and new susceptible humans would increase the spread of the illness.

Ringworm "athletes foot" yeast infections

Many fungal skin infections (mycosis)

Why might a large number of competing microorganisms in a food sample result in lack of sensitivity of culture methods for detecting pathogens? a) The sheer number of microbes present might shut down the growth/replication of a pathogenic microbe in a food sample due to a 'starving out' effect. If it can't grow/replicate, it won't be detectable by a culturing method test. b) Many microbes secrete compounds that can be toxic for their competitors. If you need a pathogenic microbe to grow in a culture in order to detect it, these toxic compounds might inhibit the growth of the pathogen and impede the test AND culture methods often rely on biochemical changes taking place in medium for identification. With multiple microbes present, you may not be sure that the biochemical change observed is from the presence of a pathogen or from some other non-dangerous bacterium that also induces the biochemical change. c) Many microbes secrete compounds that can be toxic for their competitors. If you need a pathogenic microbe to grow in a culture in order to detect it, these toxic compounds might inhibit the growth of the pathogen and impede the test. d) You can't obtain a pure culture from a food sample that has a large number of microbes present. It would be impossible to detect only one from within the sample. e) Culture methods often rely on biochemical changes taking place in medium for identification. With multiple microbes present, you may not be sure that the biochemical change observed is from the presence of a pathogen or from some other non-dangerous bacterium that also induces the biochemical change.

Many microbes secrete compounds that can be toxic for their competitors. If you need a pathogenic microbe to grow in a culture in order to detect it, these toxic compounds might inhibit the growth of the pathogen and impede the test AND culture methods often rely on biochemical changes taking place in medium for identification. With multiple microbes present, you may not be sure that the biochemical change observed is from the presence of a pathogen or from some other non-dangerous bacterium that also induces the biochemical change.

Select the statement about plague that is TRUE. DIC causes the dusky color of skin and mucous membranes characteristic of bubonic plague. When Yersinia pestis infects the lungs, septicemic plague always develops. The causative agent of plague is a facultatively anaerobic, Gram-positive rod. Bubonic plague is transmitted by the bite of infected mosquitoes that have also bitten rats. Bubonic plague is almost always fatal.

Maybe the first one. - that's septic plague - not always - facultative anaerobe, G-, not + - bite of infected fleas - 50-80% fatal is not treated... but it is treatable so NOPE (tried, it was wrong)

Which of the following cannot be used in composting? Nutrient-poor potting soil Grass clippings Vegetable peelings Meats and fats Leaves

Meats and fats

Haemophilus influenzae

Meningitis, Upper respiratory infections Conjunctivitis, Pink Eye Gram - negative pleomorph nonmotile facultative anaerobe relies on lysed RBCs for metabolism type B (Hib) is main type- capsule (PRP)- meningitis type A is conjunctivitis- no capsule- direct contact treatment is antibiotics in eyes prevention: vaccine has capsule Ag in it droplet transmission (meningitis) highly communicable treatment: antibiotics morbidity: neurological change prevention: vaccine (DTaP)

Which of the following is a disadvantage of landfills? Methane gas that is produced in landfills as the organic waste material anaerobically decomposes must be removed. Only a limited number of sites are available for making landfills near urban and suburban areas. Pollutants such as heavy metals and pesticides frequently leak from landfill sites, reaching underground aquifers and contaminating drinking water, AND only a limited number of sites are available for making landfills near urban and suburban areas. Pollutants such as heavy metals and pesticides frequently leak from landfill sites, reaching underground aquifers and contaminating drinking water. Methane gas that is produced in landfills as the organic waste material anaerobically decomposes must be removed AND only a limited number of sites are available for making landfills near urban and suburban areas.

Methane gas that is produced in landfills as the organic waste material anaerobically decomposes must be removed AND only a limited number of sites are available for making landfills near urban and suburban areas. p.795

What are indicator organisms? Microbes that alert a physician to the presence of a gastrointestinal illness. Microbes that are contaminants in laboratory media. Microbes whose presence in an environment suggests the presence of animal urine. Microbes whose presence in an environment suggests fecal contamination. All of the choices are correct.

Microbes whose presence in an environment suggests fecal contamination.

conjuctivitis

Moraxella gram - bacteria

Which of the following diseases of the gastrointestinal system is transmitted by the respiratory route? A) staphylococcal enterotoxicosis B) mumps C) Vibrio gastroenteritis D) bacillary dysentery E) traveler's diarrhea

Mumps Virus INHALED via saliva droplets, spreads via bloodstream

Which of the following refers to a symbiotic association between plant roots and fungi? Lichens Mycorrhizae Ant gardens Extremophiles Geosmins

Mycorrhizae fungi = myco, roots = rizzhae

Characteristic inclusion bodies formed by cells of organisms infected with rabies are called: A. Negri bodies. B. metachromatic bodies. C. polyphasic bodies. D. Koplik spots.

Negri bodies Koplik spots is measles (rubeola)

Epidemics of meningitis appear to involve: A. Streptococcus pneumoniae. B. Haemophilus influenzae. C. Neisseria meningitidis. D. Streptococcus pyogenes.

Neisseria meningitidis p699 - Neisseria meningitidis, often called meningococcus, is frequently responsible for epidemics of meningitis. -cause of meningococcal meningitis - Gram negative diplococcus with capsule - respiratory droplets, attach with pili

Epidemics of meningitis appear to involve Streptococcus pneumoniae. Haemophilus influenzae. Neisseria meningitidis. Staphylococcus aureus. Streptococcus pyogenes.

Neisseria meningitidis.

You go for your regular Pap test and are informed by your OBGYN that you are positive for HPV16. You are surprised because you have not noticed any signs or symptoms of infection. Your doctor explains that sexually transmitted HPV strains are among the most common of the STI agents, and that while some HPV strains cause warty growths of the external and internal genitalia, other strains cause non-warty lesions of mucosal surfaces such as the uterine cervix. These strains are of concern because they are a major factor in the development of cervical cancer. You ask your doctor a number of questions regarding HPV and HPV infection. Is the risk of developing cervical cancer the same for all strains of HPV? No. Wart-causing HPV types exists in infected cells as extrachromosomal, closed DNA circles, while the genome of high-risk (cancer-associated) HPV types integrate into the chromosome of the host cell. Yes. HPV is a double-stranded DNA virus, so any HPV strain will definitely integrate into the host cell chromosome, becoming oncogenic and leading to the development of cancer. No. It depends on the infected person's immune system rather than the virus. Any HPV type can cause cancer but some people have a stronger immune system than others and are able to fight off the virus before it gets a chance to integrate into the host chromosome. No. Wart-causing HPV types are double-stranded DNA viruses which are easily eliminated by the immune system while cancer-causing HPV types are retroviruses that can integrate into the host cell chromosome. No. Wart-causing HPV types are double-stranded RNA viruses which are easily eliminated by the immune system while cancer-causing HPV types are double-stranded DNA viruses that can integrate into the host cell chromosome.

No. Wart-causing HPV types exists in infected cells as extrachromosomal, closed DNA circles, while the genome of high-risk (cancer-associated) HPV types integrate into the chromosome of the host cell. p.752

It has been estimated that 99% of the intestinal bacteria are

OBLIGATE ANEROBES

Comparatively greater energy is released when

OXYGEN IS THE FINAL ELECTRON RECEPTOR

C3b is involved in

Opsonization

Which of the following produces a greenish pigment that may appear in infected wounds? E. coli S. aureus S. pyogenes P. aeruginosa

P. aeruginosa slide 21 Water-soluble pigments (fluorescent yellow pyoverdin and blue pyocyanin) yield characteristic green color

The Gram-negative opportunistic rod that can grow in a wide variety of environments, including disinfectants and soaps, is: E. coli. S. aureus. S. pyogenes. P. aeruginosa.

P. aeruginosa slide 24 Pseudomonas aeruginosa widespread in nature, grows in most places that have moisture - Soaps, ointments, eyedrops, contact lens solutions, cosmetics, disinfectants, swimming pools, hot tubs, even distilled water - Introduced into hospitals on ornamental plants, flowers, produce; also found on hospital equipment, inner soles of shoes, illegal injectable drugs

A patient presents with a wound that contains green pus. This suggests to you that the wound is infected with P. aeruginosa, which multiplies in photosynthetic biofilms that are green. S. epidermidis, which produces the water-soluble pigments pyoverdin and pyocyanin. S. pyogenes, which produces a water-soluble blue-green capsule. P. aeruginosa, which produces the water-soluble pigments pyoverdin and pyocyanin. either S. epidermidis or S. aureus, both of which produce pigment soluble coagulase

P. aeruginosa, which produces the water-soluble pigments pyoverdin and pyocyanin. yellow and blue make green pyocyanin - sidiophore to steal host cell iron Correct

Haemophilus influenzae meningitis

PATHOGEN: Haemophilus influenzae MORPHOLOGY: Gram-negative rod PORTAL OF ENTRY: Respiratory tract METHOD OF TRANSMISSION: Endogenous infection; aerosols TREATMENT: Cephalosporin Prevention: Capsular Hib vaccine

Rubulavirus (mumps)

Paramyxoviridae RNA virus

The most common infectious agent acquired from the bite wounds of a number of kinds of animals is: - Escherichia coli. - Pasteurella multocida. - Actinomyces israelii. - Pseudomonas aeruginosa.

Pasteurella multocida slide 44 Humans and many animals carry P. multocida as normal oral and upper respiratory microbiota, serve as reservoirs • Cats more likely to carry than dogs Correct

You are looking after your sister's cat Singa. While playing one evening, Singa accidentally bites you instead of the toy mouse you are holding. The bite wound is very small but is in the fleshy part of your hand. You wash the wound, which is not bleeding much, and put a band aid on it. The next day, your whole hand is red, swollen, and painful, and some lymph nodes in your arm are enlarged and tender. You call your sister, who is an RN. She urges you to go to the doctor, telling you that animal bites can sometimes be infected, and that if that is the case, you will need treatment before the infection gets any worse.You go to the urgent care in your neighborhood. There, a physician's assistant (PA) tells you that wounds caused by animal bites (especially cats) can be infected with: Pasteurella multocida AND Streptobacillus moniliformis. Bartonella henselae OR Streptobacillus moniliformis. Streptococcus epidermidis AND Streptobacillus moniliformis. Staphylococcus aureus AND Streptobacillus moniliformis. Pasteurella multocida OR Bartonella henselae.

Pasteurella multocida OR Bartonella henselae. First one - worry for all animal bites Second one - more cat bite specific Correct

Fungus imparting texture, flavor and bluish green veins in cheese

Penicillin roquefortii

Which of the following statements regarding Vibrio vulnificus infections in FALSE? The causative agent of the disease, V. vulnificus, is a Gram-negative halophile. People consuming V. vulnificus typically develop signs and symptoms of gastroenteritis. V. vulnificus causes different signs/symptoms, depending on the portal of entry. Liver disease increases the risk of developing severe illness with V. vulnificus infections. People with V. vulnificus infection always develop life-threatening sepsis.

People with V. vulnificus infection always develop life-threatening sepsis. - Is a halophile - seafood right? So tolerates sea water. Salty. - when ingested, an otherwise healthy person may develop only a self liming case of gastroenteritis. - Signs and symptoms do depend on the portal of entry (mouth/open wounds). - in a person with liver disease, the bacteria may invade the bloodstream, leading to fevers, chills, severe skin blistering and shock due to sepsis, which is often fatal. - nope! can be self-limiting or more severe. Depends.

Please identify the mismatched pair. - Alveolar macrophages—lungs - Peritoneal macrophages—meninges - Mesangial macrophages—kidneys - Microglial cells—brain - Kupffer cells—liver

Peritoneal macrophages—meninges

Which of the following does not apply to staphylococcal toxic shock syndrome? Person-to-person spread frequently occurs. It may lead to kidney failure. The causative organism produces superantigens. The causative organism usually does not enter the bloodstream. It is can occur in males and females.

Person-to-person spread frequently occurs. p.737 - it does not spread from person to person

What sign would differentiate meningococcal meningitis from pneumococcal meningitis? It is impossible to differentiate between pneumococcal meningitis and meningococcal meningitis. Petechiae are seen in pneumococcal meningitis but not in meningococcal meningitis. Headache and neck stiffness are experienced with pneumococcal meningitis but not with meningococcal meningitis. Petechiae are seen in meningococcal meningitis but not in pneumococcal meningitis. Headache and neck stiffness are experienced with meningococcal meningitis but not with pneumococcal meningitis.

Petechiae are seen in meningococcal meningitis but not in pneumococcal meningitis.

Free-floating, photosynthetic organisms found in marine environments are

Phytoplankton

Enterovirus (polio)

Picornaviridae RNA virus

Please select the FALSE statement regarding plague and its transmission.

Plague is caused by a Gram-negative organism and is thus completely untreatable with antibiotics.

How would crowded conditions in cities favor spread of bubonic plague? Plague is transmitted by mosquitoes; more people close together gives an infected mosquito more chances to bite humans and transmit the causative agent, spreading plague. Plague is transmitted by infected fleas; these fleas may be found on rodents (such as rats). More people in an area means a greater chance of interactions with animals carrying infected fleas, increasing the spread of plague. Plague is transmitted by rats; more people means more waste, and more waste means more rats. This would favor the spread of plague. Plague is transmitted by direct contact (e.g. skin to skin). More people in an area provides more chances for infected individuals to directly contact and infect other individuals, spreading plague. Bubonic plague is transmitted by aerosol droplets. People in cities are always dependent on public transport, which is typically overcrowded, so are at risk of acquiring plague bacteria.

Plague is transmitted by infected fleas; these fleas may be found on rodents (such as rats). More people in an area means a greater chance of interactions with animals carrying infected fleas, increasing the spread of plague. Correct Plague - fleas - rats.

One of the greatest causes of human deaths through time has been due to

Plasmodium spp.

Untreated streptococcal pharyngitis can lead to all of the following except: A. Scarlet fever B. Rheumatic fever C. Otitis media D. Glomerulonephritis E. Pneumococcal pneumonia

Pneumococcal pneumonia The others are due to Streptococcus pyogenes but Pneumococcal pneumonia is caused by Streptococcus pneumoniae - a gram-positive diplococcus known as pneumococcus.

"Pneumonia" in weakened immune system

Pneumocystis fungi

This opportunist is the most frequent cause of life-threatening pneumonia in AIDS patients: A. Cryptococcus neoformans B. Candida albicans C. Malassezia furfur D. Pneumocystis jiroveci E. None of the choices is correct

Pneumocystis jiroveci

Dental plaque E.A biofilm on teeth. Dysentery D.Condition characterized by crampy abdominal pain and bloody diarrhea. Enterotoxigenic E. coli, ETEC B.Pathogenic E. coli which secrete plasmid-encoded enterotoxins. Fasciitis C.Inflammation of the fascia, bands of fibrous tissue that underlie the skin and surround muscle and body organs. Fibrinogen A.A blood protein that is converted to fibrin at a wound site.

Probiotic E.Live, beneficial microorganisms. PulseNet A.Surveillance network established by the CDC to facilitate the tracking of foodborne disease outbreaks. Pus B.Thick, opaque, often yellowish material that forms at the site of infection, made up of dead neutrophils and tissue debris. Superantigen C.Toxins that non-specifically activate many T cells, resulting in excessive cytokine production that leads to severe reactions and sometimes fatal shock. Tetanospasmin D.Neurotoxin produced by Clostridium tetani.

Which single-celled eukaryotic organisms lack chlorophyll?

Protozoa.

Which of the following virulence factors of Yersinia pestis aid(s) in attachment to host cells? Endotoxin Pla PsaA Yersinia outer proteins (Yops) V antigen

PsaA p.670/Table 25.3 endotoxin = no Pla is about clearing the way to spread Yops is about inhibiting phagocytosis V antigen controls the type III secretion system that delivers the Yops to host cells. PsaA = adhesin - role in ATTACHMENT to host cells. Correct

A soluble greenish pigment is produced by

Pseudomonas aeruginosa

A frequent complication of scalded skin syndrome is a secondary infection caused by: A. M. luteus. B. S. pyogenes. C. Pseudomonas spp. D. S. epidermidis.

Pseudomonas spp.

Sarcodina move by use of?

Pseudopodia.

Pseudomonas aeruginosa

Psuesomonodaceae; HAI- endocarditis, pneumonia, UTI, bacterimia; Gram-negative bacilli

Lyssavirus (rabies)

Rhabdoviridae RNA virus

The most common type of virus leading to rhinitis is: A. Herpes simplex virus B. Rhinovirus C. Retrovirus D. Coronavirus E. Adenovirus

Rhinovirus

The zone of soil around roots of plants is called the A. Rhizosphere B. Ionosphere C. Hydrosphere D. Lithosphere E. Biosphere

Rhizosphere lots of gram-negative bacteria found here.

The more severe form of African sleeping sickness is called the: A. Gambian form. B. Rhodesian form. C. Guyanan form. D. Brazilian form.

Rhodesian form - causes acute disease that can progress rapidly and person dies from heart failure in 6 months. Gambian form is chronic and death is from secondary infection years later.

Rocky Mountain Spotted Fever typhus

Rickettsia gram - bacteria

Which of the following is an obligate intracellular parasite? A) Micrococcus luteus B) Streptococcus pyogenes C) Rickettsia rickettsii D) Pseudomonas aeruginosa E) Staphylococcus epidermidis

Rickettsia rickettsii Rickettsia rickettsii: tiny Gram-negative non-motile coccobacilli that are OBLIGATE INTRACELLULAR BACTERIA - Difficult to grow in culture.

Acute gastroenteritis that occurs after an incubation period of 4 to 24 hours isprobably caused by: A) Giardia B) Rotavirus C) Salmonella D) Staphylococcus aureus E) Trichinella

Rotavirus

Helminths

Roundworms and flatworms

A public health official was asked to speak about immunizations during a civic group lunch. One parent asked if rubella was still a problem. In answering the question,the official cautioned women planning to have another child to have their present children immunized against rubella. Why did the official make this statement to the group? A) Rubella is spread very easily by respiratory secretions and is largely asymptomatic. However, it can cause birth defects/stillbirth in pregnant women. Women with other children would want to prevent these children from acquiring the virus before attempting to conceive a new child in order to protect the fetus. B) Rubella is passed very easily between children, so the official was trying to protect the child that would soon be born from this infection that might be brought in by its siblings after it was born. C) The official is getting kickbacks and bribes from the companies making the vaccines, and he's trying to pad his pockets by getting as many people immunized as possible, regardless of whether they need it or not. D) Rubella infections often lead to very serious and potentially fatal complications. While a woman is pregnant, she may not be able to take care of a sick child as easily. The vaccine will prevent the child already in the family from falling ill and potentially dying due to these possible complications. E) All of the answer choices are correct.

Rubella is spread very easily by respiratory secretions and is largely asymptomatic. However, it can cause birth defects/stillbirth in pregnant women. Women with other children would want to prevent these children from acquiring the virus before attempting to conceive a new child in order to protect the fetus. Rubella (German measles, 3-day measles) Early in pregancy, virus particles can cross placenta, infect fetus; less likely later in pregnancy - Nearly all types of fetal cells susceptible - Some are killed, others develop persistent infection with impaired cell division and chromosomal damage - Yields pattern of fetal abnormalities: congenital rubella syndrome - May include eye defects, brain damage, deafness, heart defects, low birth weight; stillborn babies

The yeast used in making bread, beer, and wine is A. Spirulina B. Propionibacterium C. Saccharomyces cerevisiae D. Streptococcus lactis and Lactobacillus E. Leuconostoc mesenteroides

Saccharomyces cerevisiae

Which of the following is often associated with poultry products? Salmonella AND Pseudomonas Campylobacter AND Lactobacilllus Lactobacillus AND Salmonella Salmonella AND Campylobacter Pseudomonas AND Campylobacter

Salmonella AND Campylobacter

Salmonella enteritidis

Salmonellosis - Food poisoning gram negative rod chills, diarrhea, fever

Which is the first step in wastewater purification? A. Storage B. Bioremediation or BOD removal C. Aeration and settling D. Chlorination E. Sedimentation

Sedimentation Primary treatment - physical removal of what settles out.

Gardeners sometimes plant clover between productive growing seasons. Why would this practice be beneficial? A. Several types of nitrogen-fixing bacteria form a symbiotic relationship with the roots of clover. This would increase the amount of nitrogen in the soil available for subsequent seasons of other plants' growth. B. Clover is consumed by a number of animals. This would encourage the animals to feed/graze on the areas, leaving behind nitrogen-rich manure that would act as a natural fertilizer for subsequent seasons of other plants' growth. C. Clover is a unique plant that is capable of atmospheric nitrogen-fixation (pulling nitrogen directly out of the air and turning it into ammonia or amino acids). This directly provides nitrogen compounds for the soil for subsequent seasons of other plants' growth. D. The beneficial effect is more for water retention and elimination of soil erosion in between growing seasons. It has nothing to do with nitrogen compounds in the soil.

Several types of nitrogen-fixing bacteria form a symbiotic relationship with the roots of clover. This would increase the amount of nitrogen in the soil available for subsequent seasons of other plants' growth. So it's about replenishing the nitrogen in the soil.

You are a nurse on a post-surgical ward. One of your patients is an elderly woman who has just had her gallbaldder removed. When you go into her room to check on her, you find that she has significant diarrhea, as well as fever and abdominal pain. You suspect that she has Clostridium difficile infection (CDI), and send a sample of her feces down to the hospital lab for analysis. The test result indicates that your patient does indeed have CDI. You make your patient comfortable and answer her questions about her illness.Your patient wonders how she contracted CDI. Which of the following would be a good explanation for her? Most surgical equipment is contaminated with C. difficile endospores, which are ubiquitous. Most likely your patient acquired CDI directly from her surgery. She has been on antibiotic therapy because of her surgery. The antibiotics disrupted her normal intestinal microbiota, allowing any C. difficile present to flourish. Nobody knows how people contract CDI. The causative bacterium is a common environmental organism and it is not known how it becomes pathogenic. C. difficile is usually a community-acquired organism. She most likely came to the hospital already infected with it but it takes several days for signs and symptoms of CDI to appear. She is elderly; elderly people frequently develop gastrointestinal complaints such as CDI. It is to be expected in an aging person.

She has been on antibiotic therapy because of her surgery. The antibiotics disrupted her normal intestinal microbiota, allowing any C. difficile present to flourish.

Why would the Sabin oral polio vaccine need to be eliminated as we get closer and closer to eradication of the polio virus? A. Since it is a live, attenuated viral vaccine, there's always the chance it could revert back into a pathogenic state by mutation. This would introduce new strains still capable of causing the illness into human populations, thwarting elimination. B. The Sabin vaccine is cheaper than the Salk (killed virus) vaccine, but it isn't as effective at producing a protective response. We'll need to shift to the more effective Salk vaccine worldwide eventually to complete the eradication process. C. The Salk vaccine is far cheaper than the Sabin vaccine. To effectively eradicate polio, we'll need to shift all of our resources into the cheapest and easiest to deliver vaccine possible.That is the Salk vaccine. D. The Sabin vaccine can only be administered to human beings. We need to vaccinate bird populations as well to effectively control the spread of polio. Birds are a natural reservoir for the virus.

Since it is a live, attenuated viral vaccine, there's always the chance it could revert back into a pathogenic state by mutation. This would introduce new strains still capable of causing the illness into human populations, thwarting elimination.

Please select the INCORRECT statement regarding helminths and the diseases they cause.

Some helminths are inadvertently ingested. For example, Onchocerca volvulus, the cause of River blindness, is transmitted by drinking contaminated water or eating fish carrying this parasite.

Why might the presence of Apsergillus flavus in grain products be a problem? Some species of Aspergillus cause systemic infections in animals that consume them. Some species of Aspergillus produce aflatoxin, a potent carcinogen. All species of Aspergillus produce aflatoxin, a potent carcinogen. Presence of Aspergillus indicates a highly contaminated environment. Some species of Aspergillus produce aflatoxin, which kills plants and causes crop loss.

Some species of Aspergillus produce aflatoxin, a potent carcinogen. - it produces AFLATOXIN, a potent carcinogen monitored by the FDA. Aflatoxin mycotoxins are toxic to humans and even more toxic to animals. They also cause cancer in humans and animals

Treponema pallidum

Spirochaetaceae; venereal syphilis; Gram-negative spirochete

Borrelia burgdorferi

Spirocheaetaceae; Lyme disease; weak Gram-negative spirochete

Mad cow disease Creutzfeldt-Jakob disease (CJD) Scrapie; chronic wasting disease

Spongiform encephalopathies Prions

Which of the following is a foodborne intoxication? A. Salmonellosis B. Staphyloccocal aureus C. Listerosis D. Vibrio enteritis E. Campylobacteriosis

Staphyloccocal aureus It and Clostridum - they make toxins we ingest = foodborne intoxication. A and E create foodborne infections when live cells are ingests, Vibrio you get from seafood.

Staphylococcus aureus

Staphylococcaceae; skin infections, TSS, food borne illness; Gram- positive cocci

Why is ham is a common vehicle for S. aureus food poisoning? Ham is a particularly moist meat, so encourages the growth of the staphylococci, which can only survive on foods with a high aw. Ham is an untreated raw meat. As such, it is prone to contamination by the staphylococci that thrive on meat, resulting in food poisoning. Staphylococci are able to act synergistically with the normal microbiota of pigs, creating toxins that result in food poisoning. S. aureus is part of the normal microbiota of cows, so is a frequent cause of S. aureus food poisoning. Staphylococci are adapted to salty conditions and thrive with little competition on cured meats such as ham.

Staphylococci are adapted to salty conditions and thrive with little competition on cured meats such as ham.

Bacterial attachment to fibronectin and fibrinogen coating plastic devices like catheters and heart valves is a virulence mechanism of: A. Staphylococcus B. Streptococcus C. Clostridium D. Pseudomonas

Staphylococcus Staphylococcus clumping factor

Wound infection

Staphylococcus gram + bacteria

Which of the following organisms can grow at a lower water activity than most spoilage bacteria? Pseudomonas sp. most yeasts Streptococcus pyogenes Staphylococcus aureus

Staphylococcus aureus It faces too much competition normally but it is salt tolerant and can grow on salty ham where there will be less competition from other spoilage bacteria. (salty foods have a reduced water availability but S. aureus tolerates salt so it's fine.)

Boston cream pie contender at the Los Angeles County Fair pastry competition: Staphylococcus aureus Vibrio parahaemolyticus E. coli 0157:H7 Listeria monocytogenes Campylobacter

Staphylococcus aureus You cooked it (hot), let it cool and now Staphylococcus aureus is making toxins in there and you're going to eat it and end up with a foodborne intoxication.

In which organism were phagocytes first reported?

Starfish larvae

A pure or mixed sample of known microbes that are added to food is a/an A. Starter culture B. Aerator C. Flavor inducer D. Fermenter E. Leavening

Starter Culture

Which of the following is mismatched regarding protection of the body from pathogens? - Respiratory tract—mucociliary escalator - Stomach—acid conditions and normal microbiota - Saliva—lysozyme, peroxidase, and lactoferrin - Peroxidases—milk, saliva, mucus, and some phagocytes - Skin—salt, fatty acids, and normal microbiota

Stomach—acid conditions and normal microbiota

Streptococcus pyogenes (Group A strep)

Streptococcaceae; strep throat, scarlet fever, impetigo, necrotizing fasciitis

Strep throat dental caries (cavities)

Streptococcus gram + bacteria

Neonatal meningitis

Streptococcus agalactiae - Group B strep enter into infant's blood stream during delivery and is often associated with sepsis in mother and infant as well Encapsulated strains of E. coli from Mom's gut Listeria monocytogenes from bloodstream of infected mom.

The principal cause of dental caries is: S. mutans. S. salivarius. S. mitis. S. sanguis.

Streptococcus mutans Streptococcus mutans, related species are cariogenic (caries generating); live only on teeth • Thrive in acidic environments - Produce lactic acid from fermentation - Convert sucrose into extracellular insoluble polysaccharides called glucans - Glucans essential for development of dental caries on smooth tooth surfaces

Which of the following mostly causes typically pneumonia but may also cause meningitis? Klebsiella pneumoniae Streptococcus pneumoniae Mycoplasma meningitidis Neisseria meningitidis Mycoplasma pneumoniae

Streptococcus pneumoniae p.698 - Pneumococcal meningitis is caused by Streptococcus pneumoniae, part of the normal microbiota in the nose and throat of many healthy individuals. Although best known as a cause of pneumonia, it is also the leading cause of meningitis in adults.

which of the following manifests itself in three clinical stages? gonorrhea syphilis trachoma non-gonococcal urethritis

Syphilis Primay - hard, painless chancre - infectious (intense inflammatory response) Secondary - bacteria goes systemic - lesions on mucus membranes like tongue - infectious (immune complexes with antibodies) Latent - early stages still infectious, later not infectious Tertiary - not infectious (hypersensitivity reactions to small numbers of T. pallidum that grow and persist in tissues) - can be gummatous, cardiovascular or neurosyphilis

HIV attacks a variety of cell types but the most critical are: red blood cells. nerve cells. platelets. T helper cells.

T helper cells p.755 - HIV can infect a variety of human cell types, but the most significant is T helper cells. - CD4 T cells.

Early work with Clostridium tetani and Corynebacterium diphtheriae demonstrated: - the importance of thoroughly cooking food. - the usefulness of vaccines. - the potential threat from spore-forming bacteria. - that disease can be caused by poisonous substances produced by bacteria.

TAKE WITH GRAIN OF SALT - the usefulness of vaccines I think this: - that disease can be caused by poisonous substances produced by bacteria. Kitasato determined that C. tetani is obligate anaerobe, devised method of cultivating • Discovered animals injected with C. tetani developed tetanus, but no cells found elsewhere in body • SHOWED BACTERIAL TOXIN RESPONSIBLE FOR DISEASE• Emil von Behring similarly demonstrated diphtheria caused by toxin from Corynebacterium diphtheriae

You're a famous eye surgeon, specializing in corneal transplants. Give your choice, would you rather have a cornea for transplantation from a 20 year old individual, or a 60 year old individual (assuming there are no optical defects due to age), and why? A. The 60 year old-clearly, if the individual has gone 60 years with no optical defects, this is a very good organ for transplantation. The 20 year old cornea may have defects that simply haven't become apparent yet. B. The 20 year old cornea-younger is always better when dealing with tissue transplants.You want to give the transplant the longest possible time for survival in the recipient. C. The 20 year old cornea-while extremely rare, it may be possible to transmit prions through corneal transplants. It's less likely that a 20 year old would have encountered and become contaminated by prions than the 60 year old. D. The 20 year old cornea-never mind prions, there are a variety of agents that can hide in corneal tissues and that might cause disease. It's far more likely that the 60 year old would be harboring one or more of these pathogens than the 20 year old.

The 20 year old cornea-while extremely rare, it may be possible to transmit prions through corneal transplants. It's less likely that a 20 year old would have encountered and become contaminated by prions than the 60 year old.

Which would be more likely to cause illness: a water sample that tested positive for coliforms or one that tested positive for E. coli O157:H7? Both would be equally capable of causing illness because all coliforms cause illness. The E. coli O157:H7 sample would be more likely to cause illness. This strain of bacterium is highly pathogenic and capable of causing kidney damage. Neither; there is usually a small amount of coliforms (including E. coli O157:H7) in all water. The coliform-positive sample would be more likely to cause illness. Coliforms are inherently more pathogenic than the weak O157:H7 lab strain of E. coli. The coliform-positive sample would be more likely to cause illness. Coliforms indicate the presence of fecal contamination, which always leads to illness.

The E. coli O157:H7 sample would be more likely to cause illness. This strain of bacterium is highly pathogenic and capable of causing kidney damage.

Why is it important to give the MMRV vaccine to healthy, non-immune contacts of people with immunodeficiencies? A) The MMRV vaccine is a toxoid vaccine, so immunodeficient people cannot receive it. They are protected by herd immunity. B) The MMRV vaccine is an attenuated vaccine, so immunodeficient people cannot receive it. They are protected by antibodies from their close contacts. C) The MMRV vaccine is an attenuated vaccine, so immunodeficient people cannot receive it. They are protected by herd immunity. D) The MMRV vaccine is an inactivated vaccine, so immunodeficient people cannot receive it. They are protected by antibodies from their close contacts. E) The MMRV vaccine is an inactivated vaccine, so immunodeficient people cannot receive it. They are protected by herd trending.

The MMRV vaccine is an attenuated vaccine, so immunodeficient people cannot receive it. They are protected by herd immunity. Attenuated vaccines contained weakened but still able to replicate microorganisms. Possibility of causing severe disease.

What changes might occur in the vagina if lactobacilli were eliminated? A) The acidic pH might change to a more neutral (and pathogen-fostering) level. B) Lactobacilli often produce the anti-anaerobic bacteria compound hydrogen peroxide. Without it, anaerobic pathogens might cause disease states (e.g. gas gangrene). C) There would be no change in the health of the woman-other members of the normal flora would 'fill in the gap' in the population, resulting in no overall change. D) The acidic pH might change to a more neutral (and pathogen-fostering) level AND lactobacilli often produce the anti-anaerobic bacteria compound hydrogen peroxide. Without it, anaerobic pathogens might cause disease states (e.g. gas gangrene)

The acidic pH might change to a more neutral (and pathogen-fostering) level AND lactobacilli often produce the anti-anaerobic bacteria compound hydrogen peroxide. Without it, anaerobic pathogens might cause disease states (e.g. gas gangrene)

Smoking impairs the ciliated cells of the middle portion of the respiratory tract. Many analgesic drugs (painkillers) impair peristalsis (the churning motion of the digestive tract). The result of either of these activities leads to an increased risk of infection in their respective areas. Why? Chemicals in cigarette smoke and the chemicals in painkillers impair our immune systems, making us generally more predisposed to infections (regardless of the tissue area). Chemicals in cigarette smoke and the chemicals in painkillers impair the ability of our immune system cells to move into areas that are infected. As such, they can't perform their job of eliminating microbes as well as they should and infections result more easily. The actions of the cells in these areas help to propel pathogens out of the area, serving as a part of the physical barrier system. When they are impaired/slowed, bacteria and other pathogens have an easier time adhering to the tissues in the area and causing an infection. Ciliated cells also line the digestive tract, and these cells secrete strong natural antibacterial compounds. When they are impaired, bacteria can more easily infect these areas. The actions of the cells in these areas help to propel pathogens into the stomach, where they are destroyed by acid. When they are impaired/slowed, bacteria and other pathogens do not get destroyed by and can cause an infection.

The actions of the cells in these areas help to propel pathogens out of the area, serving as a part of the physical barrier system. When they are impaired/slowed, bacteria and other pathogens have an easier time adhering to the tissues in the area and causing an infection.

A 37-year-old man is admitted to the hospital where you work with dizziness, dry mouth, blurred vision, and slurred speech. In addition, he reports abdominal symptoms, including pain, nausea, vomiting, and diarrhea. The patient's family tells you that he was recently at a church potluck, where he ate bread, streak, homemade tomato-beef stew and home-canned green beans. Based on the patient's signs and symptoms, the physician with whom you are working suspects botulism, and the patient is immediately given botulinum antitoxin. He develops breathing difficulty and is placed on a mechanical ventilator.A stool sample from the patient is cultured, and confirms the presence of Clostridium botulinum. In addition, the stool sample as well as leftover stew and green beans are tested for botulinum toxin—toxin is present in the green beans and in the stool sample, but not in the stew. These results confirm the diagnosis of foodborne botulism. You are one of the RNs taking care of the patient—he remains in the hospital for several weeks, before being discharged. How did the antitoxin help the patient? The antitoxin is a mixture of antibodies that bind to and thereby neutralize the different serotypes of botulinum toxin. The antitoxin binds to and destroys Clostridium botulinum cells, preventing them from releasing botulinum toxin. The antitoxin stimulates phagocytes to neutralize botulinum toxin type A. The antitoxin contains antibodies to botulinum toxin type A, which is the type that causes illness. The antitoxin is a mixture of antigens that bind to and thereby neutralize the different serotypes of botulinum toxin.

The antitoxin is a mixture of antibodies that bind to and thereby neutralize the different serotypes of botulinum toxin. This is from a scenario question in the text! p.811 - this is the first line of the first discussion question answer. :)

A sample must contain many microorganisms in order to see any using microscopy techniques. Why?

The area on a microscope slide is very large compared to the relative size of microbes, and you take a very small amount of your sample to place on the slide surface. Therefore, you must have a large number of microbes initially in the culture to increase the chances that you'll come across one or more when looking at the slide surface through the magnifying lenses of the microscope.

Why is oil not degraded when in a natural habitat underground, yet susceptible to bioremediation in an oil spill? a) The bacteria can't be given the right amounts or types of nutrients to foster an increase in their number deep underground. Nearer the surface, human intervention can increase the factors that will raise the microbe quantity. b) It IS degraded underground-but it happens at a much slower rate because a portion of the cycle is photosynthetic in nature. This process is dramatically increased nearer to the water's surface. c) The high pressure the oil is subjected to underground prevents bacteria from growing and consuming it. d) The bacteria that degrade the oil require a higher than normal salt content, much like what is found in seawater. Underground, they lack this salt level.

The bacteria can't be given the right amounts or types of nutrients to foster an increase in their number deep underground. Nearer the surface, human intervention can increase the factors that will raise the microbe quantity. p.797 - petroleum-degrading bacteria are naturally present in seawater, but they degrade oil at a very slow rate because the low levels of certain nutrients, including nitrogen and phosphorus, limit their growth. To enhance bioremediation of oil spills, a fertilizer containing these nutrients - and which adheres to oil - was developed. When this fertilizer is applied to an oil spill, microbial growth is stimulated, leading to at least a threefold increase in the speed of degradation. - for oil spills - biostimulation is better than bioaugmentation.

Rickettsia rickettsii are readily phagocytized by macrophages. Why doesn't this control the infection and prevent the development of Rocky Mountain spotted fever? A. The bacteria interfere with the development of antibodies. B. The bacteria are toxic for T cells and prevent the development of cell-mediated immunity. C. The bacteria escape from the phagosome and reproduce in the cytoplasm of the phagocyte. D. The bacteria are unable to inactivate complement.

The bacteria escape from the phagosome and reproduce in the cytoplasm of the phagocyte. Ch 22

What is NOT a reason why an abscessed wound might not respond to antibiotic treatment? A. The bacteria within the abscess have ceased replicating, and many antibiotics require actively replicating cells in order to be effective. B. The blood vessels that would bring the drug to the area have been destroyed or clogged with clots, preventing the drug from getting to the microbes in the abscess. C. The bacteria in an abscess have all acquired multi-drug antibiotic resistance, so any drugs that are used will have no effect. D. The chemical composition of the pus in the abscess often inactivates antibiotic drugs, making them ineffective.

The bacteria in an abscess have all acquired multi-drug antibiotic resistance, so any drugs that are used will have no effect. Factors why microorganisms in abscesses often are not killed by antimicrobial agents is because - the microorganisms stop dividing, of the chemical nature of the pus AND of the lack of blood vessels. Correct

Why are secondary infections a common complication of measles? A) The causative agent damages the keratinized epidermal layer. B) The causative agent severely suppresses the immune system. C) Phagocytes are killed by the causative agent of measles. D) The causative agent damages the respiratory mucous membranes. E) The causative agent prevents B cells from producing protective antigens.

The causative agent damages the respiratory mucous membranes.

Which statement about bacterial vaginosis is FALSE? BV in pregnant women should be treated because it may cause premature birth. Women voluntarily inoculated with cultures of Gardnerella vaginalis, and members of the genera Mobiluncus, Prevotella, and Peptostreptococcus do not always develop BV. The causative agent of the disease is Gardnerella vaginalis that interacts antagonistically with members of the genera Mobiluncus, Prevotella, and Peptostreptococcus. Women with BV can be diagnosed by the presence of clue cells and a fishy vaginal odor caused by metabolic byproducts of anaerobic bacteria. Low numbers of G. vaginalis, and members of the genera Mobiluncus, Prevotella, and Peptostreptococcus can occur in vaginal secretions of healthy women.

The causative agent of the disease is Gardnerella vaginalis that interacts antagonistically with members of the genera Mobiluncus, Prevotella, and Peptostreptococcus.

Which statement regarding the pathogenesis of necrotizing fasciitis is INCORRECT? The causative agent produces SPE A, a superantigen that causes helper T cells to release large amounts of cytokines, leading to toxic shock. SPE B is a protease produced by S. pyogenes that causes tissue death and breakdown, leading to fluid accumulation in the area and intense swelling. M protein-fibrinogen complexes causes neutrophils to release inflammatory molecules that increase vascular permeability and lead to a blood pressure drop. The causative agent sheds antibodies that attach to fibrinogen and recruit natural killer cells that destroy muscle fascia through ADCC. A hyaluronic acid capsule and M protein allow the causative agent to evade the immune responses.

The causative agent sheds antibodies that attach to fibrinogen and recruit natural killer cells that destroy muscle fascia through ADCC. Correct

Which statement regarding meningitis is FALSE? If bacterial meningitis is suspected, a person is given broad-spectrum antibiotics immediately, without waiting for identification of the causative organism. The cerebrospinal fluid of people with either viral meningitis or bacterial meningitis is cloudy and opaque rather than being clear and pale yellow in color. Meningitis is a relatively rare disease, because it is very difficult for the causative agents to cross the blood-brain barrier. Bacterial causative agents of meningitis are transmitted through respiratory droplets, while viral agents of the disease may also be transmitted via the fecal-oral route. Vaccines protect against the most important types bacterial meningitis but prevention of viral meningitis depends on increased hand washing and avoiding crowded areas.

The cerebrospinal fluid of people with either viral meningitis or bacterial meningitis is cloudy and opaque rather than being clear and pale yellow in color. Viral = looks clear - aseptic meningitis

Why can't botulism be transmitted from person to person? The disease is caused by eating spores that germinate into vegetative cells that produce toxin. Botulism is a toxin-mediated blood-borne disease AND people with botulism die before they can transmit the disease to others. The disease is caused by eating spores that germinate into vegetative cells that produce toxin AND people with botulism die before they can transmit the disease to others. The disease is caused by ingestion of food contaminated with botulinum toxin AND the disease is caused by eating spores that germinate into vegetative cells that produce toxin. The disease is a toxin-mediated disease; it is caused by ingestion of food contaminated with botulinum toxin produced under anaerobic conditions by Clostridium tetani.

The disease is caused by ingestion of food contaminated with botulinum toxin AND the disease is caused by eating spores that germinate into vegetative cells that produce toxin. Check out that last choice. C. tetani? That's tetanus. This is C. botulinum.

Which one of these statements about chlamydial genital infections is false? a) The incubation period is usually shorter than in gonorrhea. b) Infected cells develop inclusion bodies. c) Pelvic inflammatory disease (PID) can be complicated by infection of the surface of the liver. d) Tissue damage largely results from cell-mediated immunity. e) Fallopian tube damage can occur in the absence of symptoms.

The incubation period is usually shorter than in gonorrhea. - gonorrhea incubation period is 2-5 days. - chlamydia incubation period is 7-14 days

Typhoid fever differs from salmonellosis in that in typhoid fever A) The microorganisms become invasive. B) The symptoms are due to an exotoxin .C) The symptoms are due to infection of the gallbladder. D) The classic symptom is diarrhea. E) Chemotherapy is highly effective.

The microorganisms become invasive.

Why is it not possible to culture Treponema pallidum in vitro? The organism is highly motile and requires actin from host cells to make tails for movement AND the organism is Gram-negative. The organism obtains most of its essential macromolecules from the host AND the organism is an obligate intracellular parasite. The organism is an obligate halophile AND the organism is highly motile and requires actin from host cells to make tails for movement. The organism lacks mitochondria and obtains all of its essential macromolecules from the host AND the organism is Gram-negative. The organism is an obligate intracellular parasite AND the organism is a psychrophilic anaerobe.

The organism obtains most of its essential macromolecules from the host AND the organism is an obligate intracellular parasite. p.747 - T. pallidum does not replicate in vitro. The organisms lacks certain metabolic abilities and does not have enzymes needed for the TCA cycle or electron transport chain for can not generate much ATP. It gets most of its essential macromolecules from the host.

Which of the following is NOT involved in the pathogenesis of RMSF? A) The pathogen releases an A-B toxin that attaches to receptors on TH cell cytoplasmic membranes, leading to destruction of those cells. B) Endotoxin released into the bloodstream from the rickettsial cell walls results in systemic inflammation. C) Disseminated intravascular coagulation that leads to organ damage may occur following release of endotoxin into the bloodstream. D) Host cells rupture following damage caused by bacterial cells propelling themselves from one cell to another. E) Vasculitis leads to clotting and small areas of necrosis, causing a hemorrhagic skin rash and damaging tissue in the brain, heart, kidneys, and other organs.

The pathogen releases an A-B toxin that attaches to receptors on TH cell cytoplasmic membranes, leading to destruction of those cells. Pathogenesis • Bacteria taken up by capillary endothelial cells, possibly by forced endocytosis • Bacteria leave phagosome, multiply in cytoplasm, nucleus - Bacteria coat themselves with actin, use to move into adjacent host cells, cause extensive membrane damage - Cells rupture, release rickettsias into bloodstream - Infection of walls of blood vessels causes inflammatory reaction, leads to clotting in blood vessels - Produces small areas of necrosis, yields skin rash • Clotting throughout body damages kidneys, heart • Release of lipopolysaccharide from cell wall into bloodstream causes shock, generalized bleeding due to disseminated intravascular coagulation

Who would have larger numbers of bacteria living on the surface of their skin—a person living in the tropics or in the desert, and why? A) The tropics would provide more shade, so the surface of the skin wouldn't be exposed to high levels of ultraviolet radiation. This would protect the bacteria on the skin, and they would have higher numbers due to this shading effect. B) The very low humidity of the desert would lead to rapid evaporation of sweat and sebum from an individual's skin. Bacteria need these secretions for a nutrient source. Without them, bacteria would be found in much lower numbers on the skin of a person in the desert than the skin of the person in the tropics. C) The constant secretion of high amounts of sweat would produce a highly salty environment on the skin of a person in the tropics. This would provide a local environment that would be too hostile for microbes to survive, so the number of microbes on the skin of the person in the tropics would be lower than that of the person in the desert. D) The constant secretion of large amounts of sweat would wash bacteria off of the skin of the person in the tropics. As such, the person in the desert should have much more bacteria on their skin than the person in the tropics would. E) The very high humidity of the desert would lead to slow evaporation of sweat and sebum from a person's skin. Bacteria thrive on these secretions, using them for a nutrient source. Thus bacteria would be found in much higher numbers on the skin of a person in the desert than the skin of the person in the tropics.

The very low humidity of the desert would lead to rapid evaporation of sweat and sebum from an individual's skin. Bacteria need these secretions for a nutrient source. Without them, bacteria would be found in much lower numbers on the skin of a person in the desert than the skin of the person in the tropics. I choose the tropics. A is goofy - wrong C - skin is left salty when sweat evaporates - which it would in the DESERT - wrong D - those bacteria are stuck on - not getting washed off with sweat. wrong E - desert doesn't have high humidity - wrong

Why does it take more than a week before a mosquito that has just become infected with yellow fever virus can transmit the disease? Mosquitoes only feed once a week, which limits their ability to transmit the disease rapidly. The mosquito actually transmits the virus almost immediately. It takes a week for the virus to multiply in the gut of the host before it causes any ill effects in that host. The virus multiplies in the gut of the mosquito, but then needs to migrate to the mouth of the insect in order to infect a new human being after a new bite. This migration takes time. Mosquitoes that acquire yellow fever virus become ill for a week thereafter. They have to recover before they can feed again and transmit the virus to the next host. The virus must replicate in the gut of the mosquito before it can reach high enough numbers for transmission to a new human.

The virus must replicate in the gut of the mosquito before it can reach high enough numbers for transmission to a new human. Correct

Which of the following are characteristics of fungi?

Their cell wall contains chitin AND they use nutrients from dead or decaying organic matter.

Protozoans classification is based on?

Their means of locomotion.

Your sister recently had her first child, a boy who she names Owen. Unfortunately, Owen seems to get sick really often, and after some investigation, he is diagnosed with an extremely rare genetic disorder called leukocyte adhesion deficiency (LAD). People with LAD suffer from repeated bacterial infections because their leukocytes have adhesion defects—they cannot adhere to the endothelium and as a result are unable to leave the blood vessels to move into tissues containing microbial invaders. Without this inflammatory response, the person experiences severe tissue infections and frequently has neutrophilia (high levels of neutrophils in the blood). You help your sister understand this disease, using your knowledge of the immune system. Which of the following is NOT true of neutrophils? - Their granules contain many enzymes and antimicrobial substances that help destroy engulfed materials. - They are the first leukocytes recruited to the site of an infection. - They are the most numerous and important granulocytes of the innate responses. - Their their numbers significantly decrease during most bacterial infections. - They are phagocytic and efficiently engulf and destroy bacteria and other material.

Their their numbers significantly decrease during most bacterial infections. - they significantly increase actually.

Please identify the INCORRECT statement regarding bacterial diseases of the lower gastrointestinal system. Cholera toxin—A-B toxin that causes intestinal cells to secrete HCl. Pseudomembranous colitis may develop in a person with Clostridium difficile infection. There are more than 2,400 serotypes of Shigella, based on differences in their O, H, and K antigens. Shiga toxin-producing E. coli may cause hemolytic uremic syndrome in an infected person. Of the four pathogenic strains of Shigella, S. dysenteriae is the most virulent.

There are more than 2,400 serotypes of Shigella, based on differences in their O, H, and K antigens. p.643 - It's salmonella that has this, not shigella Correct

Some arthropods cause disease even when they do not act as a vector. Which of the following is an example of this?

These are all examples of the situation described.

Your patient had abdominal surgery three days ago. She seemed to be doing well after the surgery, but today the skin around the surgical incision is red and swollen. In addition, your patient has a high fever and is complaining of muscle aches. She also has a rash and has diarrhea. You realize that her wound has become infected, despite your best efforts as a nurse, and you are concerned that she has developed toxic shock syndrome. You explain the details of this type of infection to the patient's family.You are extremely concerned, because the laboratory reports that the organism cultured from your patient's wound is capable of digesting collagen and fibronectin. Why are you worried about this? This would indicate that the bacterium is a lysogen, containing viral DNA conferring new characteristics on the organism. Bacteriophages pose an additional health risk to your patient. These are structural components of tissue. Their digestion means that not only could the organisms spread easily, but also important structures such as tendons could be destroyed. This would indicate that the organism is capable of forming biofilms and it is very difficult to eliminate biofilms once they have developed, because of resistance. These are structural components specifically of the heart. Their digestion means that the patient is at risk of heart failure, which will prove fatal. This would indicate that the pathogen has the ability to break down activated complement proteins, thereby avoiding the innate immune defenses and allowing it to persist.

These are structural components of tissue. Their digestion means that not only could the organisms spread easily, but also important structures such as tendons could be destroyed. Correct

Mycoplasma genitalium is an emerging cause of genital tract infections in men and women worldwide. The organism was discovered and identified only in the early 1980s. Rising incidence and increasing resistance to antimicrobial medications are problematic. You are asked by your nursing professor to prepare a short presentation on this emerging disease. The questions that follow are those that you anticipate that being asked by your classmates. Select the statement that applies to Mycoplasmas. These bacteria are intrinsically resistant to β-lactam antibiotics. Mycoplasmas resist the effects of β-lactam drugs by synthesizing a β-lactamase. These bacteria have mycolic acids in their cell wall. These organisms have a rapid generation time. These organisms all contain a terminal organelle.

These bacteria are intrinsically resistant to β-lactam antibiotics. p.744/745 - mycoplasmas do not have a cell wall. WHICH MEANS THEY HAVE INTRINSIC RESISTANCE TO PENICILLIN/BETA-LACTAM ANTIBIOTICS. - they have a long generation time - the organism produces a complex cytoplasmic projection called a TERMINAL ORGANELLE at one end of the cell - Mycoplasma genitalium does, yes, but not Mycoplasmas in general. Emerging Disease

How do bacteriocins benefit bacteria in their natural habitat? They provide a source of nutrients during adverse environmental conditions. They allow bacteria to compete against other bacteria that may attempt to use the same or similar resources. They induce resistance in the bacteria, helping them survive when medications are introduced into their habitat. They are protective structures that allow bacteria to survive adverse environmental conditions. They protect the bacteria from the effects of antibiotics in the environment.

They allow bacteria to compete against other bacteria that may attempt to use the same or similar resources.

You are a nurse on a post-surgical ward. One of your patients is an elderly woman who has just had her gallbaldder removed. When you go into her room to check on her, you find that she has significant diarrhea, as well as fever and abdominal pain. You suspect that she has Clostridium difficile infection (CDI), and send a sample of her feces down to the hospital lab for analysis. The test result indicates that your patient does indeed have CDI. You make your patient comfortable and answer her questions about her illness.You tell your patient that her CDI is caused by a bacterium belonging to the genus Clostridium. Which of the following is true of all Clostridia? They are Gram-negative, rod-shaped, endospore-forming obligate aerobes. They are Gram-positive, rod-shaped, endospore-forming obligate anaerobes. Clostridial infections only occur if a state of dysbiosis exists in the host. They are flagellated and always cause significant inflammation. They are encapsulated, flagellated, toxin-producing aerobes.

They are Gram-positive, rod-shaped, endospore-forming obligate anaerobes. p.647

Describe the characteristics of fungi.

They are aerobic or facultatively anaerobic.

What are lichens?

They are an association of algae and fungus.

Describe the characteristic of phytoplankton.

They are free-floating, photosynthetic organisms found in marine environment.

What do algae contribute to the environment?

They are major producers of oxygen.

What are aflatoxins?

They are produced by Aspergillus, are possible carcinogens, may be found in peanuts, and are toxins.

Sarcoptes scabiei transmit what disease and how?

They are responsible for scabies.

What is a Nematode?

They are roundworms and are found in the gastrointestinal tract or the blood.

If an immunodeficient person is exposed to VZV and belongs to a non-immune population, how can that person be protected from developing chickenpox? A) They can be passively immunized with zoster hyperimmune globulin. B) They can be actively immunized with the MMRV vaccine. C) They can be passively immunized with the MMRV vaccine. D) They can not be protected at all—they will get the disease. E) They can be given prophylactic antibiotics.

They can be passively immunized with zoster hyperimmune globulin. • Immunocompromised individuals at risk of severe VZV infections; partial protection achieved by passive immunization with ZVIG (hyper-immune globulin with high concentrations of antibody to VZV)

Which is true about prions? A. They cause a degenerative brain condition. B. They are naked pieces of RNA. C. They are made of DNA and protein. D. They are a normal brain protein that has folded differently. E. They cause a degenerative brain condition AND they are a normal brain protein that has folded differently.

They cause a degenerative brain condition AND they are a normal brain protein that has folded differently.

Which of the statements BEST describes tapeworms?

They do not have a digestive system AND they may be transmitted by eating undercooked meat.

Describe tapeworms.

They do not have a digestive system and may be transmitted by eating undercooked meat.

Which is true of MRSA? A. It stands for moxicillin resistant Staphylococcus aureus. B. They have R plasmids. C. They may be sensitive to Synercid. D. All MRSA strains remain sensitive to vancomycin. E. They have R plasmids AND they may be sensitive to Synercid

They have R plasmids AND they may be sensitive to Synercid - saw this one, a little different. MRSA - methacillin resistant staphylococcus aureus.

How do lice and mice transmit disease?

They may be spread by personal contact.

Describe the characteristic of algae.

They often grow in areas where other forms of life may not.

Which characteristic(s) is/are NOT common to C. tetani and C. perfringens? They are obligate anaerobes. They are rod-shaped. They produce A-B toxins. They are Gram-positive bacteria. They are spore-formers.

They produce A-B toxins They are both the rest - they are G+ rod shaped, obligate anaerobes. Clostridium is a spore-former. Correct

The stage of Lyme disease that is characterized by arthritis is the: A. primary. B. third. C. second. D. fourth.

Third Three stages; individual patients may be asymptomatic in one or more - early localized, early disseminated and late disseminated. Early localized disease: At this point, the infection has not yet spread throughout the body. Lyme is the easiest to cure at this stage. Early disseminated: Bacteria are beginning to spread throughout the body Late disseminated: The Lyme bacteria have spread throughout the body and many patients develop chronic ARTHRITIS as well as an increase in neurological and cardiac symptoms.

How do anoxygenic phototrophs benefit form possessing accessory pigments that allow light to be harvested at deeper areas of a fluid environment?

This allows even anaerobic micorbes to conduct photosynthesis, as they can harvest light energy in the deeper areas where oxygen wouldn't be present, use wavelengths of light not absorbed (filtered out) by photosynthetic organisms closer to the surface of the fluid; and don't have to compete with other cells that utilize oxygen in the upper levels for the scarce nutrients in the fluid environment.

The existence of extensive scalded skin syndrome does not indicate that Staphylococcus is growing in all the affected areas. Why not? A) This condition is caused by an exotoxin produced by certain strains of this microbe, and NOT directly by the microbe itself. B) This condition is caused by an endotoxin produced by certain strains of this microbe, and NOT directly by the microbe itself. C) This condition isn't caused by Staphylococcus at all. D) This microbe grows in the blood; endotoxins it releases are transported to the skin, where it causes the observed effect. E) Scalded skin occurs when a person eats exfoliatin-contaminated food, not by an infection.

This condition is caused by an exotoxin produced by certain strains of this microbe, and NOT directly by the microbe itself. Exotoxin (Staphylococcus is gram positive) S. aureus strains that produce toxin called exfoliatin • Only about 5% of strains produce this toxin • One type coded by a plasmid, one chromosomal

Why would you expect acyclovir to be ineffective against latent HSV infections? A. This is an antibacterial drug, and HSV is a virus. B. This is an antifungal drug, and HSV is a virus. C. This drug directly inhibits DNA polymerase activity-cells that are latently infected with HSV have no DNA polymerase activity. D. This drug is a nucleoside analogue-it becomes activated by a viral enzyme that is produced only when the virus is replicating. As such, if the virus is latent, the enzyme to activate the drug is not present (and the drug is ineffective).

This drug is a nucleoside analogue-it becomes activated by a viral enzyme that is produced only when the virus is replicating. As such, if the virus is latent, the enzyme to activate the drug is not present (and the drug is ineffective). Drugs ending in -vir are antiviral.

Why is it not surprising that Staphylococci are the leading cause of wound infections? Staphylococci acquire antibiotic resistance very readily, making them hard to eliminate by pre-surgical antibacterial soaps. As such, they commonly infect surgical wound sites. This genus is readily present as a part of the normal microbiota on most people's skin, so it could easily enter wounds. Staphylococci form endospores, so even with appropriate control measures, there is a high risk that spores will enter a wound and cause infection. Members of this genus are all particularly virulent and highly capable of causing numerous infections. Staphylococci cannot be eradicated from the skin due to multidrug resistance in most members of the genus. Drugs simply don't kill them.

This genus is readily present as a part of the normal microbiota on most people's skin, so it could easily enter wounds.

Why is it not surprising that Staphylococci are the leading cause of wound infections? A) Staphylococci are capable of a higher degree of spontaneous mutation than most microbes. This makes them able to acquire antibiotic resistance very readily, making them hard to eliminate by pre-surgical antibacterial scrubs. As such, they commonly infect surgical wound sites. B) Members of this genus are all particularly virulent and highly capable of causing numerous infections. C) Staphylococci cannot be eradicated from the skin due to multidrug resistance in most members of the genus. Drugs simply don't kill them. D) This genus is readily present as a part of the normal microbiota on most people's skin, so it could easily enter wounds.

This genus is readily present as a part of the normal microbiota on most people's skin, so it could easily enter wounds. Correct

How can the biomass of fungi in soil be greater considering that bacteria are more numerous? a) It depends on where they grow. Fungi generally grow in the most moist areas near the top of soil. This gives them an advantage over bacteria. They may not be as numerous, but this advantage in location and nutrients is what makes them larger than the bacteria (and therefore having more biomass). If the bacteria evolved to take over the top portion of the soil, THEY would have the larger biomass than the fungi. b) This is a matter of size. Fungi, whether unicellular or multicellular, are eukaryotic. As such, they will generally be larger than bacteria (which are prokaryotic). This means that, even with a lower overall number, they will have more total biomass. c) Fungi are capable of incorporating the dead materials around them into themselves as nutrient sources (while bacteria are not). This capacity allows them to greatly increase their biomass per each organism, well beyond what a simple bacterial cell could hold. This capability is what allows them to achieve higher biomass than bacteria. d) Fungi are always multicellular organisms, while bacteria are unicellular. As such, biomass of fungi would always be larger than that of bacteria, since they possess more cells in each individual organism.

This is a matter of size. Fungi, whether unicellular or multicellular, are eukaryotic. As such, they will generally be larger than bacteria (which are prokaryotic). This means that, even with a lower overall number, they will have more total biomass.

Why might Candida albicans become pathogenic in a person receiving antibacterial medications? This is a usual member of the normal microbiota. However, when the bacterial members of the normal microbiota are killed by a broad-spectrum antibacterial drug, this fungal cell type has little to no competition for resources and it quickly overgrows, causing disease. This is a protozoan species and part of the normal microbiota. When the bacterial members of the normal microbiota are wiped out by a broad-spectrum antibacterial drug, this protozoan cell type has little to no competition for resources and can overgrow, causing disease. This fungal organism can actually use the destroyed bacterial cells as a nutrient source and begin to multiply out of control. This can cause a pathogenic state. This is an opportunistic pathogen not normally found in normal microbiota. As such, when the bacterial normal microbiota is eliminated by broad-spectrum antibacterial drugs, this opportunist can adhere to and colonize the area left behind. The person's immune system is severely suppressed when they are on antibiotics. This means that the Candida is able to colonize areas normally protected by phagocytes, and can thrive there, causing disease.

This is a usual member of the normal microbiota. However, when the bacterial members of the normal microbiota are killed by a broad-spectrum antibacterial drug, this fungal cell type has little to no competition for resources and it quickly overgrows, causing disease.

Why might Candida albicans become pathogenic in an individual receiving antibacterial medications? A) This fungal organism can actually utilize the destroyed bacterial cells as a nutrient source and begin to multiply out of control. This can cause a pathogenic state. B) This is an opportunistic pathogen not normally found in normal microbiota. As such, when the bacterial normal microbiota is wiped out by broad-spectrum antibacterial drugs, this opportunist can adhere to and colonize the area left behind. C) This is a usual member of the normal microbiota. However, when the bacterial members of the normal microbiota are wiped out by a broad-spectrum antibacterial drug, this fungal cell type has little to no competition for resources in the affected area. As such, it quickly overgrows and can cause a pathogenic state. D) This is a protozoan species and part of the normal microbiota. When the bacterial members of the normal microbiota are wiped out by a broad-spectrum antibacterial drug, this protozoan cell type has little to no competition for resources in the affected area. As such, it quickly overgrows and can cause a pathogenic state.

This is a usual member of the normal microbiota. However, when the bacterial members of the normal microbiota are wiped out by a broad-spectrum antibacterial drug, this fungal cell type has little to no competition for resources in the affected area. As such, it quickly overgrows and can cause a pathogenic state. Not really a usual member but is found among them.

Got partial credit Roseola, a viral childhood rash. A. Koplik spots characteristic symptom of rabies rhabdovirus infection triggered by pain associated with swallowing water. C. hydrophobia Transmitted via an arthropod insect vector. E. arbovirus A Chlamydia antigenic type causes this serious infection of the eye. D. trachoma Bucchal mucosa membrane lesions typical of paramyxovirus infection. B. exanthema subitum

This is the correct pairing Roseola, a viral childhood rash. = exanthema subitum characteristic symptom of rabies rhabdovirus infection triggered by pain associated with swallowing water. = hyrdrophobia Transmitted via an arthropod insect vector. = arbovirus Bucchal mucosa membrane lesions typical of paramyxovirus infection.= Koplik spots Chlamydia antigenic type causes this serious infection of the eye = trachoma

Explain how Vibrio cholerae causes cholera without apparent damage to the intestinal epithelium. a. This microbe causes destruction of the cellular structures underneath the intestinal epithelium-this is what induces the watery rice-stool characteristic of the illness. This leaves the overlying intestinal epithelium intact. b. This microbe directly invades the intestinal epithelial cells, but does not kill them. Instead, while multiplying inside them, it causes them to secrete large amounts of chloride ions. This induces water to follow by osmosis, resulting in the watery rice-stool characteristic of the illness. c. This microbe attaches to the surface of intestinal epithelial cells, secreting an exotoxin that causes the epithelium to secrete large amounts of chloride ions. This induces large amounts of water to follow by osmosis, resulting in the watery rice-stool characteristic of the illness. d. The inflammatory reaction to the presence of this microbe causes the watery rice-stool characteristic of the illness. Therefore, it's technically the immune response that initiates the disease, although this response is induced by the presence of the microbe on the intestinal epithelium. c. This microbe attaches to the surface of intestinal epithelial cells, secreting an exotoxin that causes the epithelium to secrete large amounts of chloride ions. This induces large amounts of water to follow by osmosis, resulting in the watery rice-stool characteristic of the illness.

This microbe attaches to the surface of intestinal epithelial cells, secreting an exotoxin that causes the epithelium to secrete large amounts of chloride ions. This induces large amounts of water to follow by osmosis, resulting in the watery rice-stool characteristic of the illness. I think: doesn't damage the cell per say, the A-B enterotoxin (cholera toxin) just opens channels to let out electrolytes, water follows by osmosis. Get abundant water diarrhea.

The main vector(s) of Rocky Mountain spotted fever in the western United States is/are ________. A) ticks B) bats AND humans C) humans D) mosquitoes AND ticks E) fleas AND mites

Ticks

Rubivirus (rubella)

Togaviridae RNA virus

You are reviewing for your nursing school entry exam with your study group. Each person in the group agrees to cover one disease. You select toxoplasmosis for your presentation. Toxoplasmosis can be a serious problem for those who are immunosuppressed, as well as for fetuses. Even in healthy people, the immune system may not clear the organism, resulting in a latent infection that can reactivate when immunity declines.Please select the FALSE statement regarding toxoplasmosis. Infection of the fetus, especially during the first trimester, results in miscarriage, stillbirth, or birth defects. Toxoplasmosis is usually asymptomatic among healthy people, although some people develop sore throat, fever, and enlarged lymph nodes and spleen. Toxoplasmosis may cause encephalitis, brain masses, and other nervous system problems in immunodeficient people. Toxoplasmosis occurs in two main forms—tuberculoid and lepromatous—depending on the immune status of the individual. People contract toxoplasmosis by ingesting oocytes discharged in the feces of acutely infected cats or inadequately cooked meat containing tissue cysts.

Toxoplasmosis occurs in two main forms—tuberculoid and lepromatous—depending on the immune status of the individual. That's leprosy.

Which of the following can only be identified using PCR after valve surgery? Escherichia coli Staphylococcus aureus Tropheryma whippelii Staphylococcus epidermidis

Tropheryma whippelii The other 3 are able to be cultivated in vitro. Apparently it causes a systemic disease called Whipple's disease...

All pathways of complement activation follow the same sequence after C3.

True

An abscess is a collection of pus. T/F

True

Borrelia burgdorferi is a microaerophilic spirochete. T/F

True

Both malaria and yellow fever are transmitted by mosquitoes. T/F

True

Diphtheroids are responsible for body odor. T/F

True

Individuals who have recovered from tetanus are not immune to the disease and must be immunized. T/F

True

Inflammation of the membranes covering the central nervous system (CNS) is called meningitis. T/F

True

Lymphocytes are the cells primarily responsible for the adaptive immune response.

True

Many orchids require fungi for seeds to germinate. T/F

True

No eukaryotic organisms have been shown to fix nitrogen without the aid of prokaryotes. T/F

True

Protozoa, algae, and fungus are typically found near the surface of soil. T/F

True

T/F - Lymphocytes are the cells primarily responsible for the adaptive immune responses.

True

T/F Typically, arthropods serve as vectors of disease, while helminths directly cause disease.

True

True or False: The risk of acquiring a sexually transmitted infection (STI) depends on the number of sexual partners a person currently has AND the number of other sexual partners each of their partner(s) have had.

True

The activated sludge method can be stopped by the presence of toxic industrial wastes. T/F

True - the toxic wastes could kill the microbial population. p.787

Urine contains antimicrobial substances. T/F

True Also is very acidic.

Yeast alone cannot convert grain to alcohol. T/F

True Beer production - p.806 yeast lacks the enzymes needed to digest starch, the primary carb of grains. Have to have sprouted or germinated barley to get the enzymes to do so.

Both Francisella tularensis and Mycobacterium tuberculosis are able to survive phagocytosis. T/F

True Brucella sp, Fransicella tularemia and Mycobacterium of TB can.

Typically C. albicans causes no symptoms. T/F

True Candida albicans is the causative agent of Vulvovaginal Candidiasis, an genital tract infection. p.736 - normally, vaginal colonization by Candida albicans causes no symptoms. It is when the normal microbiota balance is disturbed that C. albicans can multiply freely, causing an inflammatory response.

Coagulase-positive S. aureus is often involved in disease. T/F

True So, so true. Somewhere I saw a chart...

Toxic shock syndrome toxins are superantigens. T/F

True TSST-1 is an exotoxin, a superantigen that tricks T helper cells into releasing massive amounts of cytokines (cytokine storm) which causes a massive over reaction of the immune response and you get toxic shock with the huge drop in BP and crash that follows. Is bad.

Sulfur occurs in all living matter primarily as a component of proteins. T/F

True There is another T/F question - same but 'as a component of carbohydrates' - that one if F.

Bites by little children typically cause few problems. T/F

True They don't tend to break the skin. :)

In a malarial infection, only sporozoites can infect the human liver.

True This is why malaria/Plasmodium sp. spread by drug abuse is easier to treat - its just the infection of blood cells, you need the sporozoites from the mosquito bite to infect the liver cells. - bite (sporozoites carried to liver, infect hepatocytes), multiply and change into merozoites that get released into blood stream and infect RBCs, develop into larger motile trophozoite (feeding form), then schizont (reproducing stage) that gives rise to merozoites that get releaesed and infect more RBCs. Some merozoites from gametocytes instead of schizonts and need to be ingested by mosquitoes to become gametes, zygotes and mature into oocysts that release sporozoites into mosquito mouth bits and saliva.

The bacteria associated with hemolytic uremic syndrome is Escherichia coli O157:H7. T/F

True This strain and other STEC (shiga-toxin producing E. coli) can cause HUS.

Trichomonas vaginalis lacks mitochondria. T/F

True Trichomonas vaginalis - motile protozoa with 4 anterior flagella and 1 posterior flagella attached to an undulating membrane. - has a slender, posteriorly protruding rigid rod called an axostyle that is used for attachment. - jerky motility - has no mitochondria - has an hydrogenosome for glucose metabolism and respiration. It produced hydrogen gas - frothy discharge.

Viral meningitis is much more common than bacterial meningitis. T/F

True p. 697 - Viral meningitis occurs more often than bacterial meningitis, but it is typically less severe and causes little permanent damage in those with normal immunity. - CSF in people with viral meningitis is clear, contains numerous lymphocytes. NO bacteria present so viral meningitis = aseptic meningitis.

T or F: The saliva of asymptomatic carriers of herpes simplex is commonly infectious.

True p.635 - even the saliva of asymptomatic people can be infectious, posting a risk to dentists and other healthcare workers.

Campylobacter gastroenteritis is the leading cause of foodborne illness in the United States. T/F

True p.646 "Campylobacteriosis is a leading bacterial diarrheal illness in the united states...numerous foodborne and waterborne outbreaks of C. jejuni have been reported (most are sporadic)."

All cases of paralytic polio acquired in the U.S. between 1980 and 1999 were caused by Sabin's polio vaccine. T/F

True p.708 Figure 26.9 - Wild poliovirus was eliminated from the United States by 1980...polio cases acquired in the United States after 1980 were caused by vaccine-derived poliovirus (VDPV) - which can result from the Sabin (OPV) vaccine.

Herpes simplex enhances the spread of AIDS. T/F

True p.751 Table 27.12 - herpes simplex increases the risk of contracting HIV - under Epidemiology.

The infectious dose for cholera is much larger than that for shigellosis. T/F

True shigella not easily killed by acid so needs 10-100 cells but cholera needs a lot because it is killed by acid. Correct!

T/F Recently diatoms have been found to be associated with paralytic shellfish poisoning.

True.

African Sleeping Sickness Chayas' disease

Trypanosoma protozoa (euglenozoa)

Two people are brought to the Emergency Department where you work. The first patient reports fever, abdominal pain, vomiting, and diarrhea. The second patient appears extremely ill. Her family tells you that she also had gastrointestinal signs and symptoms a few days ago. She has now developed severe skin blistering, has a very high fever and is only semi-conscious. When you question the first patient and the family of the second patient, you discover that they know each other and that they have all recently returned from a beach vacation in a major city on the U.S. Gulf Coast. The group tells you that the enjoyed a raw oyster dinner on the last day of their summer vacation. They think that both patients are probably just allergic to seafood, but given the severity of the signs and symptoms of your patients, you suspect that they may have Vibrio vulnificus infection. Lab tests confirm your suspicion. You give the families information on this disease.People with chronic liver disease frequently have elevated levels of iron in their blood. Iron is needed by V. vulnificus for growth. How does it acquire iron from the host? V. vulnificus produces a hemolysin that lyses blood cells to release iron-containing hemoglobin. V. vulnificus produces a hemolysin that lyses white blood cells to release iron-containing hemoglobin AND it produces transferrins that allow it to scavenge iron. V. vulnificus produces a hemolysin that lyses white blood cells to release iron-containing hemoglobin AND it produces lactoferrins that allow it to scavenge iron. V. vulnificus produces a hemolysin that lyses blood cells to release iron-containing hemoglobin AND it produces siderophores that allow it to scavenge iron. V. vulnificus produces siderophores that allow it to scavenge iron.

V. vulnificus produces a hemolysin that lyses blood cells to release iron-containing hemoglobin AND it produces siderophores that allow it to scavenge iron. p.674 Correct

Which statement about the cardiovascular and lymphatic systems is NOT true? The cardiovascular and lymphatic systems are typically sterile. Phagocytes in spleen red pulp remove aging or damaged RBCs, bacteria, and other antigens from the blood. Blood is exposed to immune cells as it passes through the spleen. Veins carry blood away from the heart while arteries carry blood towards the heart. Inflammation of lymphatic vessels is called lymphangitis, which presents as a red streak from an infection site toward the nearest lymph node.

Veins carry blood away from the heart while arteries carry blood towards the heart. Arteries carry blood away, veins carry blood to the heart. Correct

Which of the following is NOT a vector?

Water

The most immediate and important treatment needed to prevent death in cholera victims is: A. Water and electrolyte replacement B. Antimicrobials C. Antitoxin D. Surgery E. None of the choices is correct

Water and electrolyte replacement

Most gastrointestinal infections are treated with: A) Antitoxin. B) Penicillin. C) Water and electrolytes. D) Quinacrine. E) Thorough cooking.

Water and electrolytes

Why might it be more difficult to prepare a vaccine against noroviruses than against rotaviruses? A) We haven't been able to culture noroviruses in a lab setting yet. Without a starting culture, we can't create a vaccine. B) Noroviruses are RNA viruses, where rotaviruses are DNA viruses. RNA viruses mutate far more easily than DNA viruses, so we COULD make a vaccine, but it would be rendered useless fairly quickly as the virus mutates. C) We lack a proper culturing method for large-scale production of target cells for norovirus, whereas we have such a system for the target cells of rotaviruses. Without a system to get large numbers of target cells, we can't produce a vaccine. D) Norovirus is much more infectious than rotavirus. As such, it's much harder to work with safety. This makes production of a vaccine too dangerous and unpredictable.

We haven't been able to culture noroviruses in a lab setting yet. Without a starting culture, we can't create a vaccine.

Why is the immunization for rubella important for both boys and girls? A) As this is a sexually-transmitted disease, vaccinating girls AND boys early in their lives prevents the spread of the disease in adulthood. B) We want to achieve herd immunity, and there's no reason why only girls would be infected by this virus. By vaccinating the entire population, we achieve a higher degree of herd immunity than is possible by vaccinating only girls. C) Only boys get this disease, and one of the common complications of it is sterility, so they should be vaccinated against this virus. D) While this disease is quite very symptomatic in girls, it is completely asymptomatic in boys. Boys can be carriers and transmit it easily to girls without realizing it. This can lead to large-scale and dangerous outbreaks in the female population. E) Although boys get this disease, it doesn't cause any health problems for them, so they do not need vaccination. If girls get the disease, it almost always leads to encephalitis, so they should be protected by vaccination.

We want to achieve herd immunity, and there's no reason why only girls would be infected by this virus. By vaccinating the entire population, we achieve a higher degree of herd immunity than is possible by vaccinating only girls. Weird question.

Which symptom is least likely to occur as a result of tertiary syphilis? a) Gummas b) White patches on mucous membranes c) Emotional instability d) Stroke e) Blindness

White patches on mucus membranes. That is Secondary syphilis - picture on patchy tongue on p.746

Which of the following is NOT true of phagocytosis? - Within a phagolysosome, O2 consumption decreases dramatically, allowing an enzyme to produce reactive oxygen species (ROS), which are toxic. - The phagocytic cell binds microbial invaders and engulfs them by means of pseudopodia, internalizing them in a phagosome. - The phagolysosome releases undigested debris to the outside of the cell by fusing with the phagocyte's cytoplasmic membrane. - If a phagocyte encounters something too large to engulf, it releases its toxic contents as a means of destroying that invader. - Phagocytic cells are recruited to the site of an injury by chemoattractants such as complement C3b.

Within a phagolysosome, O2 consumption decreases dramatically, allowing an enzyme to produce reactive oxygen species (ROS), which are toxic.

Why would all protozoa be expected to require large amounts of water in their habitats?

Without being in water, they would quickly dehydrate (due to their small size) and die.

Bacillus anthracis

Wool sorter's disease is caused by _______ ________

The plague bacillus is known as Plasmodium vivax. Pneumocystis carinii. Streptococcus pyogenes. Yersinia pestis.

Yersinia pestis. Yersinia pestis - member of Enterobacteriaceae - facultative anaerobe, Gram negative rod. - non-motile, grows best at 28 C. - with certain dyes, resembles a safety pin as ends stain darker. - forms biofilms in digestive tract of flea and is transmitted by flea bite.

Would babies need to be immunizedagainst lockjaw (tetanus) if their mother had been immunized against the disease? Why or why not? No-because the mother's IgG antibodies would be passed along through the placenta before birth, protecting the baby from the infection. Yes-because even though maternal IgG antibodies might be passed along through the placenta, they will not last forever. The baby will need to create its own antibody response to be protected against future tetanus. No-because maternal IgG antibodies would be passed along in the breast milk, protecting the baby from infection. Yes-because the maternal response to the tetanus vaccine is to produce IgE antibodies, and IgD antibodies are the predominant type in breast milk. And what if the baby isn't breast-fed? Therefore, even if the baby is breast-fed, it won't receive the mother's protective antibodies.

Yes-because even though maternal IgG antibodies might be passed along through the placenta, they will not last forever. The baby will need to create its own antibody response to be protected against future tetanus. Correct

A young mother brings her three-month old baby into the doctor's office where you work as an RN. The baby is miserable and the mother reports that he has developed a fever, which is why she has brought him to the doctor. She tells you that the child is particularly upset when she changes his diaper and that he seems to have diaper rash, which she attributes to the fact that her son's diaper isn't always changed promptly enough at the day-care facility he goes to daily. In fact, she has recently switched him to another facility. You remove the child's diaper and immediately see that he has a very red rash on his buttocks and genital area. There is also red, scaly skin in the area where his diaper touches his thighs. You suspect that this may be more than just a typical case of diaper rash. Your patient comments that she has noticed the same signs and symptoms on her grandmother who has Alzheimer's diasese and who uses adult diapers. She asks you whether it is possible that her grandmother also has a Candida infection. You tell her A) No. Candida is part of the skin normal microbiota of infants only. This organism is never present on the skin of older adults. B) Yes. Candida is present among the skin normal microbiota and may cause infection in anyone who wears a diaper, regardless of age. C) No. Older adults have an exceptionally strong immune system and so are unlikely to develop a fungal skin infection. D) Yes. The older an individual gets, the more Candida cells accumulate on their skin, so it is only a matter of time before the person develops a skin infection. E) No. Older adults have extremely oily skin. Sebum (skin oil) has natural antifungal properties, so the chances of an adult developing a fungal infection are very small.

Yes. Candida is present among the skin normal microbiota and may cause infection in anyone who wears a diaper, regardless of age. i know it's yes but in an earlier question we were asked which of the following is NOT a part of normal skin microbiota and Candida was the NOT. Oh! It's not part of the normal microbiota but IS present. "Yeast Candida albicans found in normal microbiota" - Can invade deep layers of skin and subcutaneous tissues

Coccidioidomycosis is

a fungal disease AND caused by Coccidioides sp.

Macroscopic algae possess a special structure that acts as an anchor and is commonly called

a holdfast.

Human papillomavirus is A. a small, non-enveloped double-stranded DNA virus. B. a small, enveloped double-stranded DNA virus. C. a large, enveloped double-stranded RNA virus. D. a small, non-enveloped single-stranded DNA virus

a small, non-enveloped double-stranded DNA virus. p752 - Human papillomaviruses are naked, dsDNA viruses of the Papillomamiridae family. More than 100 types of HPVs and the different types are tissue and species specific. Had to look online: Human papillomavirus (HPV) is a small, non-enveloped deoxyribonucleic acid (DNA) virus that infects skin or mucosal cells. The circular, double-stranded viral genome is approximately 8-kb in length.

The bacteria cultured from a soil sample typically represents a very small number of bacteria present in that soil. all the metabolically inactive bacteria. all the metabolically active bacteria. all the bacteria present in that sample. most of the types of bacteria in that soil.

a very small number of bacteria present in that soil.

Very often clinically relevant information may be obtained by examining

a wet mount and the size and shape of the organism.

Clostridium botulinum typically does not produce its toxin under conditions that are anaerobic. alkaline. neutral. acid.

acid It's why high acid things like pickles don't have to worry about Clostridium endospores - too acidic. - Clostridium botulinum is a Gram-positive, rod-shaped, anaerobic, spore-forming, motile bacterium with the ability to produce the neurotoxin botulinum.

A patient presents with bad halitosis, red and swollen gingiva, and several loose teeth. You take a sample from the gum line and after performing a stain, note spirochete bacteria as well as some bacilli. This patient likely has dental caries. periodontal disease. acute necrotizing ulcerative gingivitis. gingivitis. dental caries AND acute necrotizing ulcerative gingivitis.

acute necrotizing ulcerative gingivitis. It's NOT the last choice. Above is correct.

The drug(s) used to treat genital herpes is/are: penicillin. erythromycin. acyclovir. famciclovir. acyclovir AND famciclovir.

acyclovir AND famciclovir. Because both are antivirals and genital herpes is caused by HSV-2 (enveloped, dsDNA virus from family Herpesviridae).

Bioaugmentation: a) adds specific microorganisms to the polluted site. b) is typically done offsite. c) only enhances the growth, onsite, of the resident population of microbes. d) usually utilizes genetically engineered bacteria.

adds specific microorganisms to the polluted site. - 1 of 2 bioremediation strategies. - relies on the activities of microorganisms added to the contaminated material, complementing the resident population. - an example is the activated sludge process used during secondary treatment of wastewater - sludge from previous batch is added to the effluent (liquid) of the new cycle

An organism that has peroxidase and superoxide dismutase but lacks catalase is most likely an A. aerobe B. aerotolerant anaerobe C. obligate anaerobe

aerotolerant anaerbe

The complement pathway that is activated by binding of C3b to cell surfaces is the: A) classical pathway. B) alternate pathway. C) C3 pathway. D) mucociliary pathway. E) All of the answer choices are correct.

alternate pathway The alternative pathway of the complement system is an innate component of the immune system's natural defense against infections. The alternative pathway is one of three complement pathways that opsonize and kill pathogens. The pathway is triggered when the C3b protein directly binds a microbe.

The rumen is found in horses and rabbits. found in carnivores. an offshoot of the colon. an anaerobic fermentation vessel found in herbivores. found in horses and rabbits AND an offshoot of the colon

an anaerobic fermentation vessel found in herbivores.

Leprosy is: A. an infectious disease caused by a bacterium. B. a disease of the blood. C. also known as Hansen's disease. D. described in the Bible. E. an infectious disease caused by a bacterium, also known as Hansen's disease AND described in the Bible.

an infectious disease caused by a bacterium, also known as Hansen's disease AND described in the Bible. Mycobacterium leprae - acid-fast, aerobic, rod-shaped, grows very slowly with a generation time of 12-13 days and likes the slightly cooler temps of extremities. - has not been grown in the absence of living cells, DNA library in E. coli. Grows in armadillos, mice and monkeys.

Truffles are a mycorrhizae. are a mushroom. are a lichen. form a sheath around a root of an appropriate tree. are a mycorrhizae AND form a sheath around a root of an appropriate tree.

are a mycorrhizae AND form a sheath around a root of an appropriate tree

Synthetic compounds are most likely to be biodegradable if they are chemically similar to naturally occurring substances. are present in very large amounts. have three chlorine atoms per molecule. are chemically different form naturally occurring substances. None of these is true.

are chemically similar to naturally occurring substances. This question is from the multiple choice questions at the end of the chapter.

Heterocysts

are found in nitrogen-fixing cyanobacteria AND are used to protect nitrogenase.

Eutrophic waters are nutrient rich AND may become hypoxic. contain 3.5% salt AND are alkaline. lack phosphate, nitrate, and iron AND contain 3.5% salt. are nutrient rich AND contain 3.5% salt. may become hypoxic AND lack phosphate, nitrate, and iron.

are nutrient rich AND may become hypoxic.

Pseudomonas

are resistant to many disinfectants and antimicrobials AND are mostly harmless except for the opportunistic P. Aeruginosa

Pseudomonas

are resistant to many disinfectants and antimicrobials and are mostly harmless except for the opportunistic P. aeruginosa.

Azotobacter are the chief suppliers of fixed nitrogen in grasslands. form symbiotic associations with algae. have a very low respiratory rate. are methanogens. are the chief suppliers of fixed nitrogen in grasslands AND form symbiotic associations with algae.

are the chief suppliers of fixed nitrogen in grasslands.

Urinary tract infections: are relatively rare. are the most common nosocomial infections. include puerperal fever .are not considered STDs. are the most common nosocomial infections AND include puerperal fever.

are the most common nosocomial infections AND include puerperal fever. p.730. UTI's - Bacterial Cystitis (Bladder Infection) and Leptospirosis are under the heading of urinary tract infections. - Bacterial Cystitis is the most common type of UTI and is a frequent HAI.

Fungal spores are a major cause of

asthma

The amount of water available in foods

aw

The best defense against arboviruses is: A.Prophylactic rifampinB.Vector controlC.VaccinationD.Prompt treatment with acyclovirE.All of the choices are correct

b

The bubo of bubonic plague is due to: A. septicemia resulting in disseminated intravascular coagulation and hemorrhaging B. fluid build-up from pneumonia C.erythrogenic toxin D. toxic shock syndrome toxin E. bacteria carried via lymph resulting in inflammation and necrosis of the lymph node

bacteria carried via lymph resulting in inflammation and necrosis of the lymph node

The genital tract infection characterized by an unpleasant odor and an increase in clue cells is: trichomoniasis. bacterial vaginosis. candidiasis. amoebiasis.

bacterial vaginosis. p.735 - BV = - most common vaginal disease of women in their childbearing years. - not -itis because there are no inflammatory changes. - Half of cases are asymptomatic. - Symptomatic BC: thin, grayish-white slightly bubbly vaginal discharge that has a characteristic strong fish-like smell. - causative agent unknown. Clue cells are epithelial cells that have sloughed off the vaginal walls and are covered in bacteria.

HBV is mainly spread by: a. blood. b. blood products. c. semen. d. saliva. e. blood, blood products AND semen.

blood, blood products AND semen. p.653 says "Hepatitis B can be transmitted in body fluids such as saliva, blood, blood products and semen."

HIV may be present in: blood. semen. tears. vaginal secretions. blood, semen AND vaginal secretions.

blood, semen AND vaginal secretions. p.757 - HIV is present in blood, semen and vaginal sections in symptomatic and asymptomatic infections. - common mechanisms of person to person spread are: sex, blood and blood products, vertical transmission (mom to baby - pregnancy, passage through birth canal, breast feeding.)

The spleen, in part, functions to cleanse the lymph. interstitial fluid. cytoplasm. blood.

blood. p.666 - Spleen, left side, behind stomach. Red and white pulp. The red pulp cleans the BLOOD by filtration. Red pulp is also a reserve of monocytes and produces new blood cells in rare situations where the bone marrow is unable to make enough. White pulp - contains B/T lymphocytes - provides an active immune response to microbial invaders.

The chief source of central nervous system (CNS) infections is through the: A. bloodstream. B. genitourinary system. C. skin. D. nerves.

bloodstream p.697 - the bloodstream is the primary source of CNS infection - think about that. The bloodstream is the primary source of CNS infections is why we have the blood-brain barrier.

The central nervous system (CNS) may become infected through the: A. bloodstream. B. nerves. C. extensions from bone. D. ventricles. E. bloodstream, nerves AND extensions from bone

bloodstream, nerves AND extensions from bone p.697 - the bloodstream is the primary source of CNS infection. Hard for infectious agents to cross from the blood stream to the brain because of the blood-brain barrier - special cells lining the capillaries in the CNS. - prevents pathogens from entering nervous tissue except when a rare high [ ] circulates for a long time in the blood. - barrier prevents many meds, including penicillin, from crossing into the CNS unless present in very high concentrations.

Human disease(s) from milk may include amoebiasis AND tuberculosis. brucellosis AND tuberculosis. brucellosis AND anthrax. anthrax AND amoebiasis. tuberculosis AND anthrax.

brucellosis AND tuberculosis.

Mercury in mercurochrome inhibits growth: through competitive inhibition. by oxidizing the sulfhydryl groups in cysteine AND by changing the shape of proteins. by changing the shape of proteins. by oxidizing the sulfhydryl groups in cysteine. through a reversible action.

by oxidizing the sulfhydryl groups in cysteine AND by changing the shape of proteins. example of irreversible non-competitive inhibition

Pasteurization of milk helps to prevent: A.PlagueB.TularemiaC.BrucellosisD.MononucleosisE.Endocarditis

c

The sexually transmitted disease characterized by painful genital ulcers is: chancroid. syphilis. gonorrhea. trachoma.

chancroid Another bacterial STI that, like syphilis, causes genital sores (sores raise the risk of getting HIV if exposed). - disease of commercial sex workers - characterized by painful genital sores called soft chancres. See syphilis has hard, painless chancres but this question say PAINFUL so chancroid. p.748 Chancroid = Haemophilus ducreyi - tastidious, pleomorphic, Gram-negative coccobacillus (short rod) that can be cultivated only on a rich medium such as chocolate agar.

Which of the following diseases may manifest themselves with painful genital ulcers? chancroid rubeola herpes leptospirosis chancroid AND herpes (chancroid AND leptospirosis)

chancroid AND herpes If given option chancroid AND Leptospirosis I would probably choose just chancroid Chancroid - STI - bacterial - genital ulcers = painful, soft chancres Leptospirosis - UTI - bacteria zoonosis - is all about damage to liver and kidneys. Gets in by mucus membranes or wounds and hitches a ride via bloodstream to urinary system. p.750 - herpes - virus - lesions - vesicles burst, producing painful ulcers.

Which animal is used as an early warning system for encephalitis? A. armadillos B. mice C. chickens D. foxes

chickens Viral encephalitis caused by arboviruses - group of enveloped ssRNA viruses transmitted by insects, mites or ticks. - include LaCrosse encephalitis virus, St. Louis and West Nile encephalitis and eastern/western equine encephalitis.

All fungi have ______ in their cell walls.

chitin

Bacterial STIs include

chlamydia, gonorrhea, Mycoplasma genitalium infections, syphilis

The most common reportable STD in the U.S. is: gonorrhea. chlamydial infections. syphilis. AIDS.

chlamydial infection. First is chlamydia, second is gonorrhea. Chlamydia trachomatis - shperical, obligate intracellular Gram-negative bacterium. Two forms: infectious elementary body and multiplying intracellular reticulate body.

The site in a photosynthetic eukaryotic cell where photosynthesis occurs is the

chloroplast.

Vibrio cholerae

cholera gram negative vibrio (curved rod) w polar flagella warm, salty, alkaline water transmission: ingested and gets into intestines fecal/oral, sometimes eaten shellfish large min. infectious dose toxins turn cAMP pumps on to alter electrolytes in lumen of small intestines incubation: a few hours signs/symptoms: sudden onset of vomiting/diarrhea, "rice water stool" treatment: rehydration, antibiotics prevention: sewage treatment, water purification, boil water, cook food

Agrobacterium

contain the Ti plasmid which modifies the growth of plant tissue.

The most common mode of HAV transmission is: A) contamination of food during preparation. B) contamination of food before it reaches a food service establishment. C) blood transfusion. D) contaminated hypodermic needles. E) airborne.

contamination of food during preparation.

Organisms that inhabit soil or water and breakdown and absorb the organic matter of dead organisms are called A. Decomposers B. Primary consumers C. Tertiary consumers D. Secondary consumers E. Quaternary consumers

decomposers

Corynebacterium diphtheriae

diphtheria is caused by ___________ _______ upper respiratory infection; pseudomembrane

Corynebacterium diphtheriae

diphtheria- upper respiratory infection pseudomembrane over pharynx Gram + positive rod, pleomorphic palisades arrangement 0.01 cases per million droplet transmission virulence factors: exotoxin, diphtherotoxin weird ECG patterns due to myocarditis treatment: antitoxin, antibiotics, remove pseudomembrane prevention: vaccine

The transmission of M. leprae is by contact with contaminated blood products. direct human-to-human contact AND contact with infected nasal secretions. contact with infected nasal secretions AND the fecal-oral route. direct human-to-human contact AND the fecal-oral route. the fecal-oral route AND contact with contaminated blood products.

direct human-to-human contact AND contact with infected nasal secretions. p.720

Which of the following E. coli serovars produces a toxin somewhat similar to that produced by Shigella dysenteriae? A. enterotoxigenic B. enteroinvasive C. enteropathogenic D. enterohemorrhagic E. All of the choices are correct.

enterohemorrhagic p.642 - STEC - Shiga toxin-producing E. coli - these strains, also referred to as EHEC or enterohemorrhagic E. coli, produce shiga toxins, a family of functionally identical toxins that includes the toxin of Shigella dysenteriae. In STEC is encoded for by lycogenic conversion.

Antibody-dependent enhancement (ADE), that explains the pathogenesis of dengue hemorrhagic fever and dengue shock syndrome, can best be described as: - failure of pre-existing anti-dengue antibodies to neutralize a second dengue virus serotype, instead facilitating viral entry into host cells. - the bleeding and shock that occur when the heterophile antibodies made in a nonproductive infection cause blood vessel leakiness. - the lysis of productively infected B cells by antibodies made in response to different dengue viruses during a dengue infection. - the production of large amounts of heterophile antibodies by B cells with a nonproductive dengue infection.

failure of pre-existing anti-dengue antibodies to neutralize a second dengue virus serotype, instead facilitating viral entry into host cells. See you have antibodies from first infection, DHF and DSS occur in person who gets a second infection by a different serotype of the virus - Antibody-dependent enhancement (ADE) means the antibodies from the first infection can't stop the seconds so they create immune complexes that actually facilitate viral entry into cells that have Fc receptors (monocytes/macrophages) where the virus replicates freely so ADE leads to increased numbers of infected cells and a high viral load.

The return of carbon dioxide to the atmosphere and nitrogen to the soil is due to the action of

fungi and bacteria

Coccidioides immitis

fungus; Coccidiodomycosis

Pneumocystis jirovecii

fungus; Pneumocystic pneumonia

Aspergillus flavus

fungus; aspergillosis

Staphylococcus aureus toxin is an endotoxin AND heat-sensitive. an exotoxin AND heat-sensitive. heat-stable AND an exotoxin. heat-stable AND an endotoxin. None of the answer choices is correct.

heat-stable AND an exotoxin.

The usual cause of genital herpes is: herpes simplex virus type 1. herpes simplex virus type 2. herpes simplex virus type 3. herpes simplex virus type 4.

herpes simplex virus type 2 p.750 - genital herpes is usually caused by herpes simplex virus type 2 (HSV-2), an enveloped, dsDNA virus from family Herpesviridae. HSV-1 causes cold sores/fever blisters but can also cause genital herpes. They look the same, are 50% same genome but HSV-2 causes more sever genital lesions with a greater frequency of recurrence.

A painful finger infection attributable to herpes virus is known as a(n): finger sore .abrasion lesion. herpetic whitlow. furuncle.

herpetic whitlow. Recurrent cold sores (LIPS) called herpes simplex labialis. Correct

Intestinal botulism, especially in infants, has been linked to ingestion of: A. seafood. B. milk. C. honey. D. applesauce.

honey give no honey to children under 1 year old

The most common of the sexually transmitted viral diseases agents is: human papillomavirus. herpes virus. human immunodeficiency virus. leptospirosis.

human papilloma virus Sexually transmitted human papillomavirus (HPV) strains are among the most common of the STI agents. p.751

Streptococcus pyogenes

impetigo, erysipelas, strep throat, scarlet fever, necrotizing fasciitis, rheumatic fever gram + positive coccus, chains facultative anaerobes parasite in throat, nasopharynx, skin virulence factors: M protein- increases adhesion capsule- hyaluronic acid hemolysins- lyse RBCs C5a protease- reduce chemotaxis reservoir: human

The first host response to a nonspecific tissue injury is described as: A) inflammation. B) reaction. C) antibodies. D) trauma. E) fever.

inflammation

C3a and C5a are involved in inflammation. interferon production. properdin activation. enhancement of phagocytosis. inflammation AND enhancement of phagocytosis.

inflammation AND enhancement of phagocytosis.

A frequent complication of untreated gonorrhea in women is: pelvic inflammatory disease. syphilis. dysuria. vaginal discharge.

inflammatory pelvic disease p.741 - In women, the infection can progress upward through the reproductive tract, casing pelvic inflammatory disease (PID) - N. gonorrhoeae is not motile so perhaps carried to fallopian tubes on swimmers.

Yops proteins, produced by Yersinia pestis, interfere with phagocytosis. activates plasminogen activator. destroys C3b and C5a. promotes apoptosis.

interfere with phagocytosis. p.670 - Yops (proteins) - interfere with phagocytosis and the immune response. - A variety of Y. pestis outer proteins (Yops) are made and delivered by type III secretion systems to the host cell, they disrupt the host cell cytoskeleton, inhibit phagocytosis, block the production of pro-inflammatory cytokines.

Advanced treatment of sewage involves the removal of phosphates and sulfur compounds. is done to promote nutrient enrichment. is done to promote growth of photosynthetic organisms. is done to prevent possible overproduction of algae and other organisms. All of the choices are correct.

is done to prevent possible overproduction of algae and other organisms.

Toxic shock syndrome: is due to exotoxins produced by S. pyogenes. is due to exotoxins produced by S. aureus. may spread from person to person. has a very low rate of recurrence, approximately 1%.

is due to exotoxins produced by S. aureus. Toxic Shock Syndrome -> strains of Staphylococcus aureus that produce toxic shock syndrome toxin-1 (TSST-1) or other related exotoxins (G+ bacteria) p.737

Hydrogen sulfide

is produced when sulfur compounds are used as terminal electron acceptors. may react with iron to produce a black precipitate. is produced by Desulfovibrio. has a rotten egg smell. All of the choices are correct.

The nervous system typically is sterile. contains a small number of viruses. has transient microbiota. contains Gram-positive bacteria. has a small number of normal microbiota.

is sterile

In a cow's udder, milk: contains Lactococcus. contains Lactobacillus. is sterile. is acidic.

is sterile Microbes are introduced as and after the milk leaves the udder.

All of the following statements about staphylococcal toxic shock are true except: a) it can lead to kidney failure. b) the causative organism usually does not enter the bloodstream. c) it occurs only in tampon users. d) the causative organism produces superantigens. e) person-to-person spread does not occur.

it occurs only in tampon users. p.737 - the syndrome can occur after infection of surgical wounds, infections associated with childbirth, and other types of staphylococcal infections.

The toxins involved in intestinal infections typically modify cell physiology resulting in increased secretion of water and electrolytes. kill cells by inhibiting DNA synthesis AND modify cell physiology resulting in decreased secretion of water and electrolytes. kill cells by inhibiting protein synthesis AND modify cell physiology resulting in increased secretion of water and electrolytes. kill cells by inhibiting DNA synthesis AND modify cell physiology resulting in increased secretion of water and electrolytes. modify cell physiology resulting in decreased secretion of water and electrolytes.

kill cells by inhibiting protein synthesis AND modify cell physiology resulting in increased secretion of water and electrolytes.

Mycoplasma

lack peptidoblycan, are the smallest free-living organisms and have sterols in their membranes.

Mycoplasma

lack peptidoglycan, are the smallest free-living organisms AND have sterols in their membrane

Mycoplasma

lack peptidoglycan, are the smallest free-living organisms AND have sterols in their membranes

Trichomonas vaginalis is an unusual eukaryote in that it: has chloroplasts. lacks mitochondria. has flagella. lacks a nuclear membrane.

lacks mitochondria

The unique flavor of yogurt is due to the production of A.lactic acid and acetaldehyde. B.casein and alcohol. C.rennin and coagulated protein. D.curd and whey

lactic acid and acetaldehyde. Possible that this isn't from this course...

The incubation period of rabies is partially determined by: A. age of the patient. B. season of the year that the disease is acquired. C. length of journey from the bite site to the brain. D. the amount of virus introduced into the wound. E. length of journey from the bite site to the brain AND the amount of virus introduced into the wound.

length of journey from the bite site to the brain AND the amount of virus introduced into the wound. p.710 - the length of time before symptoms occur depends on: - location of bite - amount of virus introduced - condition of the host

Mycobacterium leprae

leprosy (Hansen's disease) gram + positive bacterium obligate intracellular organism acid-fast incubates for years, incubation time is 2 weeks (1 cell to 2) affects peripheral nerves and skin virulence factors low warm climate armadillos transmission not from sex or casual contact Target population: immigrants, people who eat armadillos multidrug therapy nerve damage leads to secondary infection permanent disfigurement

The cells primarily involved in all immune responses are the: A) erythrocytes. B) platelets. C) leukocytes. D) lymphocytes. E) chondrocytes.

leukocytes - white blood cells

The major effect of excess water in the soil is to lower the amount of oxygen. increase the level of oxygen. increase the amount of nitrogen. reduce the amount of carbon. reduce the pH.

lower the amount of oxygen

Although endotoxemia affects many organs, the organ most seriously and irreversibly affected is the heart. lung. kidney. spleen.

lung. Also seen as, "Although sepsis affects many organs..." endotoxemia = endotoxin in the blood p.668 - still talking about sepsis - Lungs are seriously and irreversibly damaged by tissue-damaging lysosomal enzymes. Lung damage often results in death even if the infection has been cured.

The small bean-shaped bodies into which the lymphatic vessels drain are the lymph nodes. adrenals. subclavian veins. valves.

lymph nodes.

A visible red streak in an infected hand or foot is referred to as septicemia. bacteremia. lymphangitis. edema.

lymphangitis. p.665 - when a hand or a food is infected, a visible red streak may spread up the limb from the infection site toward the nearest lymph node. This is due to inflammation of one or more lympathic vessels, a condition called lymphangitis.

Which of the following is caused by a protozoan infection? malaria yellow fever tularemia infectious mononucleosis

malaria - protozoa of the genus Plasmodium, transmitted by female mosquitoes of genus Anopheles. - 5 species of Plasmodium involved: P. vivax, P. faciparum, P. malariae, P. ovale and P. knowlesi.

M. leprae may infect: A. mangabey monkeys. B. skunks. C. iguanas. D. armadillos. E. mangabey monkeys AND armadillos.

mangabey monkeys AND armadillos The strain of bacteria found in wild 9-banded armadillos and in human is the same.

Fleas

may transmit Yersinia pestis.

Schizogony

means multiple fissions AND is performed by protozoa.

The MMR vaccine is used to protect against: A) mononucleosis, mange, rubeola. B) measles, mumps, rubella. C) measles, mange, rubeola. D) mononucleosis, mumps, rubella. E) mumps, rubella, varicella.

measles, mumps, rubella.

The membranes that cover the surface of the brain and spinal cord are known as the: A. neurolemma. B. dura mater. C. meninges.

meninges - from out to in - bone, DURA MATER, venus sinus (in some areas of brain), ARACHNOID MATER, subarachnoid space, PIA MATER. Subarachnoid space is where cerebrospinal fluid is. Venus sinus under dura mater is where CSF goes back in to bloodstream. - outer dura is tough, barrier to the spread of infection from bones surounding the CNS. Adheres closelty to the skull and vertebrae, in part of the brain it encloses a blood-filled venous sinus.

An infection of the membranes covering the brain is called: A. encephalitis. B. meningitis. C. arachnitis. D. ventriculitis.

meningitis - inflammation of brain itself = encephalitis - if both meninges and brain are infected = meningoencephalitis.

viral meningitis

meningitis caused by a virus Naked RNA viruses - members of the enterovirus subgroup of the family Picornaviridae - coxsackie viruses (cause chest/throat pain) and echoviruses (cause a rash)

Streptococcus pneumoniae

meningitis is caused by __________ ___________ inflammation of meninges

Geosmins are metabolites that give soil its odor AND produced by Rhizobium. produced by Rhizobium. a strain of Bacillus. metabolites that give soil its odor AND produced by Bacillus. metabolites that give soil its odor. nitrogen-fixing bacteria.

metabolites that give soil its odor. Made by Streptomyces

The anaerobic organisms used in sewage treatment may produce the useful product(s) nitrogen AND carbon monoxide. methane. nitrogen. carbon monoxide. oxygen.

methane

During anaerobic decomposition of organic matter with carbon dioxide as the terminal electron acceptor, the primary gas(es) produced is/are methane AND hydrogen. oxygen. methane. hydrogen. hydrogen sulfide.

methane p.776 - in anaerobic environments, CO2 is used by methanogens. These archaea obtain energy by oxidizing hydrogen gas, using CO2 as a terminal electron accepter, and generating methane (CH4)

Fungi are classified according to their

method of sexual reproduction

Campylobacter bacteria are grown with a CampyPak that produces 5% carbon dioxide and 15% oxygen. This bacterium is __________. A. capnophile B. microaerophile C. aerotolerant anaerobe D. anaerobe

microaerophile A candle jar will get the oxygen down to 17% for a capnophile. Oxygen as low as 15%? That's a microaerophile. Normal FIO2 is 21%.

The environment immediately surrounding an individual cell is called its macroenvironment. microenvironment. microniche. colony.

microevironment Because microorganisms are so small, the microenvironment is difficult to identify and measure. The more easily measured gross environment - the macroenvironment - can be very different from the microenvironment. But there can be zones in the macro where the right micro conditions exist so that you might find microbes you didn't think you'd find. :)

The role that an organism plays in its environment is called its ecosystem. habit. role. position. niche.

niche

The urinary tract above the bladder usually shows: E. coli. S. aureus. P. vulgaris. no bacteria. E. coli AND S. aureus.

no bacteria p.729 - the urine and the urinary tract above the urethra are generally free of microorganisms in both men and women.

The most common agent(s) causing subacute bacterial endocarditis is/are Streptococcus pyogenes. Pseudomonas aeruginosa. normal skin or mouth flora. Escherichia coli.

normal skin or mouth flora. p.666 - Staphylococcus aureau, Staphylococcus epidermidis, enterococci, variety of Streptococcus species including viridans streptococci from mouth - bacteria Correct

The growth stage of the vector that is mainly responsible for transmitting Lyme disease is the mouse. egg. adult. moulter. nymph stage.

nymph stage Correct

It has been estimated that 99% of the intestinal bacteria are

obligate anaerobes.

Which of the following is NOT true? Microorganisms in abscesses often are not killed by antimicrobial agents because of the high level of oxygenation. the microorganisms stop dividing. of the chemical nature of the pus. of innate resistance in the microbes. of the lack of blood vessels.

of the high level of oxygenation. There is no oxygen reaching an abscess. Correct

Symptoms of plague appear in two to three months. one to two years. three to six hours. one to six days.

one to six days. p.669 - signs/symptoms depend on how the disease is acquired - through tick bite = bubonic plague - through inhaling respiratory droplets = pneumonic plague - when the causative organisms spreads via bloodstream = septicemic plague

Sprophytes

organisms that take in nutrients from dead and decaying matter.

Algae often grow in areas where

other forms of life may have difficulty.

What is the advantage of removing phosphates and nitrates from wastewater?

p.789 - advanced treatment is often designed to REMOVE AMMONIA, NITRATES AND PHOSPHATES - compounds that foster growth of algae and cyanobacteria in receiving waters (so remove so to PREVENT POSSIBLE OVERPRODUCTION OF ALGAE AND OTHER ORGANISMS). The concentration of these nutrients is normally low in receiving waters which limits the growth of photosynthetic organisms but add the nutrients and see them multiply, BOD increase and other forms of aquatic life threatened (so advanced treatment IS DONE TO PREVENT NUTRIENT ENRICHMENT of the receiving waters so that doesn't happen).

The chronic inflammatory process involving the gums and tissues around the teeth is called: dental caries. periodontal disease. dental plaque. root caries.

periodontal disease.

Yersinia pestis

plague gram negative rod, zoonosis, fleas from rodents virulence factors: capsule, endotoxin, adhesion, antiphagocytic protein, coagulase History: 500-700AD pandemic, 1300s 1/3 of europe died, 1860's it travelled on boat to US Signs/Symptoms: *Bubonic Plague* enlarged lymph nodes called buboes *Septicemic Plague* this is black plague, intravascular coagulation causing subcutaneous hemorrhaging 50% mortality *Pneumonic Plague* in lungs, very contagious, symptoms within hours, cough with bloody sputum fleas carry: coagulase in flea esophagus blocks swallowing causing flea to bite over and over again Endemic in: CA, UT, AZ, NV, NM Treatment: anti-biotics, quarantine Prevention: keep rodents and fleas under control around human population, personal hygiene

The disease responsible for the death of approximately ¼ the population of Europe from 1346 to 1350 was typhus. pneumonia. influenza. plague.

plague. Crowded conditions and a large rat population had major roles in the spread of plague. Plague is a potential bioterrorism disease, Category A.

Mycorrhizae are a symbiotic association of bacteria and algae. bacteria and virus. fungi and virus. plant roots and fungi.

plant roots and fungi

Naegleria fowleri may cause: A. meningitis. B. primary amebic meningoencephalitis. C. African sleeping sickness. D. AIDS.

primary amebic meningoencephalitis. Amoeba found in warm fresh water and soil. Infected after swimming or diving in natural waters in US. Rare event but usually fatal. Naegleria fowleri - free-living protozoa that is pathogenic for humans. Ameboid trophozoite gives rise to flagellated forms and spherical cysts. Penetrates the skull along the olfactory nerves of the nasal mucosa. Multiples, migrates to brain and eats brain tissue (frontal lobe = yum!). Not transmitted person to person.

Fungi are important in

production of antibiotics, food spoilage, food production, disease of plants.

Trichomonas vaginalis

protist; trichomoniasis (STD)

The ability to exist as either a trophozoite or a cyst is characteristic of many

protozoa

Trichomoniasis is caused by a: A. fungus. B. virus C. bacteria. D. protozoan.

protozoan Trichomonas vaginalis - a motile protozoa with 4 anterior flagella - 1 posterior flagellum attached to an undulating membrane - 1 rigid rod called an axostyle used in attachment to cells - jerky motion - lacks a cyst form to aid in its survival in the environment away from the host - eukaryote with no mitochondria - human parasite spread by sexual contact... and on moist towels.

Pollutant degradation may be enhanced by lowering the temperature. providing dry conditions. maintaining an acidic pH. providing adequate nutrients. All of the choices are correct.

providing adequate nutrients.

Which of the following agents travel up the nerves to penetrate the CNS? A) rabies B) herpes simplex C) tetanus toxin D) polio E) rabies, herpes simplex, AND tetanus toxin

rabies, herpes simplex, AND tetanus toxin

Clostridium can be cultured in an anaerobic incubator or in the presence of atmospheric oxygen if thioglycolate is added to the nutrient broth. The addition of thioglycolate makes the medium __________. A. reduced B. differential C. selective D. enriched

reduced Reduced-serum media are basal media formulations enriched with nutrients and animal-derived factors, which reduce the amount of serum that is needed - not in my text

The principal mode of transmission of rabies is through: A. blood. B. fomites. C. saliva. D. pus.

saliva The primary mode of transmission of rabies to humans is via the saliva of a rabid animal introduced into bite wounds of the skin.

On which of the following foods can Staphylococcus aureus multiply with little competition? Bread and jam Milk Bread Jam Salty ham

salty ham Staphylococcus aureus is salt-tolerant p.801 - S.aureus can grow at a LOWER WATER ACTIVITY THEN MORE SPOILAGE BACTERIA. - S. aureus can grow at a water activity of 0.86. Ham is a common vehicle for S. aureus food poisoning.

A site used for disposal of non-hazardous solid wastes in a manner that minimizes damage to human health and the environment is a(n) sanitary landfill. septic tank. activated sludger. water treatment facility. composter.

sanitary landfill

Staphylococcus aureus

scalded skin syndrome is caused by _________ _____

Where in the body does the latent, non-infectious, non-replicating form of the herpes simplex virus persist? motor neurons red blood cells cranial nerves sensory nerves

sensory nerves Correct

If a compost pile is turned frequently and other conditions are adequate for aerobic digestion, the composting can be completed in 1 month. 1 day. six months. six weeks. three years.

six weeks

The condition that develops on a previously damaged heart valve is called an aneurysm. acute bacterial endocarditis. myocarditis. subacute bacterial endocarditis.

subacute bacterial endocarditis. Slide 8/p.666 - endocarditis - inflammation inside the heart. Condition develops gradually than it is called SUBACUTE BACTERIAL ENDOCARDITIS. Rapid progression = acute bacterial endocarditis

Acute bacterial endocarditis differs from subacute bacterial endocarditis in the suddenness and severity of onset. population affected. resultant damage. development of exotoxin shock.

suddenness and severity of onset. Subacute is gradual and not as bad. Acute is rapid and bad. - so how sudden and how severe.

A DNA similarity of 75% between two organisms

suggests the organisms are very closely related at the species level.

Bite wounds are often serious as they may lead to an infection in which two or more species of pathogen act together to produce an effect that is greater than the sum of effects if each pathogen were acting alone...

synergistic infection Alone, usually harmless; together, produce large numbers of toxins and enzymes including leukocidin, collagenase, hyaluronidase, ribonuclease, various proteinases, neuraminidase, and enzymes that destroy complement system proteins and antibodies; yield synergistic infection

Compared to the first big syphilis epidemic several hundred years ago in Europe, the strains of Treponema pallidum that cause syphilis today: tend to be more virulent. tend to be less virulent. are about equally virulent. cannot be compared.

tend to be less virulent

In which disease is a gumma formed? primary syphilis secondary gonorrhea tertiary syphilis diphtheria

tertiary syphilis In gummatous syphilis, localized areas of tissue damage develop as a result of prolonged inflammatory responses. These lesions, called gummas, are granulomas similar to the tubercles of TB. Chronic and may occur anywhere on the body. p.746

Clostridium tetani

tetanus (lockjaw) gram + positive rod, endospore, anaerobe neuromuscular disease spastic paralysis caused by toxin: tetanospasmin fatality is 10-70% caused by final inhalation, unable to exhale reservoir is soil & animal intestinal tract spores get into wounds painful muscle stiffness and cramps treatment is antitoxin need supportive care in hospital prevention is vaccine DT, DTaP, Td, Tdap given toxoid (modified toxin)

Clostridium tetani

tetanus (lockjaw) is caused by _______ ____

Which of the following is incorrect about water? A. The membrane filter method is a widely used rapid method to test large quantities of water for quality. B. Finding coliforms in high numbers indicates fecal contamination of the water. C. When a water quality test is negative for coliforms, it is considered fit for human consumption. D. There is no acceptable level for fecal coliforms, enterococci, viruses, or pathogenic protozoans in drinking water. E. the (Most Probable Number) MPN specifically detects fecal coliforms.

the (Most Probable Number) MPN specifically detects fecal coliforms.

Two people are brought to the Emergency Department where you work. The first patient reports fever, abdominal pain, vomiting, and diarrhea. The second patient appears extremely ill. Her family tells you that she also had gastrointestinal signs and symptoms a few days ago. She has now developed severe skin blistering, has a very high fever and is only semi-conscious. When you question the first patient and the family of the second patient, you discover that they know each other and that they have all recently returned from a beach vacation in a major city on the U.S. Gulf Coast. The group tells you that the enjoyed a raw oyster dinner on the last day of their summer vacation. They think that both patients are probably just allergic to seafood, but given the severity of the signs and symptoms of your patients, you suspect that they may have Vibrio vulnificus infection. Lab tests confirm your suspicion. You give the families information on this disease. Patients with V. vulnificus infection are given a combination of synergistic antibiotics. This means that: the action of one antibiotic diminishes the effect of the other(s) in the combination. each antibiotic works independently, but does not inhibit the action of the other antibiotic(s) in the combination. the antibiotics given in combination are always broad spectrum medications. the antibiotics work together to produce a stronger effect than each antibiotic on its own. one antibiotic in the combination is always broad spectrum while the other(s) is/are always narrow spectrum.

the antibiotics work together to produce a stronger effect than each antibiotic on its own. Correct

Humans acquire the arbovirus that causes epidemic encephalitis by: A. the fecal-oral route. B. human to human contact. C. the bite of an infected mosquito. D. contaminated food.

the bite of an infected mosquito. But humans are an accidental dead end host. Birds, squirrels, etc - the virus can build up to concentration in blood that can use a mosquito as biological vector. In people it can't ever replicate to a high enough concentration.

Development of a dengue vaccine has proved challenging because: - there are five different viral serotypes, so a multivalent vaccine is needed. - dengue viruses, being RNA viruses, have a high mutation rate. - dengue is transmitted by different species of the Aedes mosquito. - dengue viruses are DNA viruses and can integrate into the host genome.

there are five different viral serotypes, so a multivalent vaccine is needed. 4 serotypes, my book says but I still like the answer. Dengue virus is ssRNA of family Flaviviridae.

High levels of antibodies in patients with SBE tend to be of little use because they make the bacteria clump together and adhere to the clot. the surface antigens change rapidly and become unrecognizable. the antibodies are defective. the antibodies degrade quickly.

they make the bacteria clump together and adhere to the clot. Microbes adhere to heart tissue, multiply, create a biofilm, the clot grows larger around the multiplying organisms, building up a fragile mass. Bacteria get washed off the mass into the bloodstream and pieces of infected clot (septic emboli) can break off. p.667 actually says antibodies will complex with antigens and immune complexes may lodge in body structures and trigger an inflammatory response. Says microbes adhere to heart tissue, multiply, create a biofilm, the clot grows larger around the multiplying organisms, building up a fragile mass. Bacteria get washed off the mass into the bloodstream and pieces of infected clot (septic emboli) can break off.

High levels of antibodies in patients with infectious endocarditis cause inflammation because they may form clots that lead to life-threatening emboli. the surface antigens change rapidly and become unrecognizable. they may form immune complexes that get trapped in skin and eyes. the antibodies initiate type I, type II, or type III hypersensitivities. the antibodies degrade quickly and debris gets trapped in these organs.

they may form immune complexes that get trapped in skin and eyes. p.667 Correct

During which stage of syphilis is the patient non-infectious? first second third fifth

third Tertiary - non-infectious syphilis causes so many signs and symptoms that it is easily confused with other diseases and is often called "the great imitator", its symptoms occur in defined clinical stages.

The major virulence factors of Yersinia pestis are carried on the chromosome. a plasmid. three separate plasmids. nuclear membrane.

three separate plasmids. Nothing in my chapter says anything about plasmids.

African sleeping sickness is transmitted by: A. ticks. B. black flies. C. Aedes mosquitoes. D. tsetse flies.

tsetse flies African Trypanosomiasis (African sleeping sickness) - Trypanosoma brucei - flagellated protozoa, slender with wavy undulating membranes and an anteriorly protruding flagellum - biological vector = tsetse fly of genus Glossina Two subspecies, morphologically identical. Trypanosoma brucei rhodesiense and gambiense. R is acute disease and G is chronic disease. - grow in bursts as a glycoprotein is expressed on protozoan's surface and new antibiodes must be made to it.

Mycobacterium tuberculosis

tuberculosis Gram + positive rod aerobe slow generation virulence factors: complex waxes, cord factor, lessened destruction by macrophage history: found in mummies, consumption/vampire lore 1/3 of the world is carrier signs/symptoms: primary TB- tubercles, T cell immune response secondary TB- live bacilli reactivate, tubercles drain into bronchial tubes, violent cough, green sputum & blood immunocompromised will have false negative test result treatment: antibiotics, 6-24 months combination therapy observed therapy lots of resistance BCG vaccine in countries outside US

Enlargement of lymph nodes or spleen is often associated with tularemia. brucellosis. plague. gastritis. tularemia, brucellosis AND plague.

tularemia, brucellosis AND plague. p.669 - plague - characteristically develop significantly enlarged and tender lymph nodes called BUBOES. p.675 - tularemia - regional lymph nodes enlarge p.676 - brucellosis - patients complain of mild fever, sweating, weakness, aches and pains, enlarged lymph nodes...

The cytokine released from macrophages that seems to play a major role in endotoxic shock is macrophage factor. tumor necrosis factor. protein A. interferon.

tumor necrosis factor. It's a pro-inflammatory cytokine. The host immune response is activated by TNF-α, as is the subsequent release of cytokines following trauma and infection.

Nitrogen is ultimately used in making amino acids and nucleotides AND an important constituent of carbohydrates. turned into ammonia by microorganisms AND ultimately used in making amino acids and nucleotides. primarily fixed by fungi AND ultimately used in making amino acids and nucleotides. turned into ammonia by microorganisms AND ultimately used in making bacterial capsules. an important constituent of carbohydrates AND primarily fixed by fungi.

turned into ammonia by microorganisms AND ultimately used in making amino acids and nucleotides.

Anoxygenic phototrophs

use hydrogen sulfide or organic compounds as a source of electrons.

Coccidioides immitis

valley fever dimorphic fungus, coccus true pathogen heterotrophic caused by inhaling spore(s) or open wound contact 60% inapparent; 40% flu-like symptoms resolves itself, can also use antifungal meds

A common viral rash of childhood with the popular name chickenpox is also known as: A) bariola. B) rubella. C) rubeola. D) varicella-zoster. E) salmonella.

varicella-zoster. When/if it reactivates from latency as shingles it is then called herpes-zoster.

Rotavirus

virus, viral gastroenteritis - especially concerning in children and infants


Set pelajaran terkait

Reccomended Music tempo for Group Exercise formats

View Set

Computer Test lockdown browser 🤮

View Set

Chapter 17 - The Short-Run trade off between Inflation and Unemployment

View Set

Survey of the Bible Unit 4 AND 5

View Set

The Help Study Guide (Chapters 1-20)

View Set